You are on page 1of 354

1

2
3
4
5
6
7
8
9
10
11
12
13
14
15
16
17
18
19
20
21
22
23
24
25
26
27
28
29
30
31
32
33
34
35
36
37
38
39
40
41
42
43
44
45
46
47
48
49
50
51
52
53
54
55
56
57
58
59
60
61
62
63
64
65
66
67
68
69
70
71
72
73
74
75
76
77
78
79
80
81
82
83
84
85
86
87
88
89
90
91
92
93
94
95
96
97
98
99
100
101
102
103
104
105
106
107
108
109
110
111
112
113
114
115
116
117
118
119
120
121
122
123
124
125
126
127
128
129
130
131
132
133
134
135
136
137
138
139
140
141
142
143
144
145
146
147
148
149
150
151
152
153
154
155
156
157
158
159
160
161
162
163
164
165
166
167
168
169
170
171
172
173
174
175
176
177
178
179
180
181
182
183
184
185
186
187
188
189
190
191
192
193
194
195
196
197
198
199
200
TH E SAT WRITI N G AN D
LAN G UAG E TE ST: TH E
TEN ESSENTIAL RULES

l. Don't Sweat the S m a l l Stu ff 1 59

2. Strengthen the Core 1 61

3. O rg a n ize the Ideas i n Yo u r Pa ra g ra p hs 1 66

4. U se Pa ra l le l Structu re 1 72

5. U se Mod ifiers E ffective ly 1 76

6. M a ke You r Com pa risons C l e a r a n d Prec ise 1 82

7. Ma ke S u re You r Pro n o u n s Are C l e a r a n d Prec ise 1 85

8. M a ke You r Verbs C l e a r a n d Prec ise 1 92

9. Ma ke the Rest of You r Sentence C l e a r a n d Precise 1 96

1 0. Know H ow to P u n ctuate 207

1 57

201
1 58 McGraw-Hill Education: SAT

The SAT Writing and Language Test

What is the SAT Writing and Language test?

The SAT includes a 35-minute Writing and Language test designed to assess your
proficiency in revising and editing a range of texts in a variety of content areas, both academic and career
related, for expression of ideas and for conformity to the conventions ofStandard Written English grammar,
usage and punctuation.
The Writing and Language test consists of four passages, each 400-450 words long, i n the categories of careers,
social studies, humanities, and science. ( For an example of the Writing and Language test, look at Section 2
of the Diagnostic Test i n C hapter 2 .) You are to analyze underlined portions of each passage and to determine
whether they need to b e revised according to the standards of
parallel structure
verb, modifier, and pronoun agreement
standard idiom
logical comparisons
word choice
verb tense, mood, and voice
logical transitions
coordination of ideas
punctuation
You are also asked more general editorial questions, such as
whether a certain sentence adds to or detracts from the cohesiveness of a paragraph
where a new sentence should be placed for maximum effectiveness
whether a particular passage or paragraph has the effect the author intends

How is it used?

Colleges use you r SAT Writing and Language test score as a measure of your ability to write clearly and effec­
tively. Good writing skills are essential to success in the liberal arts and sciences. The Writing and Language
test score represents one-half of your Evidence-Based Reading and Writing Score. The other half of this score
comes from the Reading test.

Sound intimidating? It's not.

There are really only 10 rules to learn i n order to ace the SAT Writing and Language test, and the 33 lessons in
this chapter will give you the knowledge and practice you need to master all of them.

202
CHAPTER 4 I THE SAT WRITING AND LANGUAGE TEST: THE TEN ESSENTIAL RULES 1 59

Ru le 1 : Don 't Sweat the Sma l l Stuff


Lesson 1 : Know the seven things to NOT worry about

1 . Don't worry about split i n fi nitives B. Second Federal is the only bank in town that does
not finance commercial mortgages.
Which is correct?
Technically, sentence B is correct because the phrase that
A. Here are seven things to not worry about.
does not finance commercial mortgages is a "restrictive
B. Here are seven things n o t to worry about.
clause," that is, it modifies the noun bank by attaching a
Sentence A includes a split infinitive: the infinitive to defining characteristic to it. I f a modifying clause is "restric­
worry has a n adverb (not) wedged inside it. Although tive" (that is, it conveys defin ing information ab o ut the
the SAT probably won't test your skill for "unsplitting" noun), it should use that. Alternately, if the clause is "non­
infinitives, you should still do it as a matter of politeness restrictive" (that is, it conveys incidental or nondefining
to the grammar scolds, for whom they are the verbal information about the noun), it should use which. Helpful
equivalent of chewing aluminum foil. You can usually tip: nonrestrictive modifying clauses are almost always
just shift the adverb over a little bit, as i n sentence B, and preceded by a comma, as in The speech, which lasted only
make everyone happy. three minutes, secured her reputation as a master orator.
But sometimes it's not so easy to unsplit infinitives Bottom line: the SAT will probably not expect you to
without destroying the tone or meaning of the sentence. distinguish restrictive from nonrestrictive clauses, so
For instance, try unsplitting the infinitive in The company don't stress out about that versus which on the SAT.
plans to more than double its revenue next year. Or, better
yet, j ust don't worry about it, since it won't be on the SAT. 4. Don't worry about starting sentences with Because,
And, or But
2. Don't worry (too much) about who vs. whom Which is correct?
Which is correct? A. Because we don 't know when Jennie will arrive,
A. To who should I give your condolences? we can 't make dinner reservations yet.
B. To whom should I give your condolences? B. We can 't make dinner reservations yet because
we don 't know when Jennie will arrive.
The who/whom distinction is the same as the he/him
and they/them distinction: the first pronoun in each pair Ms. Bumthistle (everyone's fifth grade English teacher)
has the subjective c a s e (Lesson 21),and so is used as probably told you that it's a cardinal sin to start a sen­
the subject of a verb, and the second has the objective tence with Because, A nd, or But. But it's not nice to lie to
case, and so is used as the object of a verb or preposition. children. I n fact, either sentence above is fine. The SAT
Since the pronoun i n the sentence above is the object of frequently includes perfectly good sentences that start
the preposition to, sentence B is correct. with Because. But if you want to avoid annoying the
Notice, however, that the pronoun you can be used Ms. Bum thistles of the world, avoid the practice in your
as either a subject or a n object. It represents a "merger" own writing if it's not too much trouble.
between the subjective thou and the objective thee 5. Don't worry about d isappearing thats
from Elizabethan English. (Remember Shakespeare?)
Likewise, whom seems to be in the process of merging Which is correct?
with who. For instance, even Standard English allows a A. I really love the sweater you gave me.
sentence like Who are you talking to? rather than insisting B. I really love the swea ter that you gave me.
on the rather uptight-sounding To whom are you talking?
The bottom l ine? Chances are, your SAT won't ask Both of the sentences above are acceptable in Standard
you to choose between who and whom. But if it does, just Written English. So, if that isn't necessary, why would
remember that the who/whom distinction is the same as we ever include it? Because it takes some of the burden
the they/them and he/him distinctions. And if you're still away from sweater, which is a n object in the first clause
stuck, just go with who. (I really love the sweater) as well as an object of the second
clause ( You gave me [the sweater}). By i ncluding that, we
3. Don't worry about that vs. which separate the two ideas more clearly. But since very few
people are confused by the dual role of sweater in the
Which is correct?
first sentence, that is not strictly necessary.
A. Second Federal is the only bank in town which Bottom line: don't worry about a missing that, as long
does not finance commercial mortgages. as the resulting sentence still makes sense.

203
1 60 McGraw-Hill Education: SAT

6. Don ' t worry about "parallel ellipsis" Here, performed i s an action verb. Any word that modi­
fies the manner of a n action verb i s a n adverb. Since good
Which is correct?
cannot function as an adverb i n Standard English, only
A. The Republicans reacted to the speech with choice B i s correct.
sustained applause; the Democrats, however,
Which is correct?
reacted to it with studied silence.
B . The Republicans reacted t o the speech with sus-
C . I don 't feel good.
tained applause; the Democrats, studied silence.
D. I don 'tfeel well.
Both of the sentences above are grammatically correct.
Here, feel is a linking verb rather than an action verb:
Sentence B, however, is more concise because it takes
that is, it links the subject to an essential adjective,
advantage of "parallel ellipsis." Ellipsis simply means
as in The sky is blue. So does this mean that C i s right
the omission of words that are i mplied by context. I n this
and D is wrong? No-they are both grammatically and
case, the parallel structure of the two clauses allows the
semantically correct, since well can also act as a n adjec­
reader to "fill in" the m issing words .
tive, meaning "in good health." The two sentences are
When you read a sentence like B, you might think
essentially equivalent to I a m not [feeling} good and I a m
that the missing words are a grammatical mistake. But
not well.
i f the context clearly implies the missing words, you can
leave them out. Which is correct?
You might notice that, i n sentence B, the comma
E. I feel badfor you.
plays a n unusual role. Usually, commas are used to sepa­
F. I feel badly for you .
rate items in a list, to separate modifying phrases from
clauses, or (with conjunctions) to separate clauses. Here, Here, despite what your know-it-all friends m ight say, E
however, the comma is analogous to the apostrophe in is correct and F is wrong, since badly can only function
can't: just as the apostrophe holds the place of the miss­ as an adverb. Saying I feel bad for you i s like saying I feel
ing letters from cannot, so the comma in sentence B holds sorryfor you. You wouldn't say Ifeel sorrilyfor you, would
the place of the missing words (however, reacted to it with) you?
from sentence A. Without that comma to suggest the It's important to know the difference between adjec­
ellipsis, the sentence would sound very strange indeed. tives and adverbs (Lesson 14), and between action verbs
and linking verbs.
7. Don't worry (too much) about good versus well or But the SAT is probably not going to ask you about
bad versus badly good versus well or bad versus badly.
Which i s correct?
A. Peter performed good.
B . Peter performed well.

204
CHAPTER 4 I THE SAT WRITING AND LANGUAGE TEST: THE TEN ESSENTIAL RULES 1 61

Ru le 2: Strengthen the Core


Lesson 2: Identify you r clauses, modifiers,
j
and con u nctions

The first and most i mportant step i n analyzing sentences is identifying clauses.
Every sentence contains at least one clause, which consists of a subject and a predicate. The subject i s the
noun or pronoun that is "doing" the verb, and the predicate consists of a verb and its complements (such as
direct obj ects, indirect objects, verb modifiers, or predicate adjectives).
The subject-verb unit of any clause conveys the core idea of that clause. For instance, if we take the sentence
As the sun slowly set, the desperation of the sailors revealed itself in their sullen glances.

and isolate j ust the subject and verb, we still retain the core idea:
The desperation revealed itself.

Consider these two sentences: Introduction to Organic Chemistry . . . introduces stu­


dents to important experimental methods at the heart
A. Go!
today 's most promising areas of medical research.
B . A lthough generally regarded a s the most daunt­
ing course in the undergraduate science cur­
These two clauses are l inked by a conj u nction
riculum, Introduction to Organic Chemistry
phrase (not only . . . but also), and are preceded by a
not only provides a necessary foundation in the
subord inating conjunction (Altho ugh) fol lowed by a
principles of physical chemistry, but also intro­
modi fying {participial) phrase (generally regarded
duces students to important experimental meth­
as the most daun ting course in the undergraduate sci­
ods at the heart of today's most promising areas
ence curriculum) . We' l l talk more about conj u nctions
of medical research.
i n Lessons 6 and 1 0 and about participial phrases in
Sentence A is the shortest i n the English language. It has Lesson 1 2 .
everything necessary to convey a complete thought: a So here's how you should analyze sentence B :
verb (go) and its subject (the implied subject you). Since
it i s i n the i mperative mood (Lesson 30), the subject is [Although] [generally regarded a s the most daunt­
assumed to b e the person being addressed and does not ing course in the undergraduate science curriculum],
need to b e stated . [Introduction to Organic Chemistry] [not only]
So here's h o w w e can analyze sentence A: [provides a necessaryfoundation in the principles
of physical chemistry], [but also] [introduces stu­
[You] [go}! dents to important experimental methods at the
[Implied subject] [verb] ! heart today's most promising areas of medical
research.]
[Subordinating conj unction] [participial phrase],
Sentences can also elaborate the main clause
[subject] , [conjunction part l] [predicate 1) [conj unc­
with modifiers or link clauses with conjunctions.
tion part 2 ] [ predicate 2] .

Sentence B is a bit more complicated. The main I f this analysis seems confusing now, don't worry.
clause i ncludes a compound predicate, so it combines We'll explain all of these terms in the lessons to come.
two statements with the same subject i nto one sentence: For now, focus on identifying clauses (the words i n bold
i n the sentence above) because they are the core of any
Introduction to Organic Chemistry . . . provides a sentence. Distinguishing clauses from the rest of the
necessary foundation in the principles of physical sentence is the first step to becoming a stronger reader
chemistry . . . and writer.

205
1 62 McGraw·Hill Education: SAT

Lesson 3 : Tri m every sentence to a nalyze its core

Consider this sentence: My chief concern with this budget, which Juts othe1
wise been well consitle1 eel,are the drastic cuts irt
My chiefconcern with this budget, which has otherwise schoolfttntls.
been well considered, are the drastic cuts in school
funds. Step 3: Cross out any other nonessenti a l modifiers.

How does it sound? It may sound a little bit off, but Once you learn to ident i fy p a r t icipial phrases
why, and how do we improve it? Th is is where trimming (Lesson 1 2), appositives (Lesson 1 3), and more mun­
comes in. dane modifiers l i ke adj e c t ives and adverbs (Lesson
14), you can trim them from all of you r sentences, as
well, with one exception: predicate adj ectives, such as
Diagnosing and improving sentences requires tired in the sentence Karen was tired, without which the
m astering the skill of trimming: reducing the sentence doesn't convey an idea. I n our sentence, chief
sentence to its core, or its essential elements, and drastic can go:
then analyzing that core .
My chief concern with this btttlget, whicfi htts otlu1 -
This is based on a very important rule of gram­
wise been well consitle1 eel, are the tirttstic cuts in school
mar: every sentence must "work" even when its
prepositional phrases, interrupters, and other ftmtis .
modifiers are eliminated. That is, it still must
So now we have the core:
convey a grammatically complete idea.
My concern are the cuts.
Step 1 : Cross out all nonessential prepositional
phrases. Obviously, the subject and verb d isagree (Lesson 4):
concern is a singular subj ect, but a re is a plural verb. So
A preposition is any word that can be used to complete you may just want to cha nge the verb: My concern is the
any sentence like these: cuts. But that's no good either, because now the sentence
Th e squirrel ran the tree. (e.g. up, to, around, has a number shift (Lesson 1 1) : the singular concern is
from, in, by, on, in to, etc.) equated with the plural cuts.
These problems point to an even deeper problem:
I went to the party a brain surgeon. the most essential part of the sentence, the verb, is very
(e.g., as, with . for, etc.) weak. Forms of the verb to be, 1 i ke is, are, was, and were,
Democracy is government the people. are among the weakest verbs in English .
(e.g. Jor, of, by, etc.)

A prepositional phrase is the preposition plus the To improve your writing, first focus on streng th­
noun phrase that follows it, such as from sea to shining enin g and clarifying your verbs.
sea, in the beginning, and for the money.
Our sentence has two nonessential prepositional
phrases that we can elimi nate:
This sentence is clearly i ndicating disapproval, so a
My chief concern with this budget, which has other­ more personal subject like I and a strong verb of
wise been well considered, are the drastic cuts irt disapproval like object would strengthen the sentence:
schoolJuntis.
A lthough the budget is otherwise well considered,
Step 2: Cross out all i nterrupting modifiers. I object to its drastic cuts in school funds.
Interrupting modifiers are generally easy to spot
because they come between commas or dashes. The Notice that this revision not only corrects the
sentence should always hold together even when the grammatical problems, but it also makes the sentence
interrupting modifiers are removed: stronger, clearer, and more concise.

206
CHAPTER 4 I THE SAT WRITING AND LANGUAGE TEST: THE TEN ESSENTIAL RULES 1 63

Exercise 1 : Trimming and Strengthening Sentences

Trim each of the following sentences and correct any verb problems.

1. Th e team o f advisors, arriving slightly ahead of 8. The progression of a society, or at least that popularly
schedule, were met at the airport by the Assistant regarded as advancements, are a result of gradual
Prime M i nister. modifications, not sudden drastic overhaul.
Trimmed: _______________

Revised: _______________

2. The flock of birds darting over t h e roiling lake look


like an opalescent whirlwind.

3. Carmen, not to mention her unsympathetic 9. The development of the new country's government
sisters, were unaffected by David's pleas. and social institutions were affected in a negative
regard by the lack of cohesiveness within the revo­
lutionary army.
Trimmed: _______________

4. Juggling the demands of school, family, and work Revised: _______________

often seem too much for a young mother to bear.

5. Others on t h e committee, like chairman Sanders,


is concerned about the lack of attention given to
school safety. 10. This report is i ntended for presenting arguments
in opposition to what I took to b e the less than opti­
mal response of the administration to the most
recent crises i n the M iddle East.

6. Every one of my friends, including the boys, has Trimmed: -----

supported my decision.
Revised: _______________

7. The fact that human institutions have been


responsible for so many atrocities have forced
some historians to adopt a cynical perspective on
human nature.

Tri m each sentence. Then revise it to make it clear


and concise, changing the subject and verb, if
necessary.

207
1 64 McGraw-Hill Education: SAT

Lesson 4: Make s u re you r v with their subiects

Which is correct? I f we trim sentence A, we get


A. Data gathered through polling is not as reliable Data gathe1ed th1 (]ttgh p(]lfing is not as reliable m

as data gathered objectively. dttttt gttthe1 ed (]bjectit:Jely.


B. Data gathered through polling are n o t a s reli­
The subject, data, i s plural, so the verb should b e are.
able as data gathered objectively.
Sentence B is correct.

A few Latin plurals are frequently mistaken for singulars. Don't make that m istake.
Singular Plural C orrect Sentence
bacterium bacteria The bacteria are multiplying rapidly.
continuum continua The continua of space and time are related.
criterion criteria Your criteria are hard to meet.
curriculum curricula The competing curricula were scrutinized.
datum data The data on the drive have been corrupted.
medium media The media have largely ignored this story.
phenomenon phenomena Such phenomena are surprisingly common.

Which is correct? Here, the subj ect and verb are inverted: the subject hours
C. Behind every successful work ofart lies coun tless comes after the verb lies. When we "un-invert" the clause,
hours of toil and trial.
the subj ect-verb disagreement is obvious: hours lies
D. Behind every successful work of art lie countless should be changed to hours lie. Therefore, sentence D is
hours of toil and trial.
correct.

If we trim sentence C, we get


Behind etJe1 y sttccessjttl wm k (Jjtt1 t lies countless hours
(Jft(]i/ ttnd t1 ittl .

An inverted clause, where the verb comes before the subj ect, usually begins with the dummy subject there, as
in There is or There are, or is preceded by a prepositional phrase.
Every inverted clause can be "un-inverted" by removing any dummy subject and rearranging the phrases.
Un-inverting these sentences will help you to spot any subject-verb disagreements.
Inverted: [There] [are] [over twenty applican ts] [applyingfor thejob] .
Un-inverted: [Over twenty applican ts] [are] [applyingfor the job] .
Inverted: [Behind every successful work of art] [lie] [countless hours of toil a n d trial] .
Un-inverted : [Countless hours of toil and trial] [Lie] [behind every successful work of art] .

Which is correct? proximity: the essential noun (that is, not one in a prepo­
E. One or two ofmy classmates has a strong chance
sitional phrase) that is closer to the verb gets priority. Here,
of winning an award.
since two is closer to the verb, the subject is regarded as
F. One or two ofmy classmates have a strong chance
plural. Therefore, sentence F is correct.
of winning an award.
If we trim sentence E, we get If a subject takes the for m a or b, it is assumed to
One or two rif my clttssmtttes has a strong chance fJj take the number of b.
winning ttn ttwtt1d.
Is the subject, One or two, singular or plural? In these
ambiguous situations, it helps to remember the law of

208
CHAPTER 4 / THE SAT WRITING AND LANGUAGE TEST: THE TEN ESSENTIAL RULES 1 65

Exercise 2: Subiect-Verb Agreement

Choose the correct verb form.

1. The flock o f geese (was/were) startled b y the shot­ 13. Every player on b o t h teams (was/were) a t the press
gun blast. conference after the game.

2. Th e data o n m y computer (was/were) lost when the 14. There (has/have) been a theater and a toy store in
hard drive failed. the mall ever since it opened.

3. Neither of t h e twins (is/are) allergic t o penicillin. 15. There (is/are) a great many production problems t o
iron out before show time.
4. M u c h of what I hear i n those lectures (go/goes) i n
o n e ear and o u t t h e other. 16. The proceeds from the sale of every auctioned item
(goes/go) to charity.
5. Amy, like h e r friends Jamie a n d Jen, (wants/want)
to go to Mount Holyoke College. 17. There (is/are) more than three years remaining on
her contract.
6. Among the lilies and wildflowers (were/was) one
18. Neither of the girls (was/were) frightened by the
solitary rose.
small animals that scurried past their tent.
7. Either the chairperson or her assistants (is/are)
19. This technology, developed by t h e military for field
going to have to make the decision.
communications, (have/has) become essential to
private i ndustry as well.
8. There (is/are) hard ly even a speck of dirt left on the
carpet.
20. Every player on both teams (was/were) concerned
about the goalie's i nj ury.
9. In every teaspoon of soil (are/is) over two million
tiny m icroorganisms. 21. The company's sponsorship o f mentorship pro­
grams (has/have) garnered many accolades from
10. There (is/are), i n my opinion, far too few primary other philanthropic organizations.
physicians working i n this district.
22. Neither the children nor their parents (utter/
11. B eyond that hill (is/are) hundreds o f bison. utters) a word when Mrs. Denny tells her stories.

12. Never before (have/has) there been such voices 23. How important (is/are) strength training and car­
heard on the public airwaves. diovascular training to your daily fitness regimen?

209
1 66 McGraw-Hill Education: SAT

Ru le 3 : Orga n ize the Ideas i n You r Pa ra g ra ph s


Lesson 5 : Present your id a s coh s iv ly
and with a consistent ton

What's wrong with this paragraph? The paragraph starts off well, with a clear topic sen­
tence about the politics of fracking. It then gives a quick
The politics of hydraulic fracturing, or "fracking," summary of the two positions on the topic. With the
have obscured both the dangers and the benefits of last sentence, however, the paragraph begins to lose its
this new technology. Opponents suggest that the focus and tone: the phrase pretty small, really i s too con­
high-pressure fluid used to fracture deep rock forma­ versational for the tone of this paragraph, and the ideas
tions may contain carcinogens that may seep into in the last sentence are not tied logically to the ongoing
groundwater, and thatfracking induces earthquakes. discussion. Here's a revision that more effectively links
Supporters point out that this activity is taking place to the previous sentence:
well below even the deepest aquifers and is well sealed
They also point out that the seismic activity induced
offfrom human water supplies. The technical term for
by/racking is minimal: the vast majority of thefrac­
earthquakes is seismic activity, and the fractures are
tures it induces are less than 1 millimeter wide.
pretty small, really: only about 1 millimeter or less.

Every effective prose paragraph should


be focused on a topic sentence that develops the central idea of the passage
explain or illustrate any significant claims
avoid irrelevant com mentary
maintain a consistent and appropriate tone

210
CHAPTER 4 / THE SAT WRITING AND LANGUAGE TEST: THE TEN ESSENTIAL RULES 1 67

Lesson 6: Coordi nate you r clauses effectively


and avoid com mas splices

Which is better? . . . Brian Greene is a professor of physics . . .

A. Despite being a best-selling author, Brian Greene . . . [Brian Greene} is also cofounder of the World Sci­
is a professor of physics, he is also cofounder of ence Festival . . .
the World Science Festival, and this event dra ws . . . {the World Science Festival} draws nearly half a
nearly half a million people each year. million people each year . . .
B. Cofounded by best-selling author and profes­
So the reader is left confused: what is the central idea
sor of physics Brian Greene, the World Science
of this sentence? Brian Greene's professorship? His fes­
Festival draws nearly half a million people each
tival? The popularity of the festival? Even worse, the
year.
preposition Despite doesn't make sense, since being a
It's not too hard to see that sentence B seems clearer best-selling author doesn't i nterfere i n any obvious way
and more logical than sentence A, but why? The answer with being a physics professor.
is coordination . Both sentences contain the same four Sentence B, i n contrast, packages these ideas to
ideas, but sentence B coordinates those ideas more make them easier to digest. The first two ideas are sub ­
effectively. Sentence A contains three i ndependent ordinated i n a participial phrase, and the third idea i s
clauses: emphasized as t h e independent clause.

I n a well-coordinated sentence,
• the central idea is expressed i n the main independent clause
secondary ideas are expressed in subordinate clauses or modifying phrases
• ideas are l inked with logically appropriate conjunctions, prepositions, and adverbs

Notice also that the second comma in sentence A is a comma splice, joining two independent clauses.
That's a no-no.

Avoid comma splices. A comma splice i s the error of joining two independent clauses with only a comma:

Comma splice (wrong): We were having a grea t time, T.]. played his guitar.

Independent clauses can be joined in one sentence in one of three acceptable ways:

Comma-conjunction: We were ha ving a great time, but T.J. played his guitar.
Semicolon: We were ha ving a great time; T.]. played his guitar.
Colon: We were having a great time: T.]. played his guitar.

Semicolons are used to join two ideas when the second supports or extends the first. Colons are used to join
two ideas when the second explains or specifies the first. The first sentence indicates that T.J.'s guitar didn 't
help the mood; the second indicates that T.J.'s guitar didn 't hurt the mood; the third indicates that T.J.'s guitar
explained the mood.

211
1 68 McGraw-Hill Education: SAT

Exercise 3: Coordinating Clauses

Join each set of sentences i nto a single well-coordinated sentence.

1. H . K. Schaffer's latest movie h a s received wide­ 2. Scientists h ave m a d e an important discovery. The
spread critical acclaim. She directed the movie. It scientists who made the discovery are a team from
i s the third movie that she has directed. She is the universities and research institutions from all over
daughter of famous screenwriter George Schaffer. the world. The discovery concerns a region of the
Her latest movie i s a comedy entitled The Return . brain called the prefronta l cortex. The scientists
have discovered that this region governs impulse
control i n humans. Studying this region of the
brain can help scientists learn more about crimi­
nal behavior.

Rewrite the following sentences so that the clauses coord inate logica lly a n d concisely.

3. Electric cars m ay not b e as environmentally 5. W e a r e motivated by our principles; our principles


friendly as they are widely regarded, so the elec­ change all the time, though: our experiences and
tricity they use actually comes from fossil fuels; our priorities evolve as we grow.
that electricity i s produced i n power plants that
often burn coal or other fossil fuels and that burn­
ing often produces enormous amounts of green­
house emissions.

4. Although regular exercise is good for your mus­


cles, it i s also good for your heart, so it is good for
your brain too by keeping it well oxygenated and
the increased oxygenation helps it work more
efficiently.

212
CHAPTER 4 / THE SAT WRITING AND LANGUAGE TEST: THE TEN ESSENTIAL RULES 1 69

Lesson 7: Give you r rea der helpfu l transitions, espec i a l ly


betwee n paragraphs
Consider this transition between paragraphs: the new paragraph connects with previous one. Consider
this revision:
. . . a n d so w e should b e respectful of other people, even
those with whom we disagree, while always striving to . . . and so we should be respectful of other people, even
eliminate inequities and abuses of power. those with whom we disagree, while always striving to
To Kill a Mockingbird was written by Harper Lee and eliminate inequities and abuses of power.
published in 1960. It portrays the fictional town of In To K i l l a Mockingbird (1960), Harper Lee depicts a
Maycomb, Alabama . . . fictional town, Maycomb, A labama, that is tainted by
such inequities and abuses . . .
The end of the first paragraph makes a bold claim: that we
should strive to eliminate inequities and abuses of power. Now we understand the reference better because the
But the next p aragraph abruptly shifts to mundane facts author has provided a helpfu l paragraph transition.
about the publication of a particular book. Although The phrase such inequ ities and abuses demonstrates
readers who are familiar with To Kill a Mockingbird clearly that the events i n To Kill a Mockingbird will i l lus­
might have an idea why this author is mentioning it, the t rate the importance of fighting i nequities and abuse,
author does not provide a ny helpful transitions to guide and therefore exemplify the thesis from the previous
the reader into the new p aragraph and indicate how paragraph.

P rovide your readers with helpfu l paragraph transitions to clarify the links between topic ideas. Keep i n m i nd the
common transitional words and phrases below.
To extend an idea
indeed furthermore moreover in fact
further also beyond that additionally
To illustrate or specify an idea
for example for instance in particular namely
such as especially to illustrate specifically
To make a comparison or contrast
similarly likewise actually nevertheless
however although despite on the other hand
To show consequence
as a result so thus subsequently
therefore hence accordingly for this reason
To provide explanation or reason
this is because since thus the reason is that
how because why as

213
1 70 McGraw-Hill Education: SAT

Lesson 8 : Ma ke ou r c ro s s - re fe re n c e s c l ea r

Consider these sentences from our "fracking" essay: noun i n the first sentence, namely. fracking (or, equiva­
lently, process). The last sentence also i ncludes a cross­
. . . The opponents offracking are correct to ask ques­ referencing pronoun, this. But to what does it refer? It
tions about the safety and sustainability of this pro­ doesn't seem to b e referring to fracking anymore; that
cess. Could it poison the local water supply with wouldn't make sense. Nor does it m a ke sense to refe r
carcinogens? Can we spare the vast amount of injec­ t o t h e other singular nouns i n previous sentences, l i ke
tion water it requires? Can we safely recycle its waste­ methane, water supply, or seismic activity. S o the reader
water? Could it be introducing more methane into the may b e left a bit confused. Here, we need to revise to
water supply than would naturally be present? Could it clarify the cross-reference:
be causing potentially dangerous seismic activity? But
this also must be followed by careful, scientific, and But this questioning must be followed by careful,
impartial investigation, not mere fear-mongering. scientific, and impartial investigation, not mere
fear-mongering.
Each of the five questioning sentences contai n s
the pronoun it, which m a k e s a "cross-reference" to a

When referring to concepts i ntroduced in previous sentences, using pronouns will often help you be concise,
but make sure your cross-references are clear. Sometimes clarity may require you to replace the "cross­
referencing" pronouns with more precise nouns.

214
CHAPTER 4 I THE SAT WRITING AND LANGUAGE TEST: TH E TEN ESSENTIAL RULES 1 71

Exercise 4: Effective Transitions and Cross- References

Rewrite the second sentence in each pair, providing an effective transition and
clarifying a ny cross-references.

1. . . . Modern biologists have tried for decades to 3. . . . It's easy to u nderstand, i n a society as complex,
explore the relationship between ancient humans diverse, and bureaucratic as ours, how some citi­
and Neanderthals, but analyzing DNA from pre­ zens could develop a deep distrust of governmen­
historic hominids has until recently proven very tal institutions.
difficult.
The willingness to equate all governmental
The "clean room" at the Max Planck Institute i n institutions with tyranny is an enormously dan­
Germany is l i ke t h o s e used i n t h e manufacturing gerous one that can only impede human moral,
of computer chips or space telescopes, solving the economic, and cultural progress.
issue by preventing contamination from dust par­
ticles so that biologists can extract and examine
m i nute bits of DNA from 400,000 -year-old homi­
nid bones.

2. . . . As satisfying as it may b e to punish wrongdoers,


the real impetus behind tough sentencing laws is
the belief that they actually deter crime.
The treatment so many prisoners receive i n state
and federal penitentiaries, including humiliation
and loss of autonomy, only exacerbates any short­
or long-term psychological issues that make them
susceptible to antisocial and criminal impulses,
according to evidence.

215
1 72 McGraw-Hill Education: SAT

Ru le 4: U se Pa ra l lel Structu re
Lesson 9: U ndersta nd th Law of Pa ra l l lism

Which is better?
The Law of Parallelism
A. In the '70s and '80s, high school math teach­
ers taught almost exclusively by lecture; today, When a sentence includes a list, contrast, or
more coopera tive and project- based methods comparison, the items being listed, contrasted,
are likely to be employed. or compared should have the same grammati­
B. In the '70s a n d '80s, high school math teachers cal form.
taught almost exclusively by lecture; today, they
are more likely to use cooperative and project­
based methods.
Sentence A contains two clauses that contrast teach­
Which is better? ing in the '70s and '80s with teaching today. However,
C . Ms. Kelly always tried to provide clear instruc­ the comparison is not parallel: the first sentence is
tions that showed respect and were fair to all of
in the active voice, but the second is in the passive voice
( Lesson 29). Sentence B reads more smoothly because
her students.
D. Ms. Kelly always tried to provide instructions
both clauses are in the active voice, which aligns the sub­
tha t were clear, respectful, and fair to all of her
jects and clarifies the contrast.
students.
Sentence C ascribes three adjectives to Ms. Kelly's
instructions, but not in a parallel form. Sentence D clari­
Sentences A and C don't seem glaringly wrong, but B and fies the central idea by putting these adjectives i n a clear
D sound a bit better. Why? Parallelism. and parallel list.

216
CHAPTER 4 I THE SAT WRITING AND LANGUAGE TEST: THE TEN ESSENTIAL RULES 1 73

Lesson 1 0: Watch for standa rdJ?;��allel constructions

Which is b etter? The word rather indicates that the sentence is making
a contrast. Such a contrast requires a standard par­
A. It seems sometimes that our representatives
allel construction: ra ther X than Y. When you see the
would rather generate sound bites for their par­
word rather, you should expect the word than to appear
tisans instead of working to solve our social and
soon afterward. But i n sentence A, not only does than
economic problems.
not appear, but the two words from X and Y that should
B. It seems sometimes that our representatives
be parallel are not: generate i s a present-tense verb, but
would rather generate sound bites for their
working is a gerund. Sentence B makes the correction,
partisans than solve our social and economic
and creates the parallel construction ra ther gen erate . . .
problems.
than solve.
The problem in sentence A is hard for most readers to
catch. It may take a few readings before you notice it.

Use the following standard parallel constructions precisely. When you use any of these phrases, use the
precise wording, and make sure X and Y are parallel.
rather X than Y X more than Y neither X nor Y X is like Y
prefer X to Y either X or Y both X and Y the more X, the more Y
less X than Y not so much X as Y not X but Y the better X, the better Y

217
1 74 McGraw-Hill Education: SAT

Lesson 1 1 : Avoid number shifts

If a sentence equates two things, those things should have the same number.
Which is b etter? If we trim it a bit, we get

A. Everyone enjoyed their meal. The problem with this pltln is all of the permits we
B . Everyone enjoyed his or her meal. would have to file befm e sttl1 ting the p1<Jject.
C . They all enjoyed their meals.
Again, we have a number shift: the singular problem is
Sentence A commits a number shift: the pronoun their equated with the plural all of the permits. We could try to
is plural, but its antecedent everyone is singular. fix the problem by pluralizing the subj ect:
Additionally, the object meal is singular, which doesn't
make sense-are multiple people sharing a single meal? The problems with this plan are all of the permits we
One way to correct this problem is by changing their to would have to file before starting the project.
the singular his or her, as i n sentence B. But this phrase is
But that sounds very strange. The best revision strength­
needlessly awkward. Sentence C avoids both problems,
ens the verb to avoid the number shift:
so it is the best of the three.
Consider this sentence: Filing all of the permits required by this plan will
T h e problem with this plan i s all of the permits we probably delay the project.
would have to file before starting the project.

218
CHAPTER 4 I THE SAT WRITING AND LANGUAGE TEST: THE TEN ESSENTIAL RULES 1 75

Exercise 5: Parallel Structure

Rewrite each sentence to improve its para l lel structure.

1. Th e candidate's platform i ncluded tax code reform, 6. Ms. Bennett is appreciated by her colleagues
an improved school system, and reviving good because she is very supportive and has a lot of
relations with the unions. knowledge.

2. G o o d study practices a r e not so much about work­ 7. I can't decide whether I should give Maria the
ing hard, but rather how well you use your time. tickets, or Caitlyn.

3. The more you get to know her, the more likely it is 8. The United States experienced a contraction i n
that you will like her. wealth, an increase i n r i s k spreads, and t h e credit
markets were deteriorating.

4. The food here is not only exceptionally fresh, but


its price i s also very reasonable. 9. I prefer the romantic virtuosity of L iszt, as opposed
to Chopin's emotional accessibility.

5. The financial crisis of 2007 was exacerbated by the


esoteric nature of certain financial instruments, 10. The festival draws crowds from across the coun­
skittish i nvestors, and the lack of awareness of try that come not so much for the music but rather
regulators. because of the spirit of free expression.

219
1 76 McGraw-Hill Education: SAT

Ru le 5 : Use Modifiers Effectively


Le son 1 2: Don't I t your participl s dansl

Which is correct? She broken the plate.

A. Widely considered one of the most challenging We thinking about you.


piecesfor piano, Franz Liszt stretched the bound­
aries of m usical technique with his Etude no. 5. Each participle requires a helping verb to complete the
B. Widely considered one the most challeng­ verb phrase and make a sensible clause:
ing pieces for piano, Franz Liszt's Etude no. 5
stretches the boundaries of musical technique. She has broken the plate.
We were thinking about you.
Sentence A includes a dangling participle. The past
participle considered requires a subject. Since parti­
Present participles like eating, fighting, and inter­
cipial phrases don't include their own subj e cts, they
rupting always end in - ing. Past participles, however,
must " borrow" them from the main clause. What is
fall under two categories: "regular" past participles like
the subject of the participle? That is, what, exactly, is
straightened and pushed end in - ed, but "irregular" past
considered one of the most challenging pieces for piano?
p articiples can take many forms, like fought, been, eaten,
Surely not Franz Liszt-he is the composer. It is Etude
no. 5. Because the subject of the main clause should
swum, and seen. For a list of some common irregular
forms, see Lesson 25.
also b e the subject of the p articipial phrase, the correct
I n English, we use present p articiples (with the help­
choice is B.
ing verb to be) i n verbs with the progressive aspect
(Lesson 23), such as I am eating and I had been eating.
When a participial phrase begins a sentence, its We use past participles (with the helping verb to have) i n
subject should be the subject of the main clause verbs with t h e consequential aspect (Lesson 23) such as
that follows. Otherwise, it is called a dangling I have eaten and I had eaten.
participle. When participles appear without their helping verbs,
they act as adjectives, and their phrases are called parti­
cipial phrases. Here are some more examples:
What are participles, a nyway?
When designing a user interface, software engineers
Participles are verb forms, l ike broken and thinking,
should focus on simplicity.
that cannot stand by themselves as verbs . They are only
part of the verb, hence the name "participle." Notice, for Although pleased with her victory, A ngela knew tha t
instance, that we can't say s h e still h a d more work t o do.

220
CHAPTER 4 I THE SAT WRITING AND LANGUAGE TEST: THE TEN ESSENTIAL RULES 1 77

Lesson 1 3: Know where to place you r modifiers

Which is correct? Which is correct?


A. In an emergency, I am amazed at how calm E. To illustrate his point, we watched Mr. Genovese
Marco can be. take out a giant boa constrictor.
B. I am amazed at how calm Marco can be in an F. We watched Mr. Genovese take out a giant boa
emergency. constrictor to illustrate his point.

What does the prepositional phrase in an emergency What does the infinitive phrase to illustrate his point
modify? It answers the question When can Marco be calm? modify? It answers the question Why did he take it out?
rather than When can I be amazed? Since it modifies the rather than Why did we watch it? Since it modifies the
second verb rather than the first verb, B is the better choice. second verb rather than the first, choice F is correct.

Any modifier or modifying phrase should be


placed as close (or "proximate" ) as possible to
the word it modifies without disrupting the
sentence. This is called the Law of Proximity.
Modifiers or modifying phrases that violate this
rule are called misplaced modifiers.

Which is correct?
C. A splendid example of synthetic cubism, Picasso
painted Three Musicians in the summer of 1924.
D. Picasso painted Three Musicians, a splendid
example of synthetic cubism, in the summer of
1 924.

What does the appositive phrase a splendid example of


syn thetic cubism modify? It answers the question What
is The Three Musicians? rather than Who was Picasso ?
Since it modifies the second noun, not the first, choice D
is correct.

221
1 78 McGraw-Hill Education: SAT

Exercise 6: Dangling and Misplaced Modifiers

Rewrite each u nderlined portion, if necessary, to correct a ny dangling or misplaced


modifiers.

1. Rounding the bend, the pub of my dreams finally 10. Without being aware of it, termites can infest your
came i nto view. home unless you take proper precautions.

11. Always regarded as a dutiful mother. we were


2. Although emotionally drained, Martha's creative
surprised to hear Carol complaining about domes­
instinct compelled her to keep writing.
tic life.

3. Determined to avenge his friend, the sword was 12. To get a good jump out of the starting blocks, sprinters
unsheathed by Claudius. say that proper h ip positioni ng i s essential.

13. Seeking ways t o reduce t h e budget deficit, propos­


4. To find a good Thai restaurant. there are a lot of
als for cutbacks are being considered by the town
apps and websites to help you .
council.

5. Even with a sprained a nkle. the coach forced Adam 14. Although unhappy with the tone of the debate, the
b ack i nto the game. senator's pla n w a s t o remain c a l m a n d rational.

15. Famous for i t s visual arts scene, Portland's musical


6. We found my lost earrings walking b ack to my car.
culture is also a source of local pride.

7. Lacking any real sailing skills, David's primary 16. Without seeming to move a muscle, the coin dis­
concern was keeping the boat upright. appeared instantly from the magician's hand.

17. To maintain good health. physicians recommend


8. Already exhausted from the day's climb, the looming
both vigorous exercise and disciplined eating.
storm forced the h i kers to pitch a n early camp.

18. After searching for months for the perfect r u g , one


9. Thinking that her friends were behind her, it fright­ appeared as we were exploring a garage sale.
ened Allison to realize that she was alone.

222
CHAPTER 4 / THE SAT WRITING AND LANGUAGE TEST: THE TEN ESSENTIAL RULES 1 79

Lesson 1 4: Don 't confuse adjectives a n d adverbs

Which is correct? A and B, and the phrases how poised and how cogent
have the same fo rm. However, the adjective in the sec­
A. I was impressed by how poised he was and how
ond phrase is misused: we cannot say his argument was
cogent his argument was presented.
presented cogent, but rather his argument was presented
B . I was impressed by h o w poised he was a n d how
cogently. Action verbs like presented can only be modi­
cogent his argument was.
fied by adverbs, not adjectives. Sentence B corrects the
C. I was impressed by how poised he was and how
modifier error but uses stilted p hrasing. Sentence C, the
cogently he presented his argument.
best of the three, although less strictly parallel than sen­
At first, reading, sentence A seems to follow the law of tence B, corrects the modifier error in A and the stiffness
parallelism : it fo llows the formula I was impressed by of sentence B.

Don't u s e a n adjective t o d o the j o b o f a n adverb. Many popular advertisements grab your attention by replac­
ing adverbs with adjectives, as in Think different, Eatfresh, Shine bright, and Live strong. But in Standard English,
adjectives are strictly noun modifiers. If you want to modify a verb (or an adjective or another adverb), only an
adverb will do. Most adverbs end i n -ly (as in profoundly, quickly, and desperately), but many common ones do not.

Common adverbs that do NOT end in -ly:


always, away, ever, never, there, here, so, too, yet, very

Common adjectives that D O end in - ly:


lovely, lonely, motherly, neighborly. friendly, costly, beastly, lively, womanly, likely, scholarly

Common words that can serve EITHER as adjectives or adverbs:

Adjective Adverb
very I drove that very car. It is very hot.
well The cat is not well. She performed well.
fast She is a fast reader. Don't go so fast.
straight It was a straight shot. I can 't shoot straight.
just It was a just decision. She just arrived.
late We had a late lunch. It happened late in the day.
low You have set a low bar. Don 't sink so low.
h igh I have high standards. I can 'tjump very high.
hard That test was hard. Don 't push so hard.

If you have trouble deciding b etween using an adjective and using an adverb, ask: "What question does this
word a nswer?" I f it is a question about a noun or pronoun, the modifier must b e an adj ective. I f it is a question
about a verb, adjective, or another adverb, the modifier must b e an adverb.

223
1 80 McGraw-Hill Education: SAT

Lesson 1 5: Know wh n to use -er ·est more and most

Which is correct? So, i n the sample sentences, choices D and F are


correct.
A. I don 't know which is most troubling: your
Which is correct?
apathy or your incompetence.
B . I don 't know which is more troubling: your G. Please hold the baby gen tler next time.
apathy or your incompetence. H . Please hold the baby more gently next time.
Sentence A is comparing only two things: apathy and Here, the problem with sentence G is the problem we dis­
incompetence, so it must use the comparative form, cussed i n Lesson 14: an adjective is being used where an
more, instead of most. Sentence B is correct. adverb is required. Since the modifier i s answering the
question "How should one hold the baby?" it is answer­
I f a sentence compares two t hings at a time (we ing a question about the verb hold, and therefore should
call this a binary comparison), it must use a take the adverbial form more gently.
comparative adjective, that is, one that use -er or Which is correct?
more. I f the sentence singles out one thing from a J. A nn ie is the most unique person I know.
group of three or more, it must use a superlative K. A nnie is unique.
adjective, that is, one that uses - est or most.
The adj ective unique is known as an "absolute" or "super­
lative" adj ective . It comes from the Latin uni, meaning
"one," and it means "one of a kind." Therefore, tacking
Which is correct?
on most is redundant. Sentence K makes the same point
C. Your dog couldn 't be adorabler. without the redundancy.
D. You r dog couldn 't be more adorable.
Which is correct?
E . Incorporating the company was more simple Don't modify absolutes like perfect, unique, sin­
than I expected. gular, or obliterated unless you are trying to b e
F. Incorporating the company was simpler than ironic.
I expected.
When do we use -er, and when do we use more? The rule
is actually pretty straightforward.

I f an adj ective has j ust one or two syllables, it


usually takes the -er suffix (e.g., faster, stronger,
sillier), but if it has more than two syllables, it
usually takes more (e.g., more beautiful, more
outrageous, more desperate).
However, monosyllabic past p articiples, when
used as adj ectives, also tend to take more rather
than - er: we s ay more set in h is ways rather than
setter in his ways, more shocked rather than
shockeder, and more tired rather than tireder.
Fun is another interesting exception. Although
something that is comparatively funny is funnier,
something that is comparatively fun is more fun.
For some reason, Standard English has decided
against funner.

224
CHAPTER 4 I THE SAT WRITING AND LANGUAGE TEST: THE TEN ESSENTIAL RULES 1 81

Exercise 7: Using Modifiers Correctly

Correct a ny modifier problems in the sentences below.

1. I n the second debate, the councilwoman made her 7. As you revise your essay, try to express your
points much stronger than she did i n the first one. thoughts clearer and develop your ideas more.

2. My shirt smelled foully after rugby practice. 8. The chemistry final was much more easy than the
last two chapter tests.

3. We never usually get to go on such exotic vacations.


9. Caroline's sculpture was the most u n ique among
the entries.

4. My father is the most patient of my parents, but


my mother is more knowledgeable about relation­
ships. 10. These cost-cutting measures won't barely address
the budget deficit.

5. The sixth graders weren't hardly i nterested in


going to the museum after school. 11. The teacher never told u s about the test until the
day before.

6. I can run a marathon easier than I can swim three


miles. 12. Students never usually verify the "facts" they use
in their research papers.

225
1 82 McGraw-Hill Education: SAT

Ru le 6: Ma ke You r Com pa risons C lea r


and Prec ise
Le son 1 6: Make sur you r compa risons a re logica l

Which is correct? turnout is being compared to last year. Thi s is another


type of illogical comparison called a category error:
A. Not only is A nna the captain, but she also prac­
the two things being compared are not comparable
tices harder than anyone on the track team .
things. Sentence D corrects this error because the turn­
B . Not only i s Anna the captain, b u t she also prac-
out for last year's festival is i n the same category as the
tices harder than anyone else on the track team.
turnout for this year's festival. Since this is an "apples-to­
Anna c an not work h a rder than she herself does, and apples" comparison, sentence D is correct.
she i s on the track team, so the first comparison i s
illogical. It i s logical, h owever, to s a y t h a t she works
h a rder than anyone else on the track team, so sentence M a ke sure all of your comparisons are logical
B i s correct. comparisons.
Make sure that equivalent things are not
Which is correct? treated as non-equivalent things. (For instance,
Anna can't practice harder than herself.)
C. The turnout for this year's art festival was even
• M a ke sure that non-comparable t h ings are
better than last year.
not treated as comparable things (For instance,
D. The turnout for this year's art festival was even
this year's turnout c a n't be c ompared to last
better than the turnout for last year 's festival.
year, but it c a n b e compared to last year's
The phrase even better i ndicates a comparison, but turnout.)
between what two things? In sentence C, this year's

226
CHAPTER 4 I THE SAT WRITING AND LANGUAGE TEST: THE TEN ESSENTIAL RULES 1 83

Lesson1 7: Know how to use less/fewer, many/much, or

amount/number

Which is best?
A. To decrease the amount of violent conflicts
But what if the quantities are countable and con­
among rival fans, the concession stands will sell
tinuous, like miles, gallons, or miles per gallon?
less alcoholic drinks during the game.
For instance, would you say This car gets fewer
miles per gallon or This car gets less miles per gal­
B. To decrease the number of violent conflicts lon? The answer depends on whether the context
among rival fans, the concession stands will sell suggests you should emphasize the quantity's
fewer alcoholic drinks during the game. countability (in which case you should use fewer)
C . To decrease the amount of violence among rival
fans, the concession stands will sell less alcohol
or its continuity (in which case you should use
less). Of course, you could avoid the problem
during the game. altogether by saying This car is less efficient.
Sentence A is problematic because it uses amount and less
The terms less, much, and amount apply gener­ in reference to countable and discrete quantities, conflicts
ally to uncountable or continuous quantities B
and alcoholic drinks. Sentence corrects the problem by
like traffic, money, and food. The terms fewer, switching to number and fewer, but sentence C, which
many, and number apply generally to count­ changes the quantities themselves to violence and alco­
able and discrete quantities like cars, dollars, hol, sounds more natural. The SAT will not expect you
and pizzas. B
to choose b etween choices and C on a multiple- choice
question, because technically both are correct.

227
1 84 McGraw-Hill Education: SAT

Exercise 8: Making Logical Comparisons

Correct a ny i llogical compa risons i n the sentences below.

1. The show was universally praised by critics, who 6. Modernist poetry was far less accessible to the
said consistently that it was more intelligent and readers of its time than was Shakespeare.
provocative than anything on the air.

7. Her suitcase would not close because she had


2. Team unity a n d a strong work ethic were t h e key to packed too much of her towels into it.
their success.

8. The year- end bonus w a s equally divided between


3. Mathematics lessons i n Japanese classrooms, Parker, Alyssa, and me.
unlike American classrooms, are often focused
on solving a single complex problem rather than
many simplistic problems.

9. M a ny students wanted to b e a lifeguard at the club.

4. The hybrid electric- combustion engines of the new


cars are much quieter than conventional cars.

10. The toughest thing about her class i s you h ave to do


so much homework every night.

5. To t h e cnt1cs of t h e time, t h e surrealists were


regarded as being as inscrutable, if not more so,
than the Dadaists.

228
C HAPTER 4 / THE SAT WRITING AND LANGUAGE TEST: THE TEN ESSENTIAL RULES 1 85

Ru le 7: Ma ke Sure Your Pronou ns Are C lea r


a nd Prec ise
Lesson 1 8 : Ma ke sure your pronouns agree
with their a ntecedents

Which is correct? Which is correct?


A. Our .financial team strictly maintains the confi­ F. The .filibuster is a strategy where senators can
dentiality of their clients. extend debate in order to prevent a vote.
B. O u r.financial team strictly maintains the confi­ G. The .filibuster is a strategy in which senators can
dentiality of its clients. extend debate in order to prevent a vote.
C. Our .financial counselors strictly maintain the
confidentiality of their clients. Interrogative pronouns are the pronouns we
use to ask questions, like who, what, where, and
when. When these pronouns are not used to ask
Every definite pronoun like it, him, herself, questions, they serve as definite pronouns that
and their takes the place of a noun or pronoun refer to the nouns that immediately precede
called the antecedent. Every definite pronoun them (that is, they serve as appositives) . Like all
must agree in number (singular or plural) and definite pronouns, they must agree in category
c ategory. with their antecedents.

Interrogative Antecedent
I n sentence A, the definite pronoun, their, is plural, but Pronoun Category
the antecedent, team, is singular. This is a number dis­ where place
agreement. (At least it is in Standard American English;
who person
i n Standard British English, collective nouns like team,
crowd, and committee are treated as plurals.) Sentence when time
B corrects this problem but introduces a subtle number how explanation
shift and i mplies (probably incorrectly) that the entire why reason
t e a m s h a r e s its c l i e n t s . Sentence C corrects both prob­
what thing or concept
lems and so is the best choice.

Which is correct?
The pronouns in sentence D and sentence F both dis­
D. Sabrina, surprisingly, was the one that broke the agree in category with their antecedents: Sabrina is a
silence. person, not a thing, so who is a more appropriate pro­
E . Sabrina, surprisingly, was the one who broke the noun than that. The filibuster is a procedure, not a place,
silence. so which is a more appropriate pronoun than where.

229
1 86 McGraw-Hill Education: SAT

Lesson 1 9: Avoid ambiguous pronouns


What is wrong with t h e fol lowing sentences? In sentence B, which way of thinking gives evidence
priority, scientific thinking or ideological thinking? Both
A. The coach told Mike that he was going to miss
sentences should be revised to eliminate ambiguous
the next game.
pronouns.
B. The main difference between scientific thinking
and ideological thinking is that it gives evidence C. The coach said that h e would bench Mike for the
priority over belief next game.
D. Scientific thinking, unlike ideological thinking,
Both of these sentences are ambiguous. In sen­
gives evidence priority over belief
tence A, who will miss the game, Mike or the coach?

230
CHAPTER 4 / THE SAT WRITING AND LANGUAGE TEST: THE TEN ESSENTIAL RULES 1 87

Lesson 20: Ma inta i n consistency with you r pronouns

Which i s correct? D. The flying squirrel uses its patagium-a mem­


brane extending from the wrist to the ankle-as
A. My wife and I enjoy attending our school
a parachute to help it glide safely out of the reach
reunions because you meet so many interesting
of predators.
people there.
B. My wife and I enjoy attending our school Sentence C commits a pronoun shift . The pronoun refer­
reunions because we meet so many interesting ring to the flying squirrel has shifted from its to them.
people there. Sentence D makes the correction.
The pronoun references i n sentence A are inconsistent:
the generic pronoun you conflicts with the personal
Watch your pronouns to make sure that they
explanation indicated by the context, so the use of we in
don't shift . Once you choose a pronoun to refer to
B
sentence is more appropriate.
a particular antecedent, stick with it.
Which is correct?
C. The flying squirrel uses its patagium-a mem­
brane extending from the wrist to the ankle-as
a parachute to help them glide safely out of the
reach of predators.

231
1 88 McGraw-Hill Education: SAT

Exercise 9: Using Pronouns

Circle all pronouns and rewrite to correct a ny pronoun errors.

1. Th i s is one o f those times in a game where a n 9. Neither Jack nor Ted thought that their team could
undisciplined player c a n lose focus o r forget about lose the game, even when he began m issing his
strategy. shots.

2. I f a student wants t o learn the meaning o f a word, 10. Students sometimes a ren't ready to handle the
begin by learning its relevant context. extra work that is required when his or her courses
become more demanding.

3. Caroline passed the phone to Julia, but she couldn't


bring herself to speak. 11. I enjoy reading stories where underdogs eventu­
ally triumph.

4. N o t wanting to be t h e o n e t h a t slowed t h e team


down, David dropped out of the race. 12. Everyone w i l l be expected t o d o their share t o
prepare t h e camp for visitor's day.

5. Brown is committed to assisting their students


by providing him or her with any necessary finan­ 13. The museum received so many donations that t hey
cial aid. surpassed their fund-raising goal for the year.

6. The media ignored the reports because it didn't 14. The judges usually give the trophy to the performer
consider them newsworthy. that makes the fewest mistakes.

7. No one that has been through the first week of boot 15. We have configured the pool so that each swimmer
camp ever believes that they will make it through will have a lane to themselves.
the entire six weeks.

16. Who was the player that hit the home run?
8. Although one should never read carelessly, you
should move briskly through the page to maintain
focus on the purpose behind the text.

232
CHAPTER 4 I THE SAT WRITING AND LANGUAGE TEST: THE TEN ESSENTIAL RULES 1 89

Lesson 2 1 : Use the correct pronoun case

Each of these sentences contains one pronoun error. These pronoun errors are called errors i n case. Here
Can you find it? are the corrections:
A. As the waiter was talking to Jenna and I, he E. As the waiter was talking to Jenna and me, he
showed us the tattoo on his neck. showed us the tattoo on his neck.
B. I don 't know anyone who can run a campaign F. I don't know anyone who can run a campaign
more effectively than her. more effectively than she can.
C. A lthough Carl said h e wasn 't hungry, the first G. Although Carl said he wasn 't hungry, he was the
one at the buffet was him. first one at the buffet.
D. The team voted and selected myself as the next H. Th e team voted and selected m e a s the next
captain. captain.

The case of a pronoun refers to its relationship to the verb. If a pronoun serves as or is e quated with the sub­
ject of a verb, it takes the subjective case. If it serves as the direct or indirect object of the verb, it takes that
objective case. I f the object of the verb has the same referent as the subject, then it takes the reflexive case. I f
it indicates possession, i t takes the possessive case.

Subjective case Objective case Reflexive case Possessive case


I, he, she, we, me, him, her, us, myself, himself, herself my/mine, his, her/hers,
they, who them, whom ourselves, themselves our/ours, their/theirs

In sentence A, the pronoun I is the object of the prep ­ I n sentence C, the verb was i s a linking verb, which
ositional phrase to Jenna and I, and so it requires the means that the pronoun him is being "equated" with the
objective case, as i n sentence E . In sentence B, the com­ subject one, and therefore should b e changed to the sub­
parative phrase more effectively i s adverbial, indicating j ective case, as i n sentence G. (Notice, also, that sentence
that the comparison i s between verbs i n the clauses who G "inverts" the main clause from sentence C so that it
can run and she [can run}, so the pronoun her should is parallel with the first clause.) Sentence D abuses the
b e changed to the subjective case, as i n sentence F. reflexive case, which is the subject of our next lesson.

233
1 90 McGraw-Hill Education: SAT

Lesson 22: Don 't a buse reflexive pronouns

Which i s correct?
A. Either Caroline or myself will open the account
this week.
B. Either Caroline or I will open the account this
week.

A reflective pronoun should only be used as


the object of a verb when it is identical to the subject:
e.g. , I did it all by myself. She cut herself.
• an emphatic appositive (Lesson 13): I myself would never do such a thing.
Do NOT use reflexive pronouns as ordinary subjects or objects.

Since myself is part of the subject phrase, it must


B
take the subjective case; therefore sentence is correct.

234
CHAPTER 4 / THE SAT WRITING AND LANGUAGE TEST: THE TEN ESSENTIAL RULES 191

Exercise 1 0: Pronoun Case

Circle the correct pronoun i n each sentence.

1. The climb was much easier for Camille than i t was 10. It seems quite clear that you and (I/me) will have to
for Jeff and (I/me/myself). work together to solve this problem.

2. The other contestants did not seem as confident as 11. I t might b e difficult for (him and me/he and I ) to
(he/him/himself). agree on a topic.

3. (Us/We) detectives are always careful to follow 12. (We/Us) a n d t h e other n e w members debated the
every lead. issue for over two hours.

4. Every student should make (his or her/their) own 13. The owners of the club offered my wife and
study plan. (I/me/myself) a free bottle of wine with dinner.

5. The administrators never seem to listen to the 14. No other member of the team could outrun
opinions of (us/we) students. (I/me/myself) .

6. Jim gave control of the project to Fiona and 15. The teachers were getting tired of (him/his)
(me/myself/I). constantly falling asleep i n class.

7. The university presented the honor to David and 16. M ajor League ballparks have always held a special
(he/him/himself) . attraction for Dave and (I/me) .

8. Justine and (me/I/myself) have always been 17. I am concerned about (you/your) taking so much
closest friends. time off work.

9. There is no point i n (our/us) delaying the tests any


longer.

235
1 92 McGraw-Hill Education: SAT

Ru le 8: Ma ke You r Verbs C lea r a n d Prec ise


Lesson 23: Know how to use the consequentia l or
" erfect" as ect

Which is correct? Tense Aspect


I have been to Present Consequential
A. It doesn 't really matter now, because I have been
the mountaintop. (or "perfect")
to the mountaintop.
B. I t doesn 't really matter now, because I was o n I went to the Past Simple
the mountaintop. mountaintop. (isolated action)
C. It doesn 't really matter now, because I went to Sentence A is obviously about who King is now as a
the mountaintop. consequence of a previous event, not simply about what
he did in the past. I n other words, it requires the pres­
Why do sentence s B
and C sound so u n inspmng
ent tense and the consequential (or "perfect") aspect.
i n comparison to sentence A (adapted from Martin
Luther King Jr.'s last speech)? They fal l flat because they
B
Sentences and C destroy this essential meaning by
putting the verb in the simple past tense.
destroy the meaning conveyed by the tense and aspect
of the verb in sentence A. The tense of a verb indicates
its place i n time: past, present, or future, but the aspect
of a verb indicates how its action or status extends to
the subject.

The aspect of a verb indicates how its action or status extends to the subject, and is general ly independent of
tense. For instance, a present tense verb can have many different aspects:

I eat. I am in the habit of eating. (Habitual aspect)


I am eating. I am in the process of eating. (Progressive aspect)
I have to eat. I feel compelled to eat. (Compulsive aspect)
I have eaten. My current status is the consequence of previous eating. ( Conse­
quential or "perfect" aspect)
I have been eating. My current status is the consequence of previous eating,
and I am still eating. (Consequential and p rogressive aspects)

Grammatical forms of the consequential (or "perfect") aspect:

Present perfect has/have + past participle e.g., I have eaten.


Past perfect had + past participle e.g., They had never smoked.
Future perfect will have + past participle e.g., By Friday, we will have completed the project.

Use the consequential (or "perfect") aspect (e.g., have taken, had taken, will have taken) when you want to
indicate that a status is the consequence of a previous action or sta tus.

I have eaten. My current status is the consequence of previous eating.


They had never smoked. Their status at that point in the past was the consequence
of previous non-smoking.
By Friday, we will have completed the project. Our status next Friday will be the consequence of the fact
that we completed the project.

236
CHAPTER 4 I THE SAT WRITING AND LANGUAGE TEST: THE TEN ESSENTIAL RULES 1 93

Lesson 24: Know how to express h istorical facts


and genera l ideas

Which is correct? For the first pair of sentences, context is everyth i ng.
I f the sentence were part of a p aragraph discussing
A. In his book Walden, Thoreau provided a mani­
Thoreau's life or the h istory of Transcendentalism, it
festo for self- reliance.
would be a statement of historical fact, and so choice A
B. In his book Walden, Thoreau provides a mani­
would be preferred. I f, however, this sentence were part
festo for self reliance.
of a discussion of the ideas i n Walden, then sentence B
Which is correct? would be correct.
The second sentence includes two clauses. The first
C. The ancient Greek philosopher Zeno taught that
refers to the historical fact that Zeno was a teacher, and
motion was an illusion.
the second refers to a general idea about motion. I f you
D. The ancient Greek philosopher Zeno taught that
wish to emphasize the "idea-ness" of the second clause,
motion is an illusion.
then sentence D is the best choice. I f you wish to empha­
E. The ancient Greek philosopher Zeno teaches
size the fact that this claim is " history" (that is, no longer
that motion was an illusion.
believed ) , then sentence C is the best choice.
F. The ancient Greek philosopher Zeno teaches
that motion is an illusion.

Because both Zeno and Thoreau are long dead, the first
version of each sentence, with past tense verbs, may
seem correct. However, it is important to ask: do these
sentences indicate historical facts or general ideas?

In Standard English, historical facts take the past


tense, but statements about general ideas and ref­
erences to the content of widely available artistic
works usually take the present tense. In an ambig­
uous case, such as when referring to an idea that
has been refute d over the course of history, choose
the t e n s e t h a t emphasizes the appropriate quality:
use the present tense if you intend to emphasize its
"idea-ness," but use the past tense if you intend to
emphasize the fact that it is "history."

237
1 94 McGraw-Hill Education: SAT

Lesson 2 5 : Watch for irregular verbs

Which is correct? participle torn, not tore. The verb to tear i s a n irregular
verb, which means that its past participle is not an - ed
A. Peter was in pain after the run because he had
form of the verb. The correct sentence i s B .
tore his Achilles tendon.
Here is a list of some common irregular verbs.
B . Peter was i n pain after the r u n because he had
Remember that verbs i n the consequential or "perfect"
torn his Achilles tendon.
aspect require the past participle form, not the past
The verb i n the second clause takes the consequential tense form. For instance, I have drank is the wrong form;
(or "perfect") aspect (Lesson 24), which requires the past I have drunk i s correct.

Infinitive form Past Tense Past Infinitive form Past Tense Past
Participle Participle
to arise arose arisen to hurt hurt h u rt
to awaken awoke awoken to kneel kneeled, knelt knelt
to beat beat beaten to know knew known
to begin began begun to lay (to put in place) laid laid
to blow blew blown to lie (to recline) lay lain
to break broke broken to ride rode ridden
to burst burst burst to run ran run
to cast cast cast to shrink shrank shrunk,
to come came come shrunken
to creep crept crept to sink sank sunk
to do did done to speak spoke spoken
to dra w drew drawn to spring sprang sprung
to drink drank drunk to swim swam swum
to drive drove driven to take took taken
to forsake forsook forsaken to tear tore torn
to get got got, gotten to write wrote written
to go went gone

238
CHAPTER 4 I THE SAT WRITING AND LANGUAGE TEST: THE TEN ESSENTIAL RULES 1 95

Exercise 1 1 : Verb Tenses and Aspects

Circle the verb form(s) that make each sentence coherent.

1. This morning, Ryan (came/has come/comes) to Truman Capote, to embody youthful innocence
work with bags under h i s eyes because he (stayed/ and imagination.
had stayed/was staying) up all last night.
12. That evening, w e (had/had had) a lovely m e a l with
2. Already, and without (spending/having spent) s o the group with whom we (hiked/had hiked) all
much as an hour on research, Dale (wrote/has afternoon.
written/will write) the first draft of her essay.
13. (Walking/Having walked) all n ight, t h i s morning
3. (Developing/Having developed) t h e first hydrogen we (were/had been) desperate to find a resting
cell automobile, the team (hoped/had hoped) to spot.
revea l it to the world at the technology exposition.
14. By the time I am done with finals, I (will write/will
4. Right after school, we (went/had gone) to Mario's have written) four major papers.
for pizza.
15. (Winning/Having won) her previous three races,
5. Surprisingly, Catcher in the Rye (is/was/would Anna (was/had been) confident that she (will win/
be) the only full-length novel that the late J. D. would win) the next one as well.
Salinger ever (has published/published/will have
published) . 16. It surprised us to learn that Venus (is/was/had
been) almost the same size as Earth.
6. (Finding/Having found) no evidence against the
accused, the detectives (had/had had) to release 17. Buyers often (worry/have worried/will worry)
him. too much about finding a low mortgage rate, and
(forget/have forgotten/will forget) to scrutinize
7. (Being/Having b een) captured by the rebels, the terms of the contract.
David soon (began/had begun) to fear he would
never escape. 18. I am qualified for this job because I (completed/
have completed/had completed) two courses in
8. When I (arrived/had arrived) home from the digital marketing.
museum, I (started/had started/will start) to plan
my art project. 19. During the time of the ancient Greeks, many
physicians (believed/had believed) that illnesses
9. By the time the committee (adjourned/had (are caused/were caused) by imbalances i n bodily
adj ourned), it (voted/had voted) on all four key fluids.
proposals.
20. Students (often worry/will often worry) excessively
10. As the seventh inning stretch began, we (did not about grades and not enough about understanding.
score/had not scored) a single run.

11. I n To Kill a Mockingbird, Harper Lee (uses/used/


has used) the character of Dill Harris, whom she
(bases/based/has based) on her real-life friend

239
1 96 McGraw-Hill Education: SAT

Ru le 9: Ma ke the Rest of You r Sentence C l ea r


and Prec ise
Lesson 26: Avoid red u nda n cy

Which is correct?
The Law of Parsimony
A. With only seconds remaining left to go in the
game, Michael grabbed the ball and sped quickly All else being equal, shorter is better.
down the court.
B. With only seconds t o go i n the game, Michael Only one of remaining, left, or to go is necessary,
grabbed the ball and sped down the court. because they all h ave the same meaning. Also, since sped
Notice that sentence A does not convey any idea that is means moved quickly, the adverb quickly is redundant.
not also conveyed i n sentence B.
Therefore, the three
words that have been removed are redundant . Sentence
Bi s better because it obeys the Law of Parsimony.

240
CHAPTER 4 I THE SAT WRITING AND LANGUAGE TEST: THE TEN ESSENTIAL RULES 1 97

Lesson 27: Avoid diction errors

Which sentence is best? Sentence C uses propagated, which means spread or


promoted, as an idea or theory. Since news spreads very
A. The news about the court 's ruling extended
much as an idea or theory does, the verb is being used
quickly throughout the Internet.
appropriately.
B. The news about the court 's ruling scattered
Sentence D uses expanded, which, like extended,
quickly throughout the Internet.
typically refers to something growing beyond its typi­
C. The news about the court 's ruling propagated
cal size or limit. Since news doesn't have a typical size or
quickly throughout the Internet.
limit, expanded is not quite the right word.
D. The news about the court 's ruling expanded
quickly throughout the Internet.
Which sentence is correct?
None of these sentences is grammatically wrong, but
E. We interviewed about thirty perspective candi­
sentence A sounds odd. The word extended is not quite
dates for the job.
right for this context. From the Latin tendere which
F. We interviewed about th irty prospective candi­
means "to stretch," extend applies to things, like base­
da tes for the job.
ball games or necks, that are made to go beyond their
typical lengths. But news, unlike a baseball game or a The diction error in sentence E is a "sound-alike" error.
neck, does not have a "typical length," so trying to apply The word perspective is a noun meaning "point of view,"
the verb extend to it is a diction error: the inappropriate but the sentence clearly calls for a n adjective describ­
use of a word. ing the candidates. Prospective i s a n adjective meaning
Sentence B sounds a bit better, but scatter applies to "expected to play a particular role or to achieve a par­
a bunch of individual things, like seeds or mice, that are ticular goal in the future," which is certainly appropriate
suddenly moving away from their group. Since this news in describing a job candidate.
is a single fact, not many individual items in a bunch,
scattered doesn't quite work, either.

Common "sound-alikes"

accept (v) = to agree to take <accept an offer>


except (prep) = not including <every day except Sunday>
except (v) = exclude <present company excepted>

adapt (v) = to make suitable for a particular purpose <adapted to a new use>
adopt (v) = to choose as one's own < adopt a child>
adept (adj) = highly skilled <an adept player>

affect (v) = to influence <it affected me deeply>


effect (n) = result or consequence <had a good effect>

allude (v) = to make a subtle or indirect reference (to) <he alluded to their first meeting>
elude (v) = to escape from; to avoid <elude capture>

allusion (n) = a subtle reference <an allusion to Othello>


illusion (n) = misconception or misperception <optical illusion>

ambivalent (adj) = having conflicting feelings (about) < !feel ambivalent about going to the party>
ambiguous (adj) = unclear or having more than one interpretation <an ambiguous signal>

cite (v) = to credit as a source of information <cite an article>; to commend for meritorious action <cited for
bravery>
site (n) = location where a particular activity occurs < the site of the battle>
sight (v) = to see at a specific location <she was sighted in the crowd>

241
198 McGraw-Hill Education: SAT

compliment (n) = a praising personal comment <compliments are always appreciated>


complement (n) = something that completes or makes a whole < Brie is a fine complement to this wine>

council (n) = a committee < the executive council>


counsel (v) = to give advice <he counseled me wisely>

discrete (adj) = distinct <dozens of discrete parts>


discreet (adj) = prudently modest i n revealing information <please be discreet about our meeting>

elicit (v) = to bring out or to call forth < the joke elicited uncomfortable laughter>
illicit (adj) = unlawful < illicit activities>

eminent (adj) = promi nent and distinguished <an eminent historian>


imminent (adj) = about to happen < imminent doom>

flaunt (v) = to show (someth i ng) off < ifyou 've got it, flaunt it>
flout (v) = to show disregard for <flout the rules>

gambit (n) = a careful strategy or an opening move < a bold gambit>


gamut (n) = the complete range < run the gamut>

imply (v) = to suggest or hint at <a handshake implies agreement>


infer (v) = to d raw a conclusion from evidence < we can infer hostile intent>

morale (n) (mor-AL) = shared enthusiasm for and dedication to a goal < the team 's morale was high>
moral (n) (MOR-al) = lesson or principle about good behavior < the story had a nice moral>

phase (n) = stage in a process < third phase of the project>


faze (n) = to disturb (someone's) composure <fazed by the interruption>

precede (v) = to come before < thunder is always preceded by lightning>


proceed (v) (pro- CEED) = to go on, usually after a pause (pro- forward) <proceed with the task>
proceeds (n) (PRO - ceeds) = fu nds received from a venture <proceeds from the raffle>

principal (n) = head of a school <principal Skinner is well liked>; the i n itial i nvestment in an interest-bearing
account < many investments risk a loss of principal>
principle (n) = guiding rule < the principle of the matter>

reticent (adj) = reserved or reluctant to talk freely <she has been reticent in therapy>
reluctant (adj) = disinclined to do something < reluctant to reveal personal information>

242
CHAPTER 4 / THE SAT WRITING AND LANGUAGE TEST: THE TEN ESSENTIAL RULES 1 99

Exercise 1 2: Diction Problems

C hoose the best word i n the sentences below.

1. Even the most trivial news seems t o (affect/effect) 14. The acid solution was so potent that we had to
the stock price immediately. (dilute/delude) it with water before we could use it
safely.
2. Even t h e most aggressive pesticides could not
(delete/remove/eradicate/abolish) the beetles. 15. Annie's project (excelled/overshadowed/outstrip­
ped/exceeded/preceded) all of our expectations.
3. The (moral/morale) of the troops was at an all-time
low during the Christmas season. 16. Originally built for a small tractor, the engine had
to be (correlated/attuned/converted/reoriented/
4. That scarf really (compliments/complements) your improved) for use as a boat motor.
outfit.
17. As someone committed to fairness i n education,
5. Many well-trained oenologists can (separate/ she could not accept the (iniquity/inequity) of the
distinguish/acknowledge/certify) the tastes of admissions policy.
dozens of different grapes.
18. Although most of the manuscripts were signed by
6. The article emphasized the low voter turnout i n their authors, some were written (anonymously/
order to (imply/infer) that t h e senator may not unanimously).
have been elected by a true majority.
19. It was hard for the comic to (elicit/illicit) even the
7. The justices can debate a case for weeks before a slightest laugh from the crowd.
formal ruling is (appointed/specified/chosen/
predetermined/given/designated). 20. We needed to (adapt/adopt/adept) the play to
make it appropriate for younger audiences.
8. It may b e years before we u nderstand how pollu­
tion from the new power plant m ight (affect/effect) 21. Darryl's self- esteem (enlarged/blossomed/multi­
the regional environment. plied/escalated/proliferated) once she found a peer
group that shared her i nterests.
9. The negotiations became very (apprehensive/
tense/neurotic/fretful/anxious) when the topic of 22. She thought she should be (discreet/discrete)
old tribal conflicts was broached. about their relationship.

10. Heather was the (principal/principle) author of 23. The (council/counsel) will decide how to finance
the study that was recently published i n a promi­ the new city park.
nent scientific magazine.
24. Rather than obeying the coach, Richard always
11. Although enormously popular among filmgoers, tries to (flaunt/flout) the team rules.
the movie was soundly (disparaged/confronted/
molested/eradicated/charged/impaired) by critics. 25. His knowledge of sports runs the ( gamut/gambit)
from table tennis to arena football.
12. Th e words a n d images i n advertisements are care­
fully chosen to subtly (propel/compel/extort/ 26. The jury should not (in fer/imply) guilt from the
oppress/oblige) consumers into buying things defendant's refusal to answer these questions.
they may not want.
27. The builders had to (truncate/curtail/lower/
13. Try as they m ight, t h e hikers could n o t find the belittle/subside) their work during the evening
(antidote/anecdote) to the snake venom. hours after the neighbors filed a complaint.

243
200 McGraw-Hill Education: SAT

28. Rather than eliminate the department all at once, the conflict by promoting nonconfrontational dia­
they decided to (faze/phase) it out gradually. logue of all sorts.

29. Barking dogs can often signal (imminent/eminent) 32. I always felt (reticent/reluctant) to talk i n class.
danger.
33. The democratically elected government has
30. After our vacation, we decided to (proceed/ been forcefully (shifted/substituted/exchanged/
precede) with the plan. supplanted) by a m i l itary cabal.

31. Recent diplomatic efforts have focused on (defus­ 34. The police officer was (cited/sighted) for her efforts
ing/declining/dwindling/degrading/discounting) i n the hostage rescue.

Eliminate a ny redundant words or ph rases in the paragraph below.

35. When we look back to past history, we see that


whenever a new i nnovation is introduced for the
first time, people rarely accept the whole entire
concept, at least not right away. I f and when some­
thing threatens the ways of the past, people don't
easily accept this new concept. Societies neces­
sarily need stability because consistency and
predictability make people feel comfortable and
minimize conflict. Even when technology gives
us a more efficient method, we often continue on
with our older, less efficient ways. For instance,
it's not uncommon to see people using e-mail for
quick communications while at the same time
t hey could have just texted to accomplish the same
thing. I f we take a moment to pause and consider
for a second, it doesn't take much to see we can see
that we can communicate more efficiently by text.
And t here are even some traditionalists who like
the old way of doing things and will write letters on
paper, which requires killing trees!

244
CHAPTER 4 / THE SAT WRITING AND LANGUAGE TEST: THE TEN ESSENTIAL RULES 201

Lesson 2 8 : Avoid e r rors in id iom

What is the difference between these two sentences? D. Effective therapy depends both on consistent
adherence to the protocol and regular recalibra­
A. Ifyou want to make friends, you should go on in
tion of the medication dosage.
the party.
E. Effective therapy depends on both consistent
B. Ifyou want t o make friends, you should go i n on
adherence to the protocol and regular recalibra­
the party.
tion of the medication dosage.
These sentences use different semantic idioms, and so give
very different advice. When you tell someone to go on in,
Sentence C uses the word both, which can either be fol­
you are giving him or her casual permission to enter, so
lowed by a simple plural noun (both legs, both kinds)
sentence A says that casually inserting yourself into a social
or a prepositional phrase (both of them) or be part of a
situation can make you more likeable. When you ask some­
standard parallel construction, both X and Y, which
one to go in on something, you are asking him or her to con­
we saw i n Lesson 10. A standard parallel construction
B
tribute money to the effort, so sentence says that the folks
is a syntactical idiom, that is, a rigid way of phrasing
throwing the party would like you more if you kicked in a
relationships between ideas. Notice that the phrasing in
few bucks. A semantic idiom is a common phrase with an
sentence C- both X as well as Y-is nonidiomatic. The
established meaning, like push through, onfire, see the light,
phrasing i n D is idiomatic but nonparallel (Lesson 9):
or go in on, that differs from its literal meaning.
X is a prepositional phrase but Y is a noun phrase.
Sentence E is both idiomatic and parallel, and is the best
Errors in idiom are usually "wrong preposition" choice.
errors. In some idiomatic phrases, the choice
of preposition i s essential to the meaning: for
instance, breaking up, breaking down, break­
When writing formally, remember to ESP: elimi­
ing in, and breaking ou t are all very different
nate superfluous prepositions. We often use
activities. In other idiomatic phrases, such as the
"extra" prepositions in informal speech, such
standard parallel constructions described in
as the redundant prepositions in climb up, fall
Lesson 10, the preposition is simply a matter of
down, and fight against. Notice how eliminating
convention. For instance, the sentence Thai food
the unnecessary prepositions in these sentences
is very different than Can tonese food contains an
makes them sound more "prop er " :
error in syntactical idiom. The preposition than
should only b e used with comparative adjectives, Her superior skill and strength helped her to
as in smaller than, fas ter than, and harder than. dominate mter her opponents.
But diffe rent is not a comparative adj ective and
instead takes the prepositionfrom. We should s ay Many of our neighbors helped tmt with the
Thai food is very differentfrom Can tonese food. renovation of the old firehouse.

You don 't want to miss mtt-tm all the Jun.


Which is correct?
C. Effective therapy depends both on consistent Their attempt to extract tmt the harmful chemi­
adherence to the protocol as well as regular cals was unsuccessful.
recalibration of the medication dosage.

245
202 McGraw-Hill Education: SAT

Exercise 1 3: E rrors in Idiom

Choose the correct preposition, or "none" if none is required.

1. I prefer the soft light o f a n incandescent bulb (to/ 10. The i nterview provided insight (about/into/for/
over/more than/none) the harsh l ight of some fluo­ none) the creative process of great d i rectors.
rescent bulbs.
11. We were very angry (about/with/at/agai nst/none)
2. We all agreed (on/with/about/none) a plan to go him for ignoring our phone calls.
skiing rather than hiking.
12. We a l l agreed (with/on/to/about/none) t h e high
3. The defendant would n o t agree (to/on/with/about) quality of the food.
the plea bargain.
13. Her tests include questions that seem very
4. We found dozens of old photographs h idden different (than/from/of/none) those that we see i n
(in/none) between the pages. t h e homework.

5. Good study habits are necessary (to/for/in/none) 14. When she arrived on campus, she felt truly
academic success. i ndependent (of/from/none) her parents for the
first time.
6. The new house color is not very different
(from/than/to/none) the old one. 15. We were very angry (about/at/with/none) the exor­
bitant price of gasoline at the corner gas station.
7. M argot was angry (with/about/at/none) Brian for
not telling her that he was leaving. 16. It was hard not to agree (to/about/with/none) her
offer of a free evening of babysitting.
8. They were both angry (about/at/with/none) the
boys' behavior. 17. I arrived at the meeting too late to raise my
objection (against/to/of/none) the proposal.
9. A lawyer should review the contract to see that it
complies (with/in/about/to/none) the laws of your 18. I f we don't act soon, we may miss (out on/none) the
state. opportunity to lock i n the lowest rates.

246
CHAPTER 4 I THE SAT WRITING AND LANGUAGE TEST: THE TEN ESSENTIAL RULES 203

Lesson 29: Know how to u se the active and passive voices

Which is better? Overusing the passive voice not only makes your
A. I broke the paddle.
sentences wordier, but also often indicates eva­
siveness, because the passive voice does not
B. The paddle was broken by me.
require the actor. For instance, a statement like I
made a mistake cannot be construed as an evasion
Sentence A and sentence B make the same statement, of responsibility when phrased in the active voice.
but in different voices: sentence A uses the active voice However, the passive voice form A mistake was
and sentence B uses the passive voice. In the active made by me, when "trimmed" (Lesson 3) becomes
voice, the subject is the "actor" of the action, but in the A mistake was made, which is clearly evasive.
passive voice, it is not.
Which is better?
For most declarative statements in which the actor E . Although w e enjoyed the hike to the peak, o n the
is known, the active voice (e.g., I kicked the ball) way down mosquitoes bit us, a thunderstorm
is clearer and more direct than the passive voice drenched us, and countless thorns scratched us.
(e.g., The ball was kicked by me) . F. Although we enjoyed the hike to the peak, on
the way down we were bitten by mosquitoes,
drenched by a thunderstorm, and scratched by
countless thorns.
Which is better? In sentence E, all three clauses at the end of the sen­
C . Henry ate all of his steak, but his vegetables were
tence are parallel and active, yet the sentence sounds
uneaten .
strange. In sentence F introducing the passive voice
D. Henry ate all of his steak but none of his
improves the sentence by creating another level of par­
vegetables.
allelism, because now all four clauses have the same
subject: we enjoyed . . . we were bitten . . . [we were}
drenched . . . [we were} scratched.
In sentence C, the first clause is active, but the sec­
ond is passive. This is not only a violation of the Law of
Parallelism (Lesson 9), but also a subtle evasion: who Sometimes parallel structure requires using the
failed to eat the vegetables? Sentence D is more parallel, passive voice.
clear, and direct.

247
204 McGraw-Hill Education: SAT

Lesson 30: Understa nd you r moods

Which is correct? Counterfactuals can also i nclude indirect com­


mands, wishes, expressions of doubt, hypothetical
A. If I was more patient, I would become a good
consequences, and suggestions, all of which take the
violinist.
subjunctive mood.
B. If I were more patient, I can become a good
violinist.
C. If I were more patient, I could become a good A mood is a verb category that indicates whether
violinist. a clause is a factual statement (indicative mood,
These sentences are conditionals, which take the form
as in I went to the park), a direct command
(imperative mood, as in Go to the park!), a ques­
"IfX, then Y" or simply "IfX, Y" where X is a clause called tion (interrogative mood, as in Did you go to the
the hypothesis and Y is a clause called the conclu­ park?), or a counterfactual (subjunctive mood,
sion. The hypothesis takes different forms depending as in I should have gone to the park).
on whether it is occasional, unlikely, or counterfac­
tual. The hypothesis here is unlikely or wishful and the
conclusion indicates a possibility, so, as our discussion Verbs that are i n the subjunctive mood often require
below will clarify, only sentence C has the correct form. a subjunctive auxiliary, otherwise known as a "verb
I f the hypothesis is occasional or likely, then it takes modal."
the indicative mood; that i s , it i s s t a t e d a s a fac t . For
instance, theorems in mathematics and logic and state­ Subjunctive indicates example
ments about common consequences take this form: auxiliary
Can present ability I can play the piano.
If two sides of a triangle are congruent, then the two
base angles are also congruent. Could present I could be losing my
possibility eyesight.
If I eat too much, I will have a hard time sleeping. Could past ability I remember when I
could run.
Ifyou turn the switch, the light will go on.
Could past permission Last year, we could
I f the hypothesis is present counterfactual, that is, it use the pool.
is unlikely or wishful, then it takes the present subjunc­ May present You may enter.
tive mood. (Notice that a present subjunctive hypoth­ permission
esis, if it does not use the verb to be, can take the same May present That may be true.
form as the simple past tense.) possibility
If I had a million dollars, I would buy a new house. Might l i kelihood I might go fishing
later.
IfKate could tolerate the noise, she would come to the Might purpose I took a nap so I
club with us. might be rested.

If I were taller, I would play in the NBA . Must compulsion I must have that
dress.
I f the hypothesis is past counterfactual, that is, it Should suggestion You should eat more.
contradicts a state or event in the past, then it takes the Should likelihood The train should
past subjunctive mood. (Notice that a counterfactual arrive soon.
hypothesis takes the same form as the past consequen­
Will futu re Your day will come.
tial, and the counterfactual conclusion takes the conse­
inevitability
quential aspect (Lesson 23).)
Would conditional If I had tried harder,
If I had caught the ball, we would have won the game. conclusion l would have won.
Would inclination l would eat tha t.
past i nevitability They said l would
If I had been more studious in college, I could have
Would
graduated cum Laude.
never walk again.

248
CHAPTER 4 I THE SAT WRITING AND LANGUAGE TEST: THE TEN ESSENTIAL RULES 205

The verb to be can sometimes take its subj ective form Which is correct?
without an auxiliary:
A. If we would have left earlier, we would not have
been ca ught the storm.
Subjunctive forms of the verb to be B. If we had left earlier, we would not have been
ca ugh t the storm .
If I were faster, I could play wide (unlikely)
receiver. Again, sentence A is a conditional with a counterfac­
He plays as if he were never (counterfactual) tual hypothesis, i ndicating that a nonfactual condition
injured. would have a particular result. However, the auxiliary
would indicates a conditional conclusion, not a condi­
I wish I were ten pounds lighter. (wishful)
tional hypothesis. The counterfactual hypothesis takes
He asked tha t we be there exactly (indirect the same form as the past consequential (Lesson 23),
at 6. command) had left, as in sentence B.

249
206 McGraw-Hill Education: SAT

Exercise 1 4: Mood and Voice

Circle the correct verb form i n each of the following sentences.

1. If our wide receiver (was/were) a little faster, he 11. If! (were/was/had been) in Paris, I would probably
would get more open in the secondary. be spending most of my time at the Louvre.
2. As a matter of fact, Theo (was/would have been) 12. If I (might have/would have/had) known that the
only six years old when the Civil War (had begun/ food was so good here, I (would have come/would
began). come/came) sooner.
3. Denny would be more successful if only he (pro­ 13. The coach demanded that we (would be/be/should
moted/would promote) himself more aggressively. be/were) in bed by eleven o' c lock.
4. The brochure suggested that we (are/be/would be) 14. Yvonne acted as if she (was/were) the only
at the camp first thing in the morning. customer in the restaurant.
5. I wish that my horse (were/was) not so lethargic 15. Gina wished that she (had/would have/will have)
this morning. chosen the red dress instead of the pink one.
6. If the goalie (would have/had) lifted his glove even 16. The professor spoke to us as if he (was/were) an
slightly, the puck (would have gotten/would get) ancient Athenian general.
through.
17. I (would have wanted/wanted) to (have seen/see)
7. He acted as though the concert hall (was/were) the countryside, but I was sick in bed for the entire
filled with screaming fans. vacation.
8. I wish that summer camp (was/were) two weeks 18. Had I found his wallet, I (would have/had/will
longer. have) returned it to him immediately.
9. If the class (would have/had) voted against it, we 19. If only the doctor (had/would have) told me to cut
would not have purchased the new gerbil cage. back on eating red meat, I (would have/should
have) complied.
10. We doubted that Joanna (will/would/might) get
the part, since she was sick during her audition.

250
CHAPTER 4 I THE SAT WRITING AND LANGUAGE TEST: THE TEN ESSENTIAL RULES 207

Rule 1 0: Know How to Punctuate


Lesson 3 1 : Know how to use apostrophes

Which is correct?
A. Its hard to know when you 're dog has reached C. It 's hard to know when you 're dog has reached
the limit of it 's stamina if your not checking it the limit of it's stamina if you 're not checking it
regularly during your run. regularly during you 're run.
B. It 's hard to know when your dog has reached the
limit of its stamina ifyou 're not checking it regu­
larly during your run.

Apostrophes serve two main functions: to indicate missing letters in a contraction as in can 't (from cannot),
and to indicate possession, as in we went to Jacob 's house.
When turning a singular noun into a possessive adjective, simply add 's, as in the committee 's decision. If the
noun is a plural ending in s, simply add an apostrophe, as in the sisters ' relationship.
Several common contractions are homophones (sound-alikes) of possessives, and so the two are commonly
confused. Fortunately, there is a simple rule to keep them straight: the contraction always gets the apostrophe:
contraction possessive
it's (it is) its
you 're (you are) your
who's (who is) whose
they 're (they are) their

Notice that sentence B above is the only one of the three that uses apostrophes correctly and avoids the its/it 's and your/
confusion.
you 're

251
208 McGraw-Hill Education: SAT

Lesson 3 2 : Know how to use com mas

What is wrong with these sentences? Notice that this treats Massachusetts as an inter­
A. The subject that intimidates me the most, is
rupter (Lesson 3), which is fine because the sentence
calculus.
reads correctly even when it is omitted.
B . I could n o t help Justine with her project, I had
just begun a new job. Substantial modifying phrases in the middle
C. As we passed through Springfield, Massachusetts of a sentence are called interrupting modi­
we stopped at the Basketball Hall of Fame. fiers (Lesson 3) and should be separated from
D. We will be discussing myfavorite poem, "Leaves the main clause by commas. Remember that a
of Grass, " next semester. sentence should read properly even when the
E. I would like to thank my parents, God a n d Ayn interrupters have been removed.
Rand.

Sentence A suffers from the stray comma syndrome. In sentence D, the title of the poem works the same
Simply put, the comma doesn' t belong. Chuck it. way as the state name in sentence C. It is a specifying
modifier and requires commas before and after:
The primary job of the comma is as a separator. It We will be discussing my favorite poem, "Leaves of
is used to separate Grass, " next semester.
• items in a list (e. g . , He was fat, dumb, and
lazy.) When a comma follows a title or phrase in quotes,
• coordinate adjectives (e. g . , She gave a dron­
ing, uninspired speech.)
the comma must precede the end quote.
modifying phrases from the main clause
(e. g . , In summary, I am appalled.) Sentence E omits the serial comma, the comma that
dependent clauses that precede independent separates the second-to-last item in a list from the con­
clauses (e. g . , Whenever I try, Ifail.) junction and. The serial comma is almost universally
• (with a conjunction) independent clauses
from other independent clauses (e.g., I think, accepted as proper and necessary in Standard American
therefore I am.)
English, because without it sentence E becomes absurd.
In this apocryphal dedication of a book, the lack of a
It can also be used to serial comma makes it seem like the author believes
• introduce a quotation (e. g . , Tom said, "I ain't she is the offspring of a deity and a childless woman.
goin ' where I ain 't needed. ")
Of course, the author intends her dedication as a list of
• format an address or date (e. g . , Saturday, four, not two:
July 19, 2014; Cleveland, Ohio) I would like to thank my parents, God, and Ayn Rand.
• to signal an addressee in dialogue or collo­
quial prose (e. g . , Get going, buster!) 1\vo notable authorities that do not accept this
rule are the New York Times and the AP (Associated
Press) Stylebook, which recommend against the Oxford
Sentence B commits a comma splice (Lesson 6). comma except to prevent an ambiguity such as that in
1\vo independent clauses cannot be joined with just a sentence E.
comma. Either change the comma to a colon or semico­
lon, or insert a conjunction: The use of the serial comma (the second comma
I could not help Justine with her project, because I had in the phrase A, B, and C) in Standard American
just begun a new job. Usage is still a matter of debate and therefore will
almost certainly not be tested on the SAT.
Sentence C omits the comma after the state name. It
should read
As we passed through Springfield, Massachusetts, we
stopped at the Basketball Hall of Fame.

252
CHAPTER 4 / THE SAT WRITING AND LANGUAGE TEST: THE TEN ESSENTIAL RULES 209

Lesson 33: Know how to use dashes


What is wrong with this sentence? If the interruption is not much of a departure from the
A. The best that they could do -at least without a main idea, then commas will work also:
splint, was to set the broken bone and wait for The best that they could do, at least without a splint,
help to arrive. was to set the broken bone and waitfor help to arrive.
The dash (or, as it is sometimes known, the em dash) is
used to insert an abrupt break in thought in the middle The punctuation on the two sides of an inter­
or at the end ofa sentence. If the break comes in the mid­ rupter must be identical: either both em dashes
dle, then two dashes signify the beginning and the end or both commas.
of the interruption. In this case, the end of the interrup­
tion is indicated by a comma, where it should be a dash:
The best that they could do-at least without a splint­
was to set the broken bone and waitfor help to arrive.

253
210 McGraw-Hill Education: SAT

Exercise 1 S : Punctuation

Correct any errors in punctuation (apostrophes, commas, dashes, colons, and semicolons)
in the following sentences.

1. Truman Capote's nonfiction book, In Cold Blood is 9. Isabella sprained her ankle, now she won't be able
considered the first, greatest true crime novel. to practice for several weeks.

2. I could not see clearly, until my eyes adjusted to 10. Ifyou can't take care ofyou're own dog don't expect
the bright lights. me to pay for it's grooming.

3. Runners, who step out of they're lanes during the 11. Don' t expect this to be cheap, perfection has it's
first lap, will be disqualified. price.

4. Contrary to popular belief water will reach it's 12. What disappoints me most, is that you didn't even
boiling point more slowly, when its under greater tell me you were leaving.
pressure.

13. I told you, don't go near the street!


5. In my opinion the most interesting part of the trip,
was the river cruise.

14. I remember that, The Monkey 's Paw, was my favor­


ite short story in the ninth grade.
6. Its easy to see, even on the dreariest of days-how
Paris has earned it's reputation as the City of Love.

15. The DVD's that they just received, don't seem to


work in they're player.
7. Having decided to postpone her college education
Jill began looking for a job.

16. A cyclotron, like the one Ernest Lawrence built at


Berkeley-accelerates particles in a spiral path.
8. Regardless of who's phone rings the entire class
will be punished for any disruption.

254
The 18 SAT Grammar Rules You Must Know
1. Subject-Verb Agreement
1. The diner near the dorms which (houses/house) the students (serves/serve)
breakfast all day.
2. The widely recognized red coloring of stop signs everywhere (alerts/alert) people
who can’t even read them to stop.
3. Each team made up of one girl and one boy (has/have) to reenact a scene from
Romeo and Juliet.
4. Her jewelry, in addition to her pokemon cards, (was/were) stolen by the robber.
5. Neither the employees nor the owner (cares/care) about the customer.
6. Beside the bins, where one could smell the stench of rotten eggs, (was/were) a pack
of philosophy majors gathering cans for recycling.

2. Pronoun Reference
1. Whenever Jason and Alexander sit down at a buffet, he eats way more food.
2. Even if a student gets in early, they still have to maintain good grades during senior
year.
3. At the police station, they found a pile of cash stashed in her bra.
4. She always takes an hour in the bathroom, and this completely ticks me off!
5. Although it is small and furry, koalas are able to protect themselves from predators
by quickly climbing trees.

3. Run-Ons
1. He was hungry, he bought a Chipotle burrito.
2. In New York, the train system is difficult to learn, however, the food is fantastic and
diverse.
3. He believed that a career in nursing would guarantee a stable job, Joseph applied to
medical school.

4. Modifiers
1. The magician dazzled and surprised the audience members wearing a cloak and top
hat.
2. Decorated with colorful ornaments and stars, we took pictures by the Christmas tree.
3. After missing an easy goal, the crowd booed the soccer player.

255
5. Parallelism
1. I respect his eloquence and that he is brave.
2. In chess, remember these three goals: get your pieces to the center, capture the
opposing pieces, and attacking the opposing king.
3. To learn what it means to love someone is accepting the flaws of others.

6. Tenses
1. Although the cheetah holds the record for fastest land animal, many other mammals
outlasted it.
2. Whenever we stopped by the market, my mom always tries to negotiate the prices.
3. Every Sunday, Jane cleans the house and does the laundry at the same time her
dad could have mowed the lawn.

7. Sentence Fragments
1. In the middle of the night, when most people are sleeping while I sneak to the
kitchen to eat.
2. Although pandas are one of the most likable mammals but are one of the most rare.

8. Shift in Point of View


1. Even when we arrive ahead of time at the doctor's office, he makes you wait at least
15 minutes.
2. If someone wants to play tennis, you should know how to serve.
3. If one does not believe, you will not succeed.

9. Idioms
1. The Olympic athlete was capable in climbing Mt. Everest.
2. The public was opposed against the war.
3. The children were prohibited against playing outside at dark.
4. Unless you comply to those food safety standards, we will shut you down.

10. Redundancy
1. The reason why red pandas have ringed tails is because they are relatives of both
the giant panda and the raccoon.
2. After hearing the spy's information, the general knew that an attack was imminent in
the future.
3. It's only on the night before the test that I wish my notes had been more clearer.
4. The legal documents were reviewed in a way that was deemed thorough.

256
5. We should evacuate the building immediately in the hypothetical event that a fire
occurs.

11. Word Choice


1. The startup didn’t become financially beneficial until it reached a critical mass of
customers using the app on a daily basis.
2. A recently passed law requires that public transportation meet new safety standards,
forcing state governments to foot the bill for the construction of new railroads.
3. Be careful when driving under averse weather conditions.
4. It's easy to fool your brain with a few cleverly drawn diagrams called
optical allusions.
5. Please be discrete with what I tell you because you don't want these dark secrets to
ever come out.

12. Commas
1. Because she's been so busy I haven’t seen her in a month.
2. His hobbies included jumping off planes, crashing helicopters and eating jellyfish.
3. Great white sharks the most fearsome creatures of the sea are actually less
dangerous than they appear.
4. Stephen King's first novel Carrie was a surprise success.
5. Crowds stood in line to see author, J.K. Rowling, at the bookstore in London.
6. Lions are carnivorous or meat-eating mammals.
7. Penguins unlike most other birds cannot fly.
8. Most bats are blind. Their sense of hearing however is amazing.

13. Dashes
1. When my teacher found the cookies I was hiding; all 154 of them; she ate them all
herself.
2. The city is full of people you would never meet in my hometown bums, actors,
models, the crazy, the oddly dressed.
3. I like to walk everyday—not for exercise—but for alone time.

14. Colons
1. Tokyo is one of the cleanest cities in Asia, the street cleaners sometimes have no
work to do.
2. Cambridge is home to two of the best universities in the world MIT and Harvard.
3. The dangerous animals you have to watch out for are: lions, tigers, and pythons.
4. The evidence consists of: emails, text messages, and phone calls.

257
15. Apostrophes
1. Tonys hat is on the floor.
2. Louis' scarf is 3 feet long.
3. Both players's jerseys were soaked with sweat.
4. The book has a cool picture on it's cover.
5. He is the actor whose most known for his role in Batman.
6. Jake wasn't at the office, so he must of gone to the store.

16. Transitions
1. Although women in cities from New York to Boston demanded equality in academic
opportunities, most East Coast universities did not yield to such demands. In
fact, coeducational balance did not become a prominent issue for East Coast
admissions officers until the 1960s.

A) NO CHANGE
B) In addition,
C) For example,
D) Be that as it may,

2. As it turned out, Senator Aldrich did not plan his Jekyll Island trip for relaxation
purposes. Therefore, he confidentially planned the weeklong affair to confer with
Wall Street executives for a specific purpose—to draft a banking reform bill that
would create a centralized American banking system.

A) NO CHANGE
B) Nevertheless,
C) Instead,
D) Afterwards,

3. Some conservatives claim that America was founded as a Christian nation by devout
men who sought to establish a system of law and governance based on the Bible.
More secular voices, in summary, have argued that the "Christian nation" concept is
a misnomer.

A) NO CHANGE
B) likewise,
C) for instance,
D) on the other hand,

17. Combining Sentences


1. Since Joan took office as governor, some public issues have been solved in very
clever ways. Others have been completely ignored.

258
A) ways, whereas others
B) ways, since others
C) ways, provided that others
D) ways, considering that others

2. Jojoba oil is made from the seeds of jojoba plants. These plants are sometimes
mistaken for boxwood shrubs.

A) plants; these
B) plants, for they
C) plants, which
D) plants as they

3. Located near the abundant coasts of Maine, the town of Portland is a famous
fishing port. That is wherelobstermen set their traps in the early morning.

A) port, which is where


B) port, the location where
C) port, a place at which
D) port where

4. The online retailer Amazon completely has changed how many consumers
purchase goods. They managed to make the whole online buying and selling
experience trustworthy and easy.

A) goods; they made


B) goods by making
C) goods to make
D) goods, and this made

18. Who vs. Whom


1. Jane is the girl for who I brought these gifts.
2. The chaperones who the students were assigned to made sure they walked in a
single file.
3. The librarian yelled at the boy whom never returned his books

259
SYNTAX

In 1962, Michael Murphy and Dick Price founded the Esalen Institute. The Big Sur, California location was one
of the first homes of the Western New Age movement.
Which choice most effectively combines the two sentences at the underlined portion?

A. Institute; the location, in Big Sur, California, was


B. Institute in Big Sur, California,
C. Institute in Big Sur, California; the location was
D. Institute; located in Big Sur, California, it became

Capitalism is a controversial economic system. The argument that capitalism rewards innovation and hard work
has been made by some people, while others believe it encourages greed.
Which choice most effectively combines the sentences at the underlined portion?

A. system, because some people are arguing that capitalism, rewarding innovation, also rewards hard
work,
B. system: some argue that it rewards innovation and hard work,
C. system: the argument that it rewards innovation and hard work has been made by some people,
D. system, because rewarding hard work and innovation is an advantage some people argue for,

Rachelle and Lydia did not know what to expect on their first catamaran trip to the Channel Islands. Therefore,
they were surprised and thrilled to see dolphins leaping out of the water.
Which choice most effectively combines the two sentences at the underlined portion?

A. The catamaran trip to the Channel Islands was Rachelle and Lydia's first trip, and they did not know
what to expect, so
B. The Channel Islands trip was the first catamaran trip Rachelle and Lydia would be taking, so they did
not know what to expect, and
C. Rachelle and Lydia did not know what to expect on their first catamaran trip to the Channel Islands, so
D. Rachelle and Lydia, on their first catamaran trip to the Channel Islands, did not know what to expect, so

Yesterday, I drove to work past the lemon tree. It was, as is usual, covered with fruit.
Which choice most effectively combines the sentences at the underlined portion?

A. tree; as usual, it was


B. tree; it was, as is usual,
C. tree, which was, as usual, being
D. tree, and, as usual, it was

The Oarfish, a serpentine deep-sea fish, has long been featured in Japanese folklore as a predictor of
earthquakes. Their predictive power comes from living close to the ocean floor. With that proximity to the sea
bottom, Oarfish can pick up the sensitive vibrations of tectonic plate movement.
Which choice most effectively combines the sentences at the underlined portion?

A. floor, where they


B. floor; with that proximity to the sea bottom, Oarfish
C. floor; Oarfish, because they have such a close proximity to the sea bottom,
D. floor, where, because they have such a close proximity to the sea bottom,

260
Effective language use: Precision and concision.

1.
More than half a decade after the magnitude 7.0 earthquake that leveled Haiti, serious public health and social
problems tolerate. These include inadequate housing, political corruption, and the spread of cholera.

A. NO CHANGE
B. pursue
C. persist
D. keep going

2.

Some residents were surprised by how Senator Nikita Kapoor voted on the tax bill, but she argued that her
intentions had been crystalline from the beginning of her campaign.
A. NO CHANGE
B. clear
C. limpid
D. sheer

3.

Voltaire was both disappointed and excited when he found out that his favorite coffee shop, Candid Coffee, was
moving to a new and larger point.

A. NO CHANGE
B. position.
C. location.
D. orientation.

4.

Sahil believed in the importance of excellent customer service, but it was a challenge to remain professional
when the customer ordered a fruit smoothie, complained that it tasted like fruit, and then dumped the drink on
the floor when she was denied a refund.

A. NO CHANGE
B. an impediment
C. an obstruction
D. an opposition

5.

Numerous research projects, including the benchmark Framingham Heart Study, demonstrate that taking a
vacation can have well-connected health benefits.
A. NO CHANGE
B. important
C. prestigious
D. weighty

261
SAT GRAMMAR REVIEW

I. Nouns

 Error in subject-verb agreement These errors occur frequently in


 Error in noun agreement Identifying Sentence Errors questions.

The subject is the noun that is “performing” the verb. The verb must agree with the subject in
number. Singular subjects take singular verbs, and plural subjects take plural verbs.
 Your cat bites me. (singular)
 Your cats bite me. (plural)

Sometimes multiple subjects perform the verb together. This is called a compound subject. A
compound subject is joined together by the word ‘and’, and takes a plural verb.
 Tony and I went to the store.
 Peter Petrelli and Hiro Nakamura travelled together through time.

If the word ‘and’ is not present, you have a subject plus one or multiple tag-alongs. Tag-alongs
often occur with the words and phrases of, in, as well as, with, along with, together with, in addition
to, no less than, rather than, and like. These do not change the relationship between the true subject
and the verb.
 The juice in those bottles has expired.
 The house made of bricks and cement was built by my grandfather.
 Robert along with Hafeez, Michael, and John is going to see the new Judd Apatow movie.
 My mother, like your parents, enjoys gardening.
 The farmer together with the milkman has gone to visit my grandmother.
 Alice as well as Gregory enjoys mathematics.

In addition to subjects, any other corresponding nouns in the sentence must agree in number.
 Parker and Hanley studied hard to become a great lawyer. (incorrect)
 Parker and Hanley studied hard to become great lawyers. (correct)

It is easy to confuse the singular and plural forms of certain nouns. Here are some to keep straight:
Singular  alumni
 alumnus/alumna  antennae
 antenna  criteria
 criterion  data
 datum  phenomenon
Plural  medium

Ivy Global
262
 millennium  media
 symposium  millennia
 phenomena  symposia

II. Pronouns

 Error in pronoun case


 Error in pronoun-antecedent agreement These errors occur frequently in
 Error in pronoun-verb agreement Identifying Sentence Errors questions.
 Error in pronoun consistency

Pronouns are words that take the place of and refer back to previously mentioned antecedent
nouns. Like their noun counterparts, pronouns can take a subject or object role in the sentence.
Each personal pronoun has a nominative (subject) and objective (object) case.
 She and I went to the store. (nominative)
Nominative Objective
 Jerry took him and me to the dance. (objective)
I me
we us
If you’re unsure, test with a single pronoun.
you you
he him
Who is a subject pronoun, and whom is an object pronoun.
she her
 Who went to the store? (nominative) it it
 Whom did Jerry take to the dance? (objective) they them
who whom
Pronouns that come after a preposition (by, of, for, after, with,
between, except, without, etc.) are considered objects of the preposition and take the objective
case.
 Everyone did well on the exam except him and her.
 Sarah left without John and me.
 Please keep this between you and me.
 By whom was this wonderful short story written?

Constructions with than: whenever a pronoun follows than, it should be in the subjective case. A
following verb is always understood, if not always articulated.
 I am taller than he (is).
 Sarah is better at math than I (am).

Pronouns must agree in number with their antecedents. If the antecedent is singular, the pronoun
must be singular. If the antecedent is plural, the pronoun must be plural.
 A student will catch their mistakes if they proofread. (incorrect)

Ivy Global
263
 A student will catch his or her mistakes if he or she proofreads. (correct)
 Students will catch their mistakes if they proofread. (correct)

Verbs must agree in number with subject pronouns. It is easy to make a mistake with some of the
less basic pronouns. When in a subject position, some of these pronouns always take a singular verb
and others always take a plural verb.
 Each of the apples you bought is bruised. Singular Pronouns
 Everything that my uncle does turns out to be a success. each either
 Neither of the twins knows how to cook. neither someone
 Nobody is at home. anyone everything
 Many are cold, but few are frozen. somebody nobody
anybody everyone
Exception! In neither … nor and either … or constructions, the Plural Pronouns
verb agrees in number with the closest noun or pronoun. many few
 Neither the twins nor Laura was willing to take the blame. several
 Either Mary or her parents are responsible for the flood.

Keep pronouns consistent. Don’t change between “we,” “you,” “they,” “he or she,” and “one” in a
single sentence.
 If one doesn’t study, your grade will drop. (incorrect)
 If one doesn’t study, one’s grade will drop. (correct)

III. Verbs

 Error in verb tense


 Error in verb form or conjugation These errors occur frequently in both
 Misuse of passive voice Identifying Sentence Errors and
 Error in gerund usage Improving Sentences questions.

The tense of a verb indicates when the action of the sentence takes place. Use the present tense
for action that is currently occurring, action that generally occurs, and action that takes place in
literature (novels, movies, comics, short stories, poetry, non-fiction books, etc).
 I always eat breakfast at home.
 In Shakespeare’s play, Macbeth murders Duncan after he hears the prophecy.

There are several ways to indicate action that occurs in the past. Use the regular past tense for
completed actions in the past. Use the imperfect past tense (was + present participle) for actions
that were continuous in the past.

Ivy Global
264
 I ran.  I was running.
 He drank.  He was drinking.
 She swam.  She was swimming.

Both the regular and imperfect past tense can be combined for actions that were simultaneous in
the past. Connect these with words like as, when, and while.
 I was swimming when my cell phone rang.
 She broke her arm while skating.

The perfect tense (have + past participle)describes an action that you have done in the past, but
are talking about in the present.
 Yes, she has seen that movie.
 I have walked one hundred miles and I will walk one hundred more…

The pluperfect or past perfect tense (had or had been + past participle) describes action that
occurred before another action in the past. Do not leap into the past perfect tense when one of your
actions occurs in the perfect, present, or future tense.
 I will not go with Sohana because I had already seen the movie. (incorrect)
 I didn’t go with Sohana because I had already seen the movie. (correct)
 Chris had already read the book, but he will read it again. (incorrect)
 Chris had already read the book, but he decided to read it again. (correct)

To describe a hypothetical action in the past tense, use would rather than will.
 I knew that I will win the game. (incorrect)
 I knew that I would win the game. (correct)

Use the subjunctive tense (formed by the past tense were) to express wishes, particularly with the
word if.
 If I was you, I would take a vacation. (incorrect)
 If I were you, I would take a vacation. (correct)

Transitive verbs take a direct object. Intransitive verbs take an indirect object or no object at all.
Certain verbs can be both transitive and intransitive without changing form or conjugation:
 I am reading a book. (direct object)  I am reading. (no object)
 He made an error. (direct object)  He made the man cry. (indirect object)

Certain verbs look similar, but have very different forms depending on whether they are transitive
or intransitive. Raise and lay (present tense) take direct objects. Rise and lie do not take direct
objects. However, lay can also be the past tense of lie. The past tense of lay is laid.
 Jill raises her hand.  I lay my coat on the bed.
 They raised their children well.  I laid my coat on the bed.

Ivy Global
265
 The sun rises in the east.  I think I should lie down.
 The farmers rose at dawn.  I lay down immediately.

The passive voice is not a verb tense but an inverted sentence order. In this construction, the noun
performing the action does not appear in the normal subject position. Passive construction often
leads to wordiness and lack of focus; re-write passive sentences in the active voice.
 Another helping was asked for by my friend. (passive)
 My friend asked for another helping. (active)
 Three finalists for the open position have been selected by the administration. (passive)
 The administration has selected three finalists for the open position. (active)

Gerunds are –ing verb forms that act as nouns. Nouns or pronouns preceding gerunds should be in
the possessive case, as they technically modify the gerund rather than act as a subject to a verb.
Gerunds are required in certain idiomatic phrases.
 He constantly babbling drives me crazy. (incorrect)
 His constant babbling drives me crazy. (correct)
 The tiger growling frightens the small animals. (incorrect)
 The tiger’s growling frightens the small animals. (correct)
 Experts suggest to water your garden later when the sun is less intense. (incorrect)
 Experts suggest watering your garden later when the sun is less intense. (correct)

IV. Sentence Structure and Punctuation

 Sentence fragments
 Run-on sentences These errors occur most frequently in
 Error in conjunction usage Improving Sentences questions.
 Error in punctuation: commas,
semicolons, colons, apostrophes

Clauses come in two forms: independent and dependent. Independent clauses can stand alone as
sentences.
 I saw a movie with Danny.
 We meet for coffee once a week.

Dependent clauses do not stand alone. Instead, they provide extra interest in the sentence.
 After I completed my homework, I saw a movie with Danny.
 In our efforts to be good friends, we meet for coffee once a week.

Ivy Global
266
A dependent clause on its own is a sentence fragment, and needs to be altered to make a complete
sentence.
 When I finished this slice of pizza. (sentence fragment)
 I finished this slice of pizza. (complete sentence)
 Because I detest Sarah. (sentence fragment)
 Because I detest Sarah, I try to avoid her whenever I can. (complete sentence)

Independent causes can be separated by periods, connected by semicolons, or joined by


coordinating conjunctions. If you try to connect two independent clauses with only a comma, you
have a comma splice. If you try to connect them with nothing, you have a run-on sentence.

 The teacher is very angry I think he is going to call my


Coordinating Conjunctions
friend’s parents. (run-on sentence) and or
 The teacher is very angry; I think he is going to call my but nor
friend’s parents. (correct) so for
 It was a beautiful day outside, I didn’t want to stay in and yet
do homework. (comma splice)
 It was a beautiful day outside, so I didn’t want to stay in and do homework. (correct)
 I finished the slice of pizza, I felt my stomach turn. (comma splice)
 After I finished the slice of pizza, I felt my stomach turn. (correct)

An independent and a dependent clause can be joined together using a subordinating


conjunction. In this construction, one idea depends on the other. Coordination and subordination
test our ability to see logical relationships between ideas. Thus, we
must understand and determine how the ideas in the clauses work Subordinating
together: does one give supporting or contrasting information? Is Conjunctions
after rather than
there a progression in time and sequence? There might be two or
although since
more ways to express the relationship, but often only one will be
as so that
correct, clear, and concise.
as if than
 It was snowing, but I wore my boots. (incorrect)
as long as that
 Because it was snowing, I wore my boots. (correct) as though though
 Henry tried to read War and Peace in the original Russian, because unless
and it was too difficult. (incorrect) before until
 Henry tried to read War and Peace in the original Russian, even if when
but it was too difficult. (correct) even though whenever
 Although Henry tried to read War and Peace in the original if where
Russian, it was too difficult. (correct) if only whereas
in order that wherever
However, consequently, furthermore, therefore, instead, thus, as a now that while
result and similar words are conjunctive adverbs and cannot be once
used in the same way as coordinating or subordinating

Ivy Global
267
conjunctions. They can be used to introduce an independent clause following a period or a
semicolon. They can also act as interrupters, separated from the main sentence with commas.
 John didn’t read the assigned homework, consequently he failed the exam. (incorrect)
 John didn’t read the assigned homework. Consequently, he failed the exam. (correct)
 We decided not to go out tonight, instead we are going to save our money. (incorrect)
 We decided not to go out tonight; instead, we are going to save our money. (correct)
 Sarah is normally clumsy, however she is a good skater. (incorrect)
 Sarah is normally clumsy; she is, however, a good skater. (correct)

Semi-colons are used in only two contexts: to join two independent clauses with or without a
conjunctive adverb, or to separate very lengthy items in a list. Use a colon to introduce examples or
items in a list.
 Four people worked on the project; only one received credit for it. (correct)
 Victoria was frequently tardy; therefore, she received a low grade. (correct)
 Elaine has three daughters; Amy, Michaela, and Christine. (incorrect)
 Elaine has three daughters: Amy, Michaela, and Christine. (correct)
 Elaine has three daughters: Amy, who is studying law at Harvard; Michaela, who is working
as a masseuse; and Christine, who is still in high school. (correct)

As we have seen, commas are used to join together two clauses with a conjunction. Commas are
also used after introductory phrases and around interrupters (words, phrases, and clauses that
interrupt and are not essential to the core meaning of the sentence). Commas should never come
between a subject and its verb.
 Mary, decided to relax with a good book. (incorrect)
 Mary decided to relax with a good book. (correct)
 After a long day at work, Mary decided to relax with a good book. (correct)
 Mary a paediatrician really enjoys her work. (incorrect)
 Mary, a paediatrician, really enjoys her work. (correct)

Apostrophes are used to form contractions by taking the place of a missing letter or number; to
form plurals of letters, figures, and numbers; and to indicate ownership with an ’s. To form a
possessive for a noun already ending in s, add ’s if the noun is singular; add only an apostrophe if
the noun is plural. Keep in mind that not all plural nouns end in s. Show possession in the last word
for names of organizations and businesses, for hyphenated words, and for cases of joint ownership.
 We’re going out of town next week. (contraction)
 My husband was in the class of ’89. (contraction)
 My street address contains three 5’s. (plural of a number)
 She has a hard time pronouncing r’s. (plural of a letter)
 The drummer’s solo was fantastic. (singular possessive)

Ivy Global
268
 Mr. Perkins’s persuasive essay was very convincing. (singular possessive)
 The customers’ access codes are confidential. (plural possessive)
 Men’s shirts come in a variety of neck sizes. (plural possessive)
 Brad and Janet’s graduation was three months ago. (joint ownership)
 I went to visit my great-grandfather’s alma mater. (hyphenated word)
 The Future Farmers of America’s meeting was moved to Monday. (name of organization)

Possessive pronouns never use apostrophes. A pronoun with an apostrophe always signifies a
contraction with the word is or are.
 Its fur is thick.  It’s a beautiful day.
 Whose book is this?  Who’s in charge here?
 Your shirt is a great color.  You’re in a great mood today.
 I enjoyed their presentations.  They’re horrible players.
 Is this sandwich yours or theirs?

V. Sentence Logic

 Misplaced modifiers These errors occur most frequently in


 Error in parallel structure Improving Sentences questions.
 Faulty comparisons

A modifier is a word, phrase, or clause that modifies or describes a noun or action in the sentence.
Modifiers need to be placed as close as possible to the word they are modifying.
 Walking to the store, the street seemed so quiet and peaceful to Laura. (incorrect)
 Walking to the store, Laura thought the street seemed so quiet and peaceful. (correct)
 Unable to beat the enemy through brute force alone, deception seemed necessary. (incorrect)
 Unable to beat the enemy through brute force alone, the army decided that deception was
necessary. (correct)
 Although a small dog, my sister found her new puppy a big responsibility. (incorrect)
 Although her new puppy was a small dog, my sister found him a big responsibility. (correct)

When you have multiple items in a list or in a comparison, these must be stated in a similar—or
parallel—manner.
 Andrew enjoys swimming, skating, and to play golf. (incorrect)
 Andrew enjoys swimming, skating, and playing golf. (correct)
 Kate is more ambitious, but Emily shows more patience. (incorrect)
 Kate is more ambitious, but Emily is more patient. (correct)

Ivy Global
269
Comparisons must be complete and logical. Always compare similar items.
 The rooms on the second floor are larger than the first floor. (incorrect)
 The rooms on the second floor are larger than those on the first floor. (correct)
 Jackson Pollack’s paintings are better than any other painter. (incorrect)
 Jackson Pollack’s paintings are better than paintings by any other painter. (correct)
 Some students prefer watching videos to textbooks. (incorrect)
 Some students prefer watching videos to reading textbooks. (correct)
 David Letterman is the best talk show. (incorrect)
 David Letterman’s talk show is the best talk show. (correct)

Use the comparative ‘er’ ending to compare two items. Use the superlative ‘est’ ending to compare
three or more items.
 There are two children in the family. Jo is the elder and Paul is the younger. (comparative)
 There are four children in the family. Jo is the eldest and Paul is the youngest. (superlative)
 Of the couple, Eric is the better cook. (comparative)
 Of all of his siblings, Eric is the best cook. (superlative)

Avoid double comparisons and double negatives.


 This is the most longest I’ve ever waited for a pizza. (incorrect)
 This is the longest I’ve ever waited for a pizza. (correct)
 This isn’t hardly the best pizza in town. (incorrect)
 This isn’t the best pizza in town. (correct)

VI. Diction

 Wordiness and redundancy These errors occur in both Improving


 Adjective/adverb confusion Sentences and Identifying Sentence
 Confused word pairs Error questions, though wordiness
 Error in idiom occurs almost exclusively in Improving
Sentences questions.

Wordiness is caused by unnecessary “filler” or repetition, inexact phrases, and overly complicated
sentence structure. The more concise, the better, as long as all the necessary information is
conveyed.

Replace the following wordy and redundant phrases with a more concise alternative:
 a lot of → many or much  are able to → can
 all of a sudden → suddenly  at the present time → currently/now
 along the lines of → like  basic fundamentals → fundamentals
 any and all → all  both of these → both

Ivy Global
270
 close proximity → close  often times → often
 due to the fact that → because  on a daily basis → daily
 end result → result  on account of the fact that → because
 final destination → destination  past history → history
 final outcome → outcome  rarely ever → ever
 first and foremost → first  take action → act
 general consensus → consensus  the majority of → most
 important essentials → essentials  the reason why → the reason
 in order to → to  through the use of → through
 in the event that → if  true facts → facts
 in the near future → soon  various differences → differences
 in the neighborhood of → about  with regard to → about/regarding
 last but not least → finally  with the exception of → except for

Avoid the following wordy phrases and “filler”:


 as a whole  by definition
 as the case may be  for all intents and purposes
 as a matter of fact  it is clear that
 being that  the fact that
 by and large  really, totally, very

Since adjectives and adverbs serve similar functions – they both modify or describe – they are often
confused. Remember that adjectives modify nouns or pronouns while adverbs modify verbs,
adjectives, and other adverbs. Adverbs often end in –ly.
 He was a quick study.  He learned quickly.
 He had a violent temper.  He was a violently angry person.
 I have a good dog.  My dog is well trained.

Use less and amount for non-countable items (water, air, intelligence, etc.). Use fewer and number
for items that can be counted.
 There is less grass on the soccer field.
 I scored fewer goals than you.
 There is a greater amount of water in this lake.
 There is a greater number of ducks on this lake.

Use between when talking about two items only. Use among for three or more.
 Keep this between you and me.
 There was peace among the three countries.

Other commonly confused word pairs:

Ivy Global
271
 accept : receive, agree to  except: exclude
 adapt: evolve  adept: skilled
 affect: verb  effect: noun
 allusion: reference  illusion: false representation
 already: previously  all ready: ready to go
 beside: next to  besides: in addition
 conscious: aware  conscience: moral sense
 disinterested: impartial  uninterested: not interested in
 eminent: important, esteemed  imminent: upcoming
 farther: distance  further: quantity
 latter: second in a list of two items  later: time
 loose: not tight, not fastened  lose: misplace
 principal: chief  principle: rule
 than: comparison  then: time

Neither is always used with nor, and either is always used with or. The correct structure following
not only is but also.
 Neither Amanda nor Jarome received an ‘A’ on that assignment.
 Leave your paper either with me or with the front office.
 Not only did he wash the dishes, but he also walked the dog.

An idiom is an expression characteristic of a particular language. Many idioms involve preposition


choice. Using the wrong preposition is a grammatical error. Be familiar with some of these:
 abide by the rule  comply with a request
 accuse someone of a crime  contrast with (when noting
 agree on an amendment differences)
 agree to do something  correspond to or with something
 agree with someone (meaning to relate)
 amazement at something  correspond with a person (meaning to
 appreciation of something communicate)
 argue about or for a proposal  differ from something
 argue with a person  independent of someone or
 apologize for an error something
 approve of a change  interested in a subject
 blame a mistake on a person  oblivious to an issue
 blame a person for a mistake  separate from something
 bored with small talk  similar to something
 concerned about or with an issue  succeed in an endeavour

Ivy Global
272
Commonly Confused Words

Words evolve with use and misuse over time, sometimes obscuring their meanings. Here are current
conjugations and clarifications of some commonly confused words. For more, see the sources we used
to compile this list: The Everyday Writer, 5th edition; Writers, Inc.; The Chicago Manual of Style, 16th
edition; and Merriam-Webster’s Dictionary of English Usage.

accept, except
Accept is a verb that means to receive or agree to. The principal accepted the boy’s story about the
broken window. Except is usually a preposition that means aside from or excluding. I want all of the
mugs to be washed except for the blue one.

affect, effect
As a verb, affect means to influence. His money affected the way people treated him. Used in the past
as a noun also, though almost exclusively in the mental health field, affect referred to a person’s mood.
The patient presented with a somber affect. Effect is usually a noun meaning a result. The drug had
several adverse side effects. But it is also used as a verb meaning to cause. The new laws effected less
cultural change than anticipated.

allusion, illusion
An allusion is an indirect reference. Did you catch my allusion to Shakespeare? An illusion is a false or
misleading appearance. Mirrors give the room an illusion of depth.

assure, ensure, insure


Assure and ensure are generally interchangeable, though assure is more often used for people. I assure
you, sir, I am unarmed. Please ensure that the lid is tight. Insure is almost exclusively used in a financial
sense. Have you insured your car yet?

a while, awhile
A while is a noun phrase. Let’s sit for a while. Awhile is an adverb. Let’s sit awhile. Many agree this is a
nitpicky distinction, since the only difference seems to be the use of a preposition before a while.

bring, take
If the action is directed toward you, use bring. Bring home the bacon. If the action is away from you, use
take. Take out the trash.

censer, censor(n), sensor


A censer is either a container of burning incense or the person who carries it. A censor is someone who
suppresses objectionable subject matter. A sensor is a mechanical or electronic detector.

censor(v), censure
To censor is to suppress objectionable subject matter. Many school libraries censored The Adventures
of Huckleberry Finn. To censure is to criticize strongly or disapprove. The press will often censure the
government if they disagree.

conscience, conscious
Conscience is a noun referring to a sense of right and wrong. I would have stayed in bed, but my
conscience said I should get up. Conscious is an adjective meaning awake or aware. Being conscious
now of the light, how could I live in the dark?

LWC Writing Center


Slider 200, 270-384-8209 • Every Writer, Every Message, Every Point in the Process • Welcome to the Conversation!

273
Commonly Confused Words

continuous, continual
Continuous means constant, without interruption. Continual means recurring or frequently repeated.

could have, could of


Promoting the confusion is the contraction “could’ve,” which sounds like “could of.” In formal prose,
have should follow “could,” “would,” “should,” or “might.” Compromise could have ensured our
success.

different than, different from


People commonly use than with different, but than is used for comparisons, and different is not
comparative. Different from is preferred. The second blast was not bigger, just different from the first.

elicit, illicit
The verb elicit means to evoke or draw out. The police elicited from the criminal the names of his
accomplices. The adjective illicit means illegal. George was guilty of committing illicit acts.

emigrate, immigrate, migrate


Emigrate means to move away from one’s country. We emigrated from Canada in 2002. Immigrate
means to move into another country. We immigrated to the United States. Migrate, when referring to
people, means to relocate within the same land. Thousands of pioneers sought their freedoms by
migrating west. For animals, migrate means to relocate. Geese spend much of the spring and fall
migrating between warm and cool climates.

eminent, imminent, immanent


Eminent means prominent or famous. We have gathered this evening to honor three eminent authors.
Imminent means pending or expected soon. Sir, core breach is imminent. Immanent means inherent or
ever-present. God’s immanence is most apparent in the phenomenon of creation.

empathy, sympathy
Empathy is a psychological attachment in which one imagines another’s experience. Sympathy is
compassion for or emotional alignment with a person or other entity.

every day, everyday


Every day is a noun phrase. What time do you wake up every day? Everyday is an adjective. How do
you like my everyday attire?

every one, everyone


Every one is a noun phrase meaning each. Every one of the flowers bloomed. Everyone is a noun
meaning all, and usually refers exclusively to people. Everyone finished the course with ease.

fewer, less; many, much; number, amount


Use fewer, many, and number with nouns that can be counted. I have fewer pieces than you do. Use
less, much, and amount with general amounts that cannot be counted. I have less candy than you do.

height, heighth
While both of these spellings are etymologically sound (heighth is an Old English variant of height), most
sources recommend using the more commonly accepted spelling height.

LWC Writing Center


Slider 200, 270-384-8209 • Every Writer, Every Message, Every Point in the Process • Welcome to the Conversation!

274
Commonly Confused Words

I, me; he, him; who, whom


I refers to the subject (the actor) in a sentence or clause; me refers to the object (the thing or person
being acted upon). Who will attend the conference, he or I? Please send your abstract to my assistant or
me. The same applies for he(s), him(o); who(s), whom(o); and whoever(s), whomever(o).

I am done, I am finished
Pedants pounce on I am done, often “correcting” it with I am finished. To get even more persnickety,
whether one uses done or finished is not really the issue. The confusion—if you’re really confused—is
caused by the passive voice of the sentence. The construction I am done uses the passive voice to say
that someone or something did you. However, I have finished keeps the sentence in active voice and
the meaning clear. Of course, it could also be argued that the construction I am done treats the word
done as an adjective describing the state of having finished, in which case I am done would be just fine.

infer, imply
To imply something is to hint it, basically. But that statement implies that my involvement was
deliberate. To infer something is to gather it, surmise it, or come to the conclusion. Am I to infer,
Governor, that if we do not surrender our land we will be evicted or killed?

lay, lie
Lay means to put or place. It takes a direct object specifying the thing that is put or placed. She laid her
books on the desk. Lie does not take a direct object. For living things, lie means to recline. She lay
awake until two. For nonliving things, it means to exist or be located. That valley lies to the north. The
confusion is caused by the conjugation of lay and lie. Here are the verb forms:

Base Form Past Tense Past Participle Present Participle -S Form


Lie (recline) lay lain lying lies
Lay (put) laid laid laying lays
th
Adapted from The Everyday Writer, 5 edition (346). A. A. Lunsford, 2013, Boston: Bedford/St.Martin’s.

led, lead
Led is the past-tense form of the verb lead (to direct or escort). Lead is the element.

log in, login, log-in


Use log in as a verb phrase. Go ahead and log in. Login and log-in are used as nouns or adjectives. Use
your own login information.

okay, OK
Okay is a younger variant of OK, but either is acceptable if kept consistent.

phenomena, phenomenon
Phenomena is the plural form of phenomenon.

than, then
Use then with sequenced events. I laughed, and then I cried. Use than in comparisons. The cat was
bigger than the dog.

LWC Writing Center


Slider 200, 270-384-8209 • Every Writer, Every Message, Every Point in the Process • Welcome to the Conversation!

275
Commonly Confused Words

there, their, they’re


The adverb there is used to point out a location. Do you see them over there? Their is a possessive
pronoun. Their dog is always chasing cars. And there is the contraction for “they are.” They’re quite a
couple.

to, too
To is a preposition meaning toward or in the direction of. Are you going to the store? It can also be
used to form an infinitive verb. The divers must be careful to swim. Too has two meanings: also (I like
candy too) and to an excessive degree (In fact, I like it too much).

toward, towards
The preferred form is without the s in American English, with the s in British English. The same is true
for other directional words, such as upward, downward, backward, and forward.

uninterested, disinterested
Uninterested means not interested. She was uninterested in my art. Disinterested means unbiased.
Finding a disinterested judge in this matter may prove difficult.

utilize, use
Utilize, which means to use to the best effect, is an overused version of use and is only occasionally the
better choice. Use is generally the best choice for simplicity.

when, whenever
These are not interchangeable. Use whenever only to emphasize uncertainty in the time of an event.
Well whenever you get here, I guess just I’ll be waiting. Use when in all other cases. When I was ten, I
went to Disneyworld.

your, you’re
Your is a possessive pronoun. Bring your sleeping bag along. You’re is the contraction for you are.
You’re in the wrong sleeping bag.

LWC Writing Center


Slider 200, 270-384-8209 • Every Writer, Every Message, Every Point in the Process • Welcome to the Conversation!

276
Commonly Confused Words
Good communication involves using words appropriately to articulate your ideas and support
your assertions. Using words correctly will also ensure that your reader will not be distracted
from the content of your writing. Below are some of the most commonly confused English word
pairs and their meanings.

accept/except: Accept is a verb; it means to receive. “We accept your invitation with pleasure.”
Except, is usually a preposition, and it means to leave out. “My grades are satisfactory in
every subject except English.”

adapt/adopt: Adapt means to change in order to fit, or be more suitable; to adjust. “To survive,
an animal must adapt to its environment.” “Some adult novels have been adapted for
young readers.” Adopt means to take something and make it one’s own. “They adopted
their physician’s suggestions and adopted a baby.”

affect/effect: In general, affect is a verb that means to influence or to cause change (it can be
used as a noun when referring to an emotional response). For example: “Her campaign
strategy affected the outcome of the election.” Effect is a noun that signifies a result
brought about by an action. “The study examined the effects of sugar on children’s
behavior.” Occasionally, effect is used as a verb when it means to bring about, as in “Her
efforts effected a change.”

amount/number and less/fewer: Amount and less can only be used with uncountable quantities,
such as the “amount of peanut butter,” or “less peanut butter.” For countable things, use
number and fewer: “the number of peanuts” and “fewer peanuts.”

because/since: The word because denotes cause and effect. “She ran because he chased her.”
Since refers to the passage of time. “I have been a professor since 2008.”

between/among: In general, use between to show a relationship between two things. “I could
not choose between the apple or the pumpkin pie.” For relationships involving three or
more things, use among: “I could not decide among the four delicious desserts.”

capital/capitol: As a noun, capital refers to a city or it can refer to wealth or resources. “Do
you know the capital of Wyoming?” “His business had enough capital to open offices
across the country.” Capitol refers to a building where lawmakers meet. “The capitol has
undergone extensive renovations.”

compliment/complement: A compliment is an expression of praise or admiration. “I received a


compliment about my new jacket.” A complement completes a number or quantity: “We
had a full complement.” It can also mean to add to or embellish, such as: “The Persian
rug complemented the furniture in the room.”

Center for Academic Excellence, Saint Joseph College, Revised 2009 1

277
connote/denote: Connote is used to suggest unstated meaning or ideas; something implied.
“The word mother connotes unconditional love.” Denote is used to refer to the literal
meaning of something. “White flags are used to denote surrender.”

could of/ought to of/ might of/must of: These are sometimes carelessly written for could have,
ought to have/might have/must have. Remember not to use of for have. “She could have
warned me about the dangerous roads.”

data: This is the plural form of the Latin datum. In standard informal English, data is a
collective noun and should be written with a plural verb. “These data were collected by
the committee.”

desert/dessert: As a noun, a desert is an arid, sandy place. “The Sahara desert is home to
nomadic tribes.” It can also be used as a verb meaning to leave behind. “Her husband
deserted her and the children.” Dessert is that sweet treat we eat after a meal. “Monique
had blueberry pie for dessert.” An easy way to commit this to memory is to remember
that dessert has a double s and is twice as pleasant as the dry, arid desert!

due to/because of: Due to means caused by and often follows the verb “to be” (is, was, were,
am, etc.) in a sentence. For example: “The game’s postponement is due to rain.” “It was
due to the storm that the game was canceled.” Note that using due to is not a formal way
to writing because of. In general, the use of because of answers the question, “why?” and
is followed by a noun or a noun phrase. “The tow truck was late because of another
accident.”

elicit/illicit: Elicit is a verb meaning to obtain, bring out, or to evoke. Illicit is an adjective
meaning unlawful. “The reporter was unable to elicit information from the police about
illicit drug trafficking.”

emigrate/immigrate: Emigrate means to exit or to leave a country. Immigrate means to move


into a new country. “The McKays were emigrants from Scotland who immigrated to
Canada.” An easy way to remember is that emigrate begins with the letter E, as does
Exit. When you emigrate, you exit a country. Immigrate begins with the letter I, as does
In. When you immigrate, you go into a country.

ensure/assure/insure: While many sources believe that ensure, assure, and insure are
synonymous, conservative grammarians believe they each have a specific use. Assure is
used when talking to or about a person and you are implying certainty; you are putting
one’s mind at ease or removing doubt. “I assure you that I will be on time for the
wedding.” Ensure means to make certain. “An excellent outline will ensure that your
research paper is well organized.” Insure is generally used when talking about protecting
against a financial loss, such as with an insurance policy. “In case of a natural disaster,
they insured their home.”

Center for Academic Excellence, Saint Joseph College, Revised 2009 2

278
One way to help remember whether to use assure, ensure, or insure, is to remind yourself
that:
You assure a person (or something that is alive; remember the a in alive).
You ensure anything else (when you guarantee something; remember the e at the end of
guarantee).
You insure your car (remember that insurance protects income; remember the i for
income).

every day/everyday: Every day means each day. “Every day Luke walks to school.” When
you use the word everyday, it should tell the reader that something is typical or ordinary.
“Rain is an everyday occurrence in Seattle.”

famous/notorious: Famous means widely known. Notorious means widely known, but is used
in an unfavorable sense. “Katherine Hepburn and Bette Davis are among the most
famous American film actresses.” “Jesse James is probably America’s most notorious
outlaw.”

farther/further: Farther is refers to physical distance or length, whereas further means to a


greater degree or additionally. “Newington is farther south than Windsor Locks.” “Her
paper requires further research to support her thesis.”

i.e./e.g.: These are both Latin terms. I.e. is an abbreviation for id est and is used to say “in other
words” or “that is.” E.g. is an abbreviation for exempli gratia and is used to mean “for
example” or “including.”

imply/infer: Imply means to suggest. “The postcard implied that they were having fun on their
trip.” Infer means to interpret or conclude. “I inferred from the article that Jones was a
proponent of the bill.”

in/into: In indicates location or condition. “I am in the grocery store.” Into is a preposition and
generally suggests movement or direction. “At two o’clock we walked into the
principal’s office.” “Rachel found the diary in the trunk after she had moved it into the
attic.”

lie/lay: Lie means to recline or to rest on a surface. Lay means to put or place. What confuses
us when using these two words is the various verb tenses or forms they take.
The principal parts of lie (to recline) are:
lie (present tense): I lie on the children’s bed to read them a story.
lay (past tense): I lay on the children’s bed to read them a story.
lain (past participle): I had lain on the children’s bed to read them a story.
lying (present participle): I am lying on the children’s bed to read them a story.

Center for Academic Excellence, Saint Joseph College, Revised 2009 3

279
The principal parts of lay (to place or put down) are:
lay (present tense): I lay the book on the table.
laid (past tense): I laid the book on the table.
laid (past participle): I had laid the book on the table.
laying (present participle): I am laying the book on the table.

A simple strategy to remember them is to write or say them as a ditty:


lie, lay, lain, lying (to recline)
lay, laid, laid, laying (to place or put down).

like/as: Like is a preposition and introduces a prepositional phrase. “She walks like her
mother.” Like should not be used for as if or as though, which are used to imply the
opposite of what something or someone seems, or if something seems unreal. “It looks
as though he forgot to call.” “He reacts as if it were her fault.” “She is acting as if/as
though I had two heads.” As is usually a conjunction, and it introduces a subordinate
clause. “She cooks spaghetti as the Italians do.”

persecute/prosecute: Persecute means to attack or annoy someone, often for a person’s beliefs.
Prosecute means to bring legal action against someone for unlawful behavior. “Hitler
persecuted people who opposed the Nazi government.” “Trespassers will be
prosecuted.”

phenomena/phenomenon: Phenomena is the plural form of phenomenon. Do not use it as a


singular noun. “These natural phenomena are most interesting.” “This natural
phenomenon is most interesting.”

principle/principal: Principal is a noun meaning the head of a school or an organization. It can


also refer to a sum of money or an important idea. “A principal theory in education is
Piaget’s Stages of Cognitive Development.” Principle is a noun meaning a basic truth or
law. “The principal of our school taught us many important life principles.” An easy
way to remember which word to use is to remember that the principal of your school can
be your “pal.”

set/sit: Set means to put or to place. Sit means to be seated. “She set the dough in a warm
corner of the kitchen.” “I sit on the bench to put on my shoes.” “The cat sat in the
warmest part of the room.”

stationary/stationery: When something is not moving, it is stationary. People use stationery,


or writing materials, when they send a letter through the mail.

that/which/in which: That is used with restrictive clauses, which is a clause that contains
information essential to the meaning of the sentence. “The photograph that was in the
gallery was a landscape.” (Who should be used when referring to people. “I just saw a
boy who was wearing a yellow banana costume.”) Which is used with nonrestrictive
clauses. These clauses contain additional descriptive information within a sentence, but

Center for Academic Excellence, Saint Joseph College, Revised 2009 4

280
are not essential to complete the sentence. Commas are placed around the clause. “The
photograph, which was in the gallery, was a landscape.” “I have to go to math next,
which is my hardest class." Use in which to avoid ending a sentence with a preposition.
“This is the book in which the essay was referenced.”

then/than: Then is an adverb denoting time. Than is a conjunction and is used to make
comparisons. “Tom raised his hand, and then he asked a question.” “Hugh ate more
pizza than I could ever eat.”

there/their/they’re: There refers to a place. Their is a possessive pronoun and means belonging
to, as in “their shoes.” They’re is a contraction for they are.

who/whom: Who and whom are pronouns used when referring to a person. Employ who when
describing the subject of a sentence. The subject is the person performing an action.
“Christina is the student who wrote the poem.” Whom describes the object of a sentence,
or the person receiving the action. “Christina is the student to whom Robert gave the
book.”

Center for Academic Excellence, Saint Joseph College, Revised 2009 5

281
Verb Meaning Example

Brian asked Judy out to dinner and


ask someone out invite on a date
a movie.

ask many people the same I asked around but nobody has
ask around
question seen my wallet.

add up to something equal Your purchases add up to $205.32.

You'll have to back up your car so


back something up reverse
that I can get out.

My wife backed me up over my


back someone up support
decision to quit my job.

The racing car blew up after it


blow up explode
crashed into the fence.

We have to blow 50 balloons up


blow something up add air
for the party.

stop functioning (vehicle, Our car broke down at the side of


break down
machine) the highway in the snowstorm.

The woman broke down when the


break down get upset police told her that her son had
died.

Our teacher broke the final project


break something down divide into smaller parts
down into three separate parts.

Somebody broke in last night and


break in force entry to a building
stole our stereo.

The firemen had to break into the


break into something enter forcibly
room to rescue the children.

wear something a few times so I need to break these shoes in


break something in
that it doesn't look/feel new before we run next week.

The TV station broke in to report


break in interrupt
the news of the president's death.

My boyfriend and I broke up


break up end a relationship
before I moved to America.

The kids just broke up as soon as


break up start laughing (informal)
the clown started talking.

282
The prisoners broke out of jail
break out escape
when the guards weren't looking.

I broke out in a rash after our


break out in something develop a skin condition
camping trip.

This sad music is bringing me


bring someone down make unhappy
down.

My grandparents brought me up
bring someone up raise a child
after my parents died.

My mother walks out of the room


bring something up start talking about a subject
when my father brings up sports.

He drank so much that he brought


bring something up vomit
his dinner up in the toilet.

phone many different We called around but we weren't


call around
places/people able to find the car part we needed.

I called the company back but the


call someone back return a phone call offices were closed for the
weekend.

Jason called the wedding off


call something off cancel because he wasn't in love with his
fiancé.

The professor called on me for


call on someone ask for an answer or opinion
question 1.

We called on you last night but you


call on someone visit someone
weren't home.

Give me your phone number and I


call someone up phone will call you up when we are in
town.

You are still mad. You need to


calm down relax after being angry calm down before you drive the
car.

not care for


not like (formal) I don't care for his behaviour.
someone/something

get to the same point as someone You'll have to run faster than that if
catch up
else you want to catch up with Marty.

check in arrive and register at a hotel or We will get the hotel keys when we

283
airport check in.

You have to check out of the hotel


check out leave a hotel
before 11:00 AM.

check The company checks out all new


look at carefully, investigate
someone/something out employees.

check out Check out the crazy hair on that


look at (informal)
someone/something guy!

She cheered up when she heard the


cheer up become happier
good news.

I brought you some flowers to


cheer someone up make happier
cheer you up.

If everyone chips in we can get the


chip in help
kitchen painted by noon.

Please clean up your bedroom


clean something up tidy, clean
before you go outside.

I came across these old photos


come across something find unexpectedly
when I was tidying the closet.

The top and bottom come apart if


come apart separate
you pull hard enough.

come down with My nephew came down with


become sick
something chicken pox this weekend.

volunteer for a task or to give The woman came forward with


come forward
evidence her husband's finger prints.

The art of origami comes from


come from somewhere originate in
Asia.

count on I am counting on you to make


rely on
someone/something dinner while I am out.

Please cross out your old address


cross something out draw a line through
and write your new one.

My doctor wants me to cut back


cut back on something consume less
on sweets and fatty foods.

make something fall to the We had to cut the old tree in our
cut something down
ground yard down after the storm.

284
Your father cut in while I was
cut in interrupt
dancing with your uncle.

pull in too closely in front of The bus driver got angry when that
cut in
another vehicle car cut in.

start operating (of an engine or The air conditioner cuts in when


cut in
electrical device) the temperature gets to 22°C.

The doctors cut off his leg because


cut something off remove with something sharp
it was severely injured.

The phone company cut off our


cut something off stop providing phone because we didn't pay the
bill.

My grandparents cut my father off


cut someone off take out of a will
when he remarried.

remove part of something


cut something out I cut this ad out of the newspaper.
(usually with scissors and paper)

do someone/something He's lucky to be alive. His shop was


beat up, ransack (Br.E., informal)
over done over by a street gang.

My teacher wants me to do my
do something over do again (N.Amer.) essay over because she doesn't like
my topic.

It's time to do away with all of


do away with something discard
these old tax records.

Do your coat up before you go


do something up fasten, close
outside. It's snowing!

It's a fancy restaurant so we have to


dress up wear nice clothing
dress up.

Andrea dropped back to third


drop back move back in a position/group
place when she fell off her bike.

I might drop in/by/over for tea


drop in/by/over come without an appointment
sometime this week.

take someone/something
drop someone/something I have to drop my sister off at work
somewhere and leave them/it
off before I come over.
there

I dropped out of Science because


drop out quit a class, school etc
it was too difficult.

285
I don't feel like cooking tonight.
eat out eat at a restaurant
Let's eat out.

We ended up renting a movie


end up eventually reach/do/decide
instead of going to the theatre.

My new dress fell apart in the


fall apart break into pieces
washing machine.

The picture that you hung up last


fall down fall to the ground
night fell down this morning.

The money must have fallen out of


fall out separate from an interior
my pocket.

(of hair, teeth) become loose and His hair started to fall out when he
fall out
unattached was only 35.

I need to figure out how to fit the


figure something out understand, find the answer piano and the bookshelf in this
room.

to write information in blanks Please fill in the form with your


fill something in
(Br.E.) name, address, and phone number.

to write information in blanks The form must be filled out in


fill something out
(N.Amer.) capital letters.

I always fill the water jug up when


fill something up fill to the top
it is empty.

We don't know where he lives.


find out discover
How can we find out?

We tried to keep the time of the


find something out discover party a secret, but Samantha found
it out.

get something communicate, make I tried to get my point across/over


across/over understandable to the judge but she wouldn't listen.

I was surprised how well my new


get along/on like each other girlfriend and my sister got
along/on.

My grandfather can get around


get around have mobility
fine in his new wheelchair.

We worked so hard this year that


get away go on a vacation
we had to get away for a week.

286
do without being noticed or Jason always gets away with
get away with something
punished cheating in his maths tests.

We got back from our vacation last


get back return
week.

Liz finally got her Science notes


get something back receive something you had before
back from my room-mate.

My sister got back at me for


get back at someone retaliate, take revenge stealing her shoes. She stole my
favourite hat.

become interested in something I finally got back into my novel


get back into something
again and finished it.

We're going to freeze out here if


get on something step onto a vehicle
you don't let us get on the bus.

recover from an illness, loss, I just got over the flu and now my
get over something
difficulty sister has it.

The company will have to close if it


get over something overcome a problem
can't get over the new regulations.

I don't know when I am going to


finally find time to do (N.Amer.:
get round to something get round to writing the thank you
get around to something)
cards.

Let's get together for a BBQ this


get together meet (usually for social reasons)
weekend.

I got up early today to study for my


get up get out of bed
exam.

You should get up and give the


get up stand
elderly man your seat.

reveal hidden information about His wife gave him away to the
give someone away
someone police.

My father gave me away at my


give someone away take the bride to the altar
wedding.

My little sister gave the surprise


give something away ruin a secret
party away by accident.

give something to someone for The library was giving away old
give something away
free books on Friday.

287
I have to give these skates back to
give something back return a borrowed item
Franz before his hockey game.

reluctantly stop fighting or My boyfriend didn't want to go to


give in
arguing the ballet, but he finally gave in.

give to many people (usually at They were giving out free perfume
give something out
no cost) samples at the department store.

I am giving up smoking as of
give something up quit a habit
January 1st.

My maths homework was too


give up stop trying
difficult so I gave up.

My brother tried to go after the


go after someone follow someone
thief in his car.

I went after my dream and now I


go after something try to achieve something
am a published writer.

We are going against the best


go against someone compete, oppose
soccer team in the city tonight.

Please go ahead and eat before the


go ahead start, proceed
food gets cold.

I have to go back home and get my


go back return to a place
lunch.

leave home to go on a social


go out We're going out for dinner tonight.
event

Jesse has been going out with Luke


go out with someone date
since they met last winter.

Please go over your answers before


go over something review
you submit your test.

I haven't seen Tina for a long time.


go over visit someone nearby I think I'll go over for an hour or
two.

When I was young, we went


go without something suffer lack or deprivation
without winter boots.

My best friend and I grew apart


grow apart stop being friends over time
after she changed schools.

grow back regrow My roses grew back this summer.

288
When Jack grows up he wants to
grow up become an adult
be a fireman.

Elizabeth needs a new pair of shoes


grow out of something get too big for because she has grown out of her
old ones.

This bike is too big for him now,


grow into something grow big enough to fit but he should grow into it by next
year.

give something used to someone I handed my old comic books


hand something down
else down to my little cousin.

I have to hand in my essay by


hand something in submit
Friday.

We will hand out the invitations at


hand something out to distribute to a group of people
the door.

The police asked the man to hand


hand something over give (usually unwillingly)
over his wallet and his weapons.

Hang in there. I'm sure you'll find a


hang in stay positive (N.Amer., informal)
job very soon.

Hang on while I grab my coat and


hang on wait a short time (informal)
shoes!

Instead of going to the party we are


hang out spend time relaxing (informal)
just going to hang out at my place.

He didn't say goodbye before he


hang up end a phone call
hung up.

hold someone/something I had to hold my dog back because


prevent from doing/going
back there was a cat in the park.

Jamie held back his tears at his


hold something back hide an emotion
grandfather's funeral.

Please hold on while I transfer you


hold on wait a short time
to the Sales Department.

hold onto hold firmly using your hands or Hold onto your hat because it's
someone/something arms very windy outside.

hold A man in a black mask held the


rob
someone/somethingup bank up this morning.

289
Keep on stirring until the liquid
keep on doing something continue doing
comes to a boil.

keep something from We kept our relationship from our


not tell
someone parents for two years.

keep someone/something Try to keep the wet dog out of the


stop from entering
out living room.

If you keep those results up you


keep something up continue at the same rate
will get into a great college.

I need you to be on time. Don't let


let someone down fail to support or help, disappoint
me down this time.

Can you let the cat in before you go


let someone in allow to enter
to school?

look after I have to look after my sick


take care of
someone/something grandmother.

Ever since we stole that chocolate


look down on someone think less of, consider inferior bar your dad has looked down on
me.

look for I'm looking for a red dress for the


try to find
someone/something wedding.

look forward to I'm looking forward to the


be excited about the future
something Christmas break.

We are going to look into the price


look into something investigate
of snowboards today.

be careful, vigilant, and take Look out! That car's going to hit
look out
notice you!

look out for Don't forget to look out for snakes


be especially vigilant for
someone/something on the hiking trail.

Can you look over my essay for


look something over check, examine
spelling mistakes?

search and find information in a We can look her phone number up


look something up
reference book or database on the Internet.

My little sister has always looked


look up to someone have a lot of respect for
up to me.

make something up invent, lie about something Josie made up a story about why

290
we were late.

We were angry last night, but we


make up forgive each other
made up at breakfast.

My sisters made me up for my


make someone up apply cosmetics to
graduation party.

mix something up confuse two or more things I mixed up the twins' names again!

His uncle passed away last night


pass away die
after a long illness.

It was so hot in the church that an


pass out faint
elderly lady passed out.

give the same thing to many The professor passed the textbooks
pass something out
people out before class.

I passed up the job because I am


pass something up decline (usually something good)
afraid of change.

Thanks for buying my ticket. I'll


pay someone back return owed money
pay you back on Friday.

be punished for doing something That bully will pay for being mean
pay for something
bad to my little brother.

I picked out three sweaters for you


pick something out choose
to try on.

point I'll point my boyfriend out when he


indicate with your finger
someone/something out runs by.

put what you are holding on a You can put the groceries down on
put something down
surface or floor the kitchen counter.

The students put the substitute


put someone down insult, make someone feel stupid teacher down because his pants
were too short.

We are putting off our trip until


put something off postpone
January because of the hurricane.

The neighbours put the fire out


put something out extinguish
before the firemen arrived.

I have to put the crib together


put something together assemble
before the baby arrives.

291
put up with I don't think I can put up with
tolerate
someone/something three small children in the car.

put clothing/accessories on your Don't forget to put on your new


put something on
body earrings for the party.

run into I ran into an old school-friend at


meet unexpectedly
someone/something the mall.

run over drive a vehicle over a person or I accidentally ran over your
someone/something thing bicycle in the driveway.

run over/through Let's run over/through these lines


rehearse, review
something one more time before the show.

The child ran away from home and


run away leave unexpectedly, escape
has been missing for three days.

We ran out of shampoo so I had to


run out have none left
wash my hair with soap.

My letter got sent back to me


send something back return (usually by mail)
because I used the wrong stamp.

Our boss set a meeting up with the


set something up arrange, organize
president of the company.

The police set up the car thief by


set someone up trick, trap
using a hidden camera.

I want to shop around a little


shop around compare prices
before I decide on these boots.

act extra special for people He always shows off on his


show off
watching (usually boastfully) skateboard

You should sleep over tonight if


stay somewhere for the night
sleep over the weather is too bad to drive
(informal)
home.

We need to sort the bills out before


sort something out organize, resolve a problem
the first of the month.

continue doing something, limit You will lose weight if you stick to
stick to something
yourself to one particular thing the diet.

The light's too bright. Could you


switch something off stop the energy flow, turn off
switch it off.

switch something on start the energy flow, turn on We heard the news as soon as we

292
switched on the car radio.

I take after my mother. We are


take after someone resemble a family member
both impatient.

He took the car brakes apart and


take something apart purposely break into pieces
found the problem.

I have to take our new TV back


take something back return an item
because it doesn't work.

take off start to fly My plane takes off in five minutes.

remove something (usually Take off your socks and shoes and
take something off
clothing) come in the lake!

Can you take the garbage out to the


take something out remove from a place or thing
street for me?

pay for someone to go My grandparents took us out for


take someone out
somewhere with you dinner and a movie.

I tore up my ex-boyfriend's letters


tear something up rip into pieces
and gave them back to him.

remember (often + to, sometimes When I think back on my youth, I


think back
+ on) wish I had studied harder.

I'll have to think this job offer over


think something over consider
before I make my final decision.

We threw our old furniture away


throw something away dispose of
when we won the lottery.

decrease the volume or strength Please turn the TV down while the
turn something down
(heat, light etc) guests are here.

I turned the job down because I


turn something down refuse
don't want to move.

Your mother wants you to turn the


turn something off stop the energy flow, switch off
TV off and come for dinner.

It's too dark in here. Let's turn


turn something on start the energy, switch on
some lights on.

increase the volume or strength Can you turn the music up? This is
turn something up
(heat, light etc) my favourite song.

turn up appear suddenly Our cat turned up after we put

293
posters up all over the
neighbourhood.

I'm going to try these jeans on, but


try something on sample clothing
I don't think they will fit.

I am going to try this new brand of


try something out test
detergent out.

The kids used all of the toothpaste


use something up finish the supply
up so we need to buy some more.

We have to wake up early for work


wake up stop sleeping
on Monday.

warm You can warm your feet up in


increase the temperature
someone/something up front of the fireplace.

I always warm up by doing sit-ups


warm up prepare body for exercise
before I go for a run.

Most of my make-up wore off


wear off fade away
before I got to the party.

I work out at the gym three times a


work out exercise
week.

work out be successful Our plan worked out fine.

We have to work out the total cost


work something out make a calculation
before we buy the house.

294
Word Definition Example Sentence
v. to become less active, less As I began my speech, my feelings of
Abate
intense, or less in amount nervousness quickly abated.
adj. existing purely in the
Julie had trouble understanding the appeal of
Abstract mind; not representing actual
the abstract painting.
reality
I got an abysmal grade on my research
Abysmal adj. extremely bad
paper!
Accordingly adv. in accordance with All students must behave accordingly.
n. the act of gaining a skill or Language acquisition is easier for kids than it
Acquisition
possession of something is for adults.
v. to make suit a new
The US has adapted many foreign foods to
purpose
better suit the tastes of Americans.
Adapt v. to accommodate oneself to
Dogs are known for their ability to
a new condition, setting, or
quickly adapt to their environments.
situation
adj. having knowledge or skill Beth loves playing the piano, but she’s
Adept
(usu. in a particular area) especially adept at the violin.
adj. having sufficient
Though his resume was adequate, the
Adequate qualifications to meet a
company doubted whether he’d be a good fit.
specific task or purpose
n. the arrival or creation of The world has never been the same since
Advent
something (usu. historic) the advent of the light bulb.
adj. relating to hostile An adversarial attitude will make you many
Adversarial
opposition enemies in life.
n. someone who promotes or
defends something I am an advocate for free higher education.
Advocate v. to defend or promote Environmental protesters often advocatefor
something (usu. a belief, cleaner energy practices.
theory, opinion, etc.)
The aesthetic decorations at the wedding
adj. relating to beauty or
Aesthetic reception made you feel as if you were a
refined taste
character in a fairy tale.
He’s saving money so he can afford to buy a
v. to be able to buy
Afford new car.
v. to be able to spare
I can’t afford to lose any more pencils!
v. to promote something
Agitate They’re agitating for better health care.
(usu. a cause)
Allow v. to permit or consent to US law allows citizens to speak freely.
v. to make a secretive She alluded to the problem at hand but didn’t
Allude
mention of something say anything more about it.
n. a noisy argument or Greg got into an altercation with a stranger at
Altercation
confrontation the bar.

295
adj. unclear or vague in Her ambiguous statement made me question
Ambiguous
meaning whether she could be trusted.
adj. having a powerful desire Penny is so ambitious, she wants to be
Ambitious
for success or achievement president someday.
n. the state of being uncertain
His ambivalence prevented him from
Ambivalence or stuck between two or more
immediately signing the contract.
options
Green onions are considered analogous to
Analogous adj. similar but not identical
spring onions.
v. to destroy or cause The dictator sent orders to annihilate the
Annihilate
devastating destruction group of rebels.
n. something different from This result is an anomaly and very rarely
Anomaly
the norm happens.
v. assume to be likely to The party was just as fun as I
Anticipate
happen had anticipated it would be.
Her antipathy toward the professor was
Antipathy n. a strong feeling of dislike obvious: she rolled her eyes whenever he
entered the classroom.
n. the highest point of The spring play was the apex of our school
Apex
something year.
n. fearful expectation of Her apprehension to leave her house
Apprehension
something resulted in her missing the train.
She articulated her opinion on the price of
Articulate v. to clearly express in words
the house.
adj. something made; not Many candies use artificial flavors to make
Artificial
occurring naturally them taste fruity.
His assertion that sharks are mammals made
Assertion n. a strong declaration
everyone laugh.
He lived in a small, austere cabin in the
middle of the woods.
adj. extremely plain
My boss had an austere expression on her
Austere adj. stern and forbidding
face.
adj. relating to self-denial
An austere lifestyle, like that of monks, isn’t
for everybody.
n. the quality of being real
The police officer doubted the authenticityof
Authenticity and true instead of fake and
the suspect’s story.
contrived
n. an intangible path or The company has decided to pursue
Avenue
approach to something other avenues.
adj. actively interested in or
Avid Gerald is an avid soccer fan.
enthusiastic about something
adj. relating to the foundation You have to start with basic Russian before
Basic
or basis of something you can move on to the advanced level.

296
She bears a strong resemblance to your
v. to have as a characteristic mother.
v. to have (a child) Judy will bear her first child last year.
Bear
v. to bring forth My garden is going to bear pumpkins this
v. to put up with year.
I can’t bear her complaining any longer!
Benevolent adj. kind, generous Many cultures believe in benevolent spirits.
n. a preconception that It’s important to avoid bias when investigating
Bias
prevents objectivity a crime.
adj. tinged with a feeling of The ending of the romance movie
Bittersweet
sadness was bittersweet.
v. to support, strengthen, or If we work together, we should be able to lift
Bolster
fortify and then bolster the couch.
The boost in profits was a welcome change.
n. an increase or growth
Boost In order to boost profits, you need to cater to
v. to increase or make grow
your customers.
n. an intense, loud fight A brawl broke out at school today after one
Brawl v. to fight loudly and student accused another of cheating.
disruptively The two students brawled for an hour.
n. the quality of being brief or The brevity of their time together made it all
Brevity
terse the more romantic.
Josh is candid about his desire to become an
Candid adj. direct, blunt
actor.
n. the trait of being honest I admire her candor, especially when nobody
Candor
and frank else bothers to speak up.
I’d like to capitalize on your math skills by
Capitalize v. to use to your advantage
having your work the cash register.
v. to trap or take possession The spy was captured by the enemy.
of Your painting beautifully captures the
v. to successfully represent ephemerality of life.
Capture
or imitate I was captured by her beauty.
v. to captivate, mesmerize The cops captured the criminal three days
v. to catch or seize after the incident.
adj. relating to the city or
Civic Voting is a civic duty.
citizens
adj. emotionally unattached
Her clinical approach to situations allows her
Clinical (usu. used in medical or
to handle them more effectively.
scientific setting)
n. special advantage or Children of rich and famous people often
Clout
power believe they have a lot of clout.
The horse’s mane was coarse, as if it had
adj. indicating a rough texture
never been washed.
Coarse adj. lacking refinement or
The queen’s coarse way of speaking
sophistication
surprised the other members of royalty.

297
It wasn’t until after I booked my ticket that I
Coincide v. to happen at the same time
realized the concert coincided with my finals.
n. the use of payment to
This painting was commissioned by a rich
Commission request something (e.g., a
merchant in 1589.
service or product)
This novel is comparable to Huckleberry
Comparable adj. able to be compared
Finn.
We need to hire a competent web developer
Competent adj. sufficiently qualified
to create a good website for our company.
Though he had never won any awards or
adj. satisfied, with no desire
Complacent even been published, he
to change or improve
was complacentwith his life as a poet.
v. to make perfect or This wine perfectly complements this platter
Complement
complete of gourmet cheese.
v. to be forced to agree or With no chance of winning the battle, the army
Concede surrender at last conceded.
v. to admit to a transgression Dan conceded to pranking his sister.
The plan to build the city was
Conceive v. to imagine or come up with
originally conceived in the early 1900s.
v. to overlook, approve, or She couldn't condone her daughter's
Condone
allow rebellious behavior.
adj. able to bring about or be The noisy students hardly made the campus
Conducive
suitable for library conducive to studying.
The group conducted their research abroad
v. to control or manage
Conduct last year.
v. to behave a certain way
Be sure to conduct yourself accordingly.
v. to share something She confided all of her biggest secrets in her
Confide
secretive with someone best friend.
We are going to confine the use of this
Confine v. to put limits on; to restrict
drinking fountain.
After weeks of debating, the panel finally
Consensus n. overall agreement
came to a consensus.
v. to form or compose (part The desire for equality constituted the civil
Constitute
of) something rights movement.
She contemplated telling her teacher about
Contemplate v. to think deeply about
the cheating student.
v. to maintain or assert (an The president contends that the US
Contend
opinion) government will not negotiate with terrorists.
Contradict v. to be in contrast with The camera footage contradicts his alibi.
adj. highly debatable and Millions of viewers watched
Controversial
causing contention the controversial debate take place.
adj. abiding by accepted
Conventional She lives a conventional life in the suburbs.
standards

298
v. to pass on or transfer I have trouble conveying my thoughts in
Convey
(information) French.
Her religious convictions prevent her from
Conviction n. a firm belief in something
eating meat.
The note signed by her
v. to provide evidence for; to
Corroborate father corroboratesher claim that she was
back up (a claim)
absent from class that day.
This ingredient seems to counteract the other
Counteract v. to work in opposition to
ones.
n. an argument used to Make sure to include a counterargumentin
Counterargument criticize or dismantle another your essay so that you can show you’ve
argument considered the topic from all perspectives.
Bill’s idea to take a shortcut was
adj. hindering the
Counterproductive ultimately counterproductive: it took us twice
achievement of a goal
as long to get to the train station.
The culmination of the performance was
Culmination n. the final act or climax
unforgettable.
Teachers don’t just pass on new information
Cultivate v. to foster the growth of to students—they cultivate their academic
potential.
v. to declare formally and The president decreed that Halloween would
Decree
with authority henceforth be a national holiday.
Her deference to the elderly makes her the
Deference n. respect; regard perfect candidate for an internship at the
retirement center.
adj. not enough in degree or I feel as though the sources for my paper
Deficient
amount are deficient.
Could you demonstrate the dance move for
me?
v. to do as an example
Demonstrate This book’s use of words such as “grim” and
v. gives evidence for
“bleak” demonstrates the author’s mournful
tone.
She demurred at my request to transfer to a
Demur v. to object to
different department.
v. to (over)use over time The lost campers quickly depleted their
Deplete
(usu. resources) supply of food.
Desolate adj. bare, barren, empty The moon is one giant, desolate landscape.
Devise v. to come up with (a plan) Lana devised a plan to make herself famous.
n. a problem, usually
The main dilemma is whether to pay for a
Dilemma requiring a choice between
commercial or not.
two options
n. conscientiousness; the
Diligence and confidence will get you far in
Diligence quality of being committed to
life.
a task

299
v. to become smaller in The itchiness of mosquito bites usually starts
Diminish
scope or degree to diminish after a few days.
When the police didn’t explain what was
adj. hopeless and dangerous
Dire happening right away, Jane knew that the
or fearful
situation must be dire.
Disputes over money caused
Discord n. disagreement
intense discord in the family.
n. a lack of respect and
He looked at me with such disdain that I
Disdain strong dislike (toward
immediately knew the job wouldn’t work out.
something or someone)
n. hopelessness, stress, or
To Nick’s dismay, he got an F on the test.
consternation
Dismay Many were dismayed by the town’s
v. to fill with woe or
implementation of metered parking.
apprehension
A good boss is stern but never disparageshis
Disparage v. to belittle or speak down to
or her employees.
v. to send off a message or The mother dispatched her daughter to their
Dispatch
messenger neighbor’s house.
Lately, there’s been
n. the act of becoming
Diversification noticeable diversification of students at
diverse
higher institutions.
n. a principle, theory, or
Devoutly religious people often live their lives
Doctrine position, usu. advocated by a
according to their doctrines.
religion or gov’t
n. power and authority (usu. The country claimed to have dominion over
Dominion over a territory) parts of Russia.
n. a legal territory Puerto Rico is a dominion of the US.
The gray clouds in the sky made the day
Dreary adj. sad, gloomy, dull
feel dreary.
The man’s claims to the throne
Dubious adj. doubtful, questionable were dubious since nobody knew where he’d
come from.
adj. peculiar or odd; deviating She’s a little eccentric but still fun to be
Eccentric
from the norm around.
After cheating on the exam, Emily began to
Egregious adj. extremely bad feel as though she’d made
an egregiousmistake.
adj. having refined or
His speech was not only eloquent but also
Eloquent expressive communication
extremely compelling.
skills (in speaking or writing)
Our town made news when
adj. superior or distinguished;
Eminent the eminentmagician came to perform at our
high in position or status
local theater.

300
v. to discharge, give forth, or Plants consume carbon dioxide
Emit
release and emitoxygen.
adj. very expressive; using Her emphatic smile told me she was excited
Emphatic
emphasis to ride the roller coaster.
adj. derived from experience,
You need empirical evidence to support your
Empirical observation, or an
claim.
experiment
v. to equip or bestow (usu. a According to the myth, the gods endowedhim
Endow
quality or ability) with the gift of healing.
v. to withstand, sustain, or I can’t endure this wait any longer. Will
Endure
hold out against Stanford accept or reject me?
A doctoral program entails long nights and a
Entail v. to involve or include
heavy workload.
Her face will forever be entrenched in my
Entrenched adj. firmly established
memory.
I can’t enumerate how many times I’ve had to
Enumerate v. to specify or count remind my students when their papers are
due.
n. excessive jealousy His envy of her is quite obvious.
Envy
v. to admire and be jealous of She envies her coworker's social skills.
adj. having no fixed course;
Erratic The car became erratic after slipping on ice.
deviating from the norm
They established a law that made it illegal to
v. to enact
drive after drinking any amount of alcohol.
Establish v. to found (a business,
Our group established a new branch in
group, school, etc.)
Chicago.
Evoke v. to draw forth or call up Horror movies are great at evoking fear.
v. to make worse or increase The doctor told me not to run as it
Exacerbate
the severity of can exacerbate my knee injury.
v. to do something extremely She was a well-rounded student
Excel
well or to be superior in but excelled especially in science.
v. to put into use (usu. as
Exert Don’t exert all of your energy at once.
effort)
adj. invigorating, stimulating, The music playing at the club was catchy
Exhilarating
or exciting and exhilarating.
v. to use up (as in energy or Be careful not to expend all your energy in
Expend
money) the first half of a marathon.
The shoddy company exploited its workers
Exploit v. to use selfishly or for profit
by paying them extremely low wages.
In grad school, advisors facilitate students’
Facilitate v. to aid the progress of
research and offer constructive criticism.

301
The feasibility of her project was doubtful;
n. the practicality or
Feasibility she’d have to go all the way to Antarctica and
possibility of something
back before the school year ended.
The lion is just one wild animal known for
Ferocity n. viciousness, violence
its ferocity.
adj. related to (government) Fiscal policy is how the government uses
Fiscal
money money to influence the economy.
v. to prosper, grow, or make After one year, the tiny plants
Flourish
fast progress had flourished into a breathtaking garden.
v. to be unstable; to rise and Stocks can fluctuate on a daily basis, making
Fluctuate
fall it difficult to determine when to buy or sell one.
The civilians accused their leader
Foment v. to stir up
of fomenting political unrest.
adj. capable of being I can't imagine aliens visiting us in
Foreseeable
predicted or anticipated the foreseeable future.
I frankly don’t see the point in learning to
Frankly adv. directly, clearly
drive.
His freewheeling attitude often got him in
Freewheeling adj. carefree
trouble at work.
adj. the most essential or A thesis is arguably the
Fundamental
most basic part most fundamentalpart of an essay.
adj. thrilling, exciting, The galvanizing performance left everyone
Galvanizing
stimulating spellbound.
I became interested in geriatric medicine
Geriatric adj. relating to old age shortly after my grandfather passed away
from cancer.
The voices around the corner sounded
Hostile adj. harmful, dangerous
angry, hostile even.
adj. supposed; related to a For my physics homework, I must come up
Hypothetical
hypothesis with a hypothetical situation.
adj. publicly shameful or The politician's expensive campaign ultimately
Ignominious
humiliating ended in ignominious defeat.
v. to transmit, bestow, or Parents must impart common sense to their
Impart
disclose children.
n. the equal and objective To ensure impartiality, we require everyone
Impartiality
treatment of opposing views to follow these general guidelines.
adj. impressive (esp. in size The old mansion was imposing in its huge
Imposing
or appearance) size and gothic architecture.
If it’s not too much of an imposition, could
Imposition n. an unnecessary burden
you proofread my paper?
adj. not cautious or prudent; Backpacking abroad can be fun, but don’t
Imprudent
rash be imprudent about money.
Incite v. to encourage or stir up Her hateful words incited anger in the crowd.

302
n. apathy, emotional The girl’s indifference toward her brother
Indifference
detachment upset their parents.
adv. randomly; with little or
Indiscriminately Lottery winners are chosen indiscriminately.
no distinction
v. to give into; to satisfy or My friend loves to indulge in cheesy romance
Indulge
gratify movies.
v. to guess, conclude, or You can infer from this quotation that the
Infer
derive by reasoning writer didn’t care for “pretty” language.
adj. novel or new (esp. as an Her invention was incredibly innovativeand
Innovative
idea or invention) won her multiple awards.
A vampire’s thirst for blood is said to
Insatiable adj. can’t be satisfied
be insatiable.
The culture’s norms were an inversion of our
Inversion n. a reversal
own.
v. to call on; to appeal to
Invoke The shaman attempted to invoke a demon.
(e.g., a higher power)
The couple’s differences were
adj. incapable of being in
Irreconcilable ultimately irreconcilable, giving them no
harmony or agreed upon
choice but to break up.
Susan lamented her missed chance at going
Lament v. to feel sorrow for; to mourn
to Europe with her high school class.
Locomotion n. movement Physics involves the study of locomotion.
Writing books isn’t a
adj. capable of making a lot
Lucrative particularly lucrativecareer, unless you’re J.K.
of money; profitable
Rowling.
The malicious spirit drove out the inhabitants
Malicious adj. harmful, spiteful
from their home.
adj. capable of being molded Children’s minds are malleable but only for so
Malleable
or changed long.
adj. superficial; focus on Many people accuse Americans of
Materialistic
material possessions being materialistic.
adj. extravagant or
The melodramatic play was well liked by the
Melodramatic exaggerated (as of a
audience.
melodrama)
They moved into a modest house in the
adj. simple and humble countryside.
Modest
adj. small in size or amount I received a modest sum of money for my
help at the company event.
Dr. Nguyen modified the gene so that it
Modify v. to change, alter, or tweak
wouldn’t carry the disease.
Momentous adj. historically significant Her win in the election was momentous.
We are looking for novel ways to approach
Novel adj. new, innovative
the project.

303
n. a subtle difference in Body-language experts even understand
Nuance
meaning the nuances of facial expressions.
adj. legally void and
Null The government declared their marriage null.
ineffective
n. judgment based on
In scientific research, objectivity is of utmost
Objectivity observations instead of
importance.
emotions or opinions
Historians assumed record players would
adj. no longer used; rare or
Obsolete be obsolete by now, but in fact they’re
uncommon
making a huge comeback.
Gods are omnipotent beings who can control
Omnipotent adj. almighty and all powerful
human destiny.
n. the beginning or early At the onset of her career as a lawyer, things
Onset
stages were looking up.
v. to openly express an The new employee opined at the company
Opine
opinion meeting.
adj. highly detailed and That ornate silverware must be worth
Ornate
decorated thousands of dollars!
v. to remove or force out of Sick and tired of putting up with his bad
Oust
(usu. a position or office) moods, the pirates ousted their captain.
adj. predominant, superior, Our paramount concern is the safety of our
Paramount
most important employees.
Upon entering the abandoned house, Kate
Peculiar adj. strange, bizarre experienced a peculiar feeling, as if someone
was watching her.
According to the news, nobody perished in
Perish v. to die; to pass away
the fire.
They will persecute anyone who doesn’t
Persecute v. to cause suffering to
agree with their views of the world.
Petulant children are especially difficult to
Petulant adj. cranky, pouty, irritable
care for.
Many believe that composers such as
Pinnacle n. highest level or degree Beethoven and Mozart represent
the pinnacle of classical music.
The frail-looking dog was pitiable, so I gave it
Pitiable adj. deserving pity
some food and took it inside to care for it.
adj. reasonable and possibly Her story is plausible, but that doesn’t mean
Plausible
true she’s telling the truth.
The literary critic postulates that romanticism
Postulate v. to assert
and naturalism are actually interconnected.
adj. having great influence The bald eagle is a potent symbol of the US.
Potent adj. having a strong, The potion was definitely potent—it healed
chemical effect my wounds immediately!

304
It’s not necessarily more pragmatic to study
Pragmatic adj. practical, useful
engineering than it is to study philosophy.
n. an example or subject from This change in law is without
Precedent
earlier in time historical precedent.
n. someone who comes
My predecessor gave me many tips for
Predecessor before you (usu. in position or
running the office.
office)
The directions for our essay prescribe a
v. to command orders
length of at least ten pages.
Prescribe v. to issue authorization for
A doctor must prescribe you this medication
medications
before you can begin taking it.
n. basic truth, assumption, or Remember the universal principle: treat
Principle
rule others as you want them to treat you.
v. to command against, to Alcohol was prohibited in the US in the
Prohibit
outlaw 1920s.
adj. punctual, on time She is always prompt when it comes to
n. a cue to begin something; turning in her homework.
Prompt instructions I had to write an essay based on a prompt.
v. to incite, propel, or cause The possibility of a scholarship promptedhim
to act to apply to Harvard.
v. to put into law or formally The ruler will at last promulgate an amnesty
Promulgate
declare with the neighboring countries.
v. to bring criminal action
Prosecute The suspect was prosecuted yesterday.
against someone (in a trial)
adj. intending to provoke, Her nude paintings are considered
Provocative
inspire, or arouse quite provocative.
adj. involving qualities of
I noticed a qualitative change in her
Qualitative something (features and
paintings.
content)
adj. involving quantities
Quantitative We must conduct a quantitative analysis.
(numbers and amounts)
Quirk n. a strange habit His biggest quirk is his love of old marbles.
v. to split into two or more Cars ramified throughout the world in the
Ramify
branches twentieth century.
adj. without attention to Her rash decision to pass the car nearly
Rash
danger or risk resulted in a crash.
He’s got raw talent as a singer, but he needs
adj. unrefined
to work on his performance skills.
Raw adj. not processed; uncooked
In some countries, such as Japan, it is normal
(as in food)
to eat raw fish.
adv. right away and without Water was readily available at different points
Readily
difficulty in the race.
n. thinking again about a The judges’ reconsideration of her
Reconsideration
previously made choice performance resulted in her victory.

305
The reform made it so that only those 18 and
n. a change for the better;
older can legally drive.
Reform improvement
The government reformed its vague policies
v. to improve via change
on marijuana use.
v. to prove to be untrue, The student refuted the professor’s claim in
Refute
unfounded, or incorrect class.
v. to strengthen or add We can use these pipes to reinforce the
Reinforce
support to structure.
Max reluctantly agreed to see the horror
Reluctantly adv. somewhat unwillingly
movie with his friends.
v. to give up (usu. power or a Our CEO renounced her position yesterday.
Renounce position) He renounced his friend after he caught her
v. to cast off stealing money from him.
The mother reproached her daughter’s
Reproach v. to criticize school for making students come in during a
blizzard.
v. to refuse to recognize as The father repudiated his son’s marriage.
Repudiate true She repudiated her son once she found out
v. to cast off he’d married someone without telling her.
n. the act of keeping Water retention can make you weigh more on
Retention
something certain days.
Satiated adj. satisfied (usu. in hunger) I felt satiated after eating a snack.
adj. having practical My brother is not very savvy when it comes to
Savvy
intelligence or knowledge using public transportation.
adj. morally offensive, often
The scandalous politician decided it was best
Scandalous causing damage to one’s
to resign from office.
reputation
v. to look down on with It’s difficult for me not to scorn those who use
Scorn
disdain improper grammar.
adj. paying great attention to I am a scrupulous proofreader and never
Scrupulous
detail miss an error.
v. to examine carefully and
Scrutinize The teacher scrutinized her students’ essays.
critically
v. to produce or release (a
Secrete Trees secrete a sticky substance called sap.
substance)
I am of the sentiment that you should never
n. opinion give out your passwords to anyone.
Sentiment
n. a tender or moving gesture Even though I’m not a big fan of porcelain
dolls, I appreciated the sentiment.
adj. so thin that light can The curtains on the window were so sheeryou
Sheer
shine through could clearly see inside the house.
adj. easy; not complex This math problem is so simple even a first
Simple
adj. undecorated grader can solve it.

306
The simple beauty of the ocean is what
makes it memorable.
Medieval peasants believed sinisterdemons
Sinister adj. ominous, evil
could harm humans.
n. the joining of I stood in solidarity with other female
Solidarity commonalities or common students by refusing to wear the school’s
purposes among a group sexist uniform.
adv. insufficiently, meagerly, Due to my condition, I must eat
Sparingly
or in a restricted manner salt sparingly.
Frogs typically spawn in ponds.
v. to release eggs
Spawn The topic spawned an ongoing debate
v. to call forth or generate
among his family members.
Spur v. to stimulate or incite Her bravery spurred others to act.
adj. run-down, sordid, or The squalid cabin needed a new roof and an
Squalid
sleazy exterminator.
adj. very plain; devoid of any Looking out at the stark landscape, I felt a
Stark
details or features keen sense of isolation.
The ball is static.
adj. motionless
Static Her life has been static for the past three
adj. changeless
years.
The subordinate officers work every day.
adj. lower in rank
My subordinate will check you in.
n. someone lower in rank
Subordinate You aren’t my boss—you
v. to make dependent on or
can’t subordinateme to the role of
put at a lower rank
receptionist!
adv. happening later or after
Subsequently I subsequently went home.
something
adj. very large in amount or I was shocked to find a substantial amount of
Substantial
degree money beneath the park bench.
It is important for scientists
v. to strengthen with new
Substantiate to substantiatetheir theories whenever
evidence or facts
possible.
I detected in her expression a subtle hint of
Subtle adj. hard to detect or analyze
irritation.
adj. enough; just meeting a These boxes should be sufficient for our
Sufficient
requirement move.
adj. unfriendly; inclined to The bartender was a surly fellow who wasn’t
Surly
anger afraid to start a fight.
v. to get on top of or They managed to surmount the language
Surmount
overcome barrier by using a translation app.
adj. to be vulnerable (to Children are more susceptible to certain
Susceptible
something) illnesses than adults are.

307
adj. skilled at dealing with Her tactful attitude toward our class made her
Tactful
people one of my favorite teachers.
The rubberband was taut and ready to be
Taut adj. pulled tight
fired.
adj. abundantly filled (usu. Doorknobs are not as clean as they look and
Teeming
with living organisms) are often teeming with germs.
She had a hostile temperament, making her
Temperament n. usual mood or feelings
intimidating to most people.
We haven’t made any official arrangements
Tentative adj. not yet finalized yet, but the tentative location for our wedding
is Hawaii.
adj. see-through; so thin that Stained window glass isn’t as transparentas
light can shine through regular window glass is.
Transparent
adj. truthful or easy to She was transparent about her plans to end
perceive her marriage.
The journey was becoming treacherous,but
Treacherous adj. dangerous and unstable
they continued on regardless.
adj. very large, good, or bad Tremendous news! You don’t have to repay
Tremendous
in degree or size your loans!
adj. being everywhere at
Ubiquitous Cell phones are ubiquitous these days.
once
Though the dress was cheap
Unadorned adj. undecorated, plain and unadorned, it was by far her favorite one
on the rack.
v. to weaken or subvert (usu. Parents should take care not to
Undermine
gradually or secretly) constantly undermine their children.
v. to emphasize or give This sentence seems to underscore the
Underscore
additional weight to overall meaning of the passage.
v. to move as ripples or in a Belly dancers are known for their ability to
Undulate
wavy pattern skillfully undulate their stomachs.
The unilateral decision was deemed unfair by
Unilateral adj. one-sided
the other party involved.
The court’s decision is unjust—he should not
Unjust adj. unfair; not justified
go free.
Unmitigated adj. downright, utter, total My speech was an unmitigated disaster!
adj. completely new and
The number of protestors
Unprecedented never having happened
was unprecedented.
before; historic
We plan to unveil our plans for the new
Unveil v. to make visible; to reveal
company project on Sunday.
He had the urge to tell his parents about his
n. desire or impulse
Urge acceptance to Columbia but decided against
v. to encourage or persuade
it.

308
She urged her sister to apply to Stanford.
Your selfish actions do not validate your
Validate v. to prove or declare valid
feelings for me.
n. ability to be done in a
Viability The viability of the solution is questionable.
practical or useful way
Vital adj. urgently necessary It is vital that you respond by the deadline.
My brother quickly broke his vow to never eat
Vow v. to promise
chocolate again.
Wanting to look cool in front of your friends
Warrant v. to prove to be reasonable
doesn’t warrant breaking the law.
The farmer’s annual pumpkin yield exceeded
n. production of an amount 10,000.
v. to give way to or surrender Cars turning right on red must yield to
Yield
to oncoming traffic.
v. to produce or supply Our experiment yielded many unique-looking
vegetables.

309
If you really want to get a good grade this semester, however, you
Brief Overview of must complete all assignments, attend class, and study your notes.
Use a comma with quoted words.
Punctuation "Yes," she promised. Todd replied, saying, "I will be back this
Summary: afternoon."

When speaking, we can pause or change the tone of our voices to Use a comma in a date.
indicate emphasis. When writing, we must use punctuation to indicate
these places of emphasis. This resource should help to clarify when and
October 25, 1999
how to use various marks of punctuation. Monday, October 25, 1999
Contributors:Morgan Sousa, Dana Lynn Driscoll 25 October 1999
Last Edited: 2014-04-04 11:52:42
Use a comma in a number.
When speaking, we can pause or change the tone of our voices to
indicate emphasis. When writing, we use punctuation to indicate 15,000,000
these places of emphases. This handout should help to clarify when
and how to use various marks of punctuation. 1614 High Street

Independent clause: a clause that has a subject and a verb and Use a comma in a personal title.
can stand alone; a complete sentence
Pam Smith, MD
Dependent clause: a clause that has a subject and a verb but
Mike Rose, Chief Financial Officer for Operations, reported the
cannot stand alone; an incomplete sentence
quarter's earnings.
Comma Use a comma to separate a city name from the state.
Use a comma to join 2 independent clauses by a comma and a West Lafayette, Indiana
coordinating conjunction (and, but, or, for, nor, so).
Dallas, Texas
Road construction can be inconvenient, but it is necessary.
Avoid comma splices (two independent clauses joined only by a
The new house has a large fenced backyard, so I am sure our dog comma). Instead, separate the clauses with a period, with a comma
will enjoy it. followed by a coordinating conjunction, or with a semicolon.
Use a comma after an introductory phrase, prepositional phrase, or
dependent clause.
Semicolon
Use a semicolon to join 2 independent clauses when the second
To get a good grade, you must complete all your assignments.
clause restates the first or when the two clauses are of equal
Because Dad caught the chicken pox, we canceled our vacation. emphasis.

After the wedding, the guests attended the reception. Road construction in Dallas has hindered travel around town;
streets have become covered with bulldozers, trucks, and cones.
Use a comma to separate elements in a series. Although there is no
set rule that requires a comma before the last item in a series, it Use a semicolon to join 2 independent clauses when the second
seems to be a general academic convention to include it. The clause begins with a conjunctive adverb (however, therefore,
examples below demonstrate this trend. moreover, furthermore, thus, meanwhile, nonetheless, otherwise)
or a transition (in fact, for example, that is, for instance, in
On her vacation, Lisa visited Greece, Spain, and Italy. addition, in other words, on the other hand, even so).
In their speeches, many of the candidates promised to help protect Terrorism in the United States has become a recent concern; in
the environment, bring about world peace, and end world hunger. fact, the concern for America's safety has led to an awareness of
global terrorism.
Use a comma to separate nonessential elements from a sentence.
More specifically, when a sentence includes information that is not Use a semicolon to join elements of a series when individual items
crucial to the message or intent of the sentence, enclose it in or of the series already include commas.
separate it by commas.
Recent sites of the Olympic Games include Athens, Greece; Salt
John's truck, a red Chevrolet, needs new tires. Lake City, Utah; Sydney, Australia; Nagano, Japan.
When he realized he had overslept, Matt rushed to his car and For more information on semicolons, please see the "90-Second
hurried to work. Semicolon" vidcast series on the Purdue OWL YouTube Channel.
Use a comma between coordinate adjectives (adjectives that are
equal and reversible).
The irritable, fidgety crowd waited impatiently for the rally
Colon
speeches to begin. Use a colon to join 2 independent clauses when you wish to
emphasize the second clause.
The sturdy, compact suitcase made a perfect gift.
Road construction in Dallas has hindered travel around town: parts
Use a comma after a transitional element (however, therefore,
of Main, Fifth, and West Street are closed during the construction.
nonetheless, also, otherwise, finally, instead, thus, of course, above
all, for example, in other words, as a result, on the other hand, in Use a colon after an independent clause when it is followed by a
conclusion, in addition) list, a quotation, appositive, or other idea directly related to the
independent clause.
For example, the Red Sox, Yankees, and Indians are popular
baseball teams.

310
Julie went to the store for some groceries: milk, bread, coffee, and Use quotation marks to indicate the novel, ironic, or reserved use
cheese. of a word.
In his Gettysburg Address, Abraham Lincoln urges Americans to History is stained with blood spilled in the name of "justice."
rededicate themselves to the unfinished work of the deceased
soldiers: "It is for us the living rather to be dedicated here to the Use quotation marks around the titles of short poems, song titles,
unfinished work which they who fought here have thus far so nobly short stories, magazine or newspaper articles, essays, speeches,
advanced. It is rather for us to be here dedicated to the great task chapter titles, short films, and episodes of television or radio
remaining before us — that from these honored dead we take shows.
increased devotion to that cause for which they gave the last full "Self-Reliance," by Ralph Waldo Emerson
measure of devotion — that we here highly resolve that these dead
shall not have died in vain, that this nation under God shall have a "Just Like a Woman," by Bob Dylan
new birth of freedom, and that government of the people, by the
people, for the people shall not perish from the earth." "The Smelly Car," an episode of Seinfeld

I know the perfect job for her: a politician. Do not use quotation marks in indirect or block quotations.

Use a colon at the end of a business letter greeting. Italics


To Whom It May Concern: Underlining and Italics are often used interchangeably. Before
Use a colon to separate the hour and minute(s) in a time notation. word-processing programs were widely available, writers would
underline certain words to indicate to publishers to italicize
12:00 p.m. whatever was underlined. Although the general trend has been
moving toward italicizing instead of underlining, you should
Use a colon to separate the chapter and verse in a Biblical remain consistent with your choice throughout your paper. To be
reference. safe, you could check with your teacher to find out which he/she
prefers. Italicize the titles of magazines, books, newspapers,
Matthew 1:6
academic journals, films, television shows, long poems, plays of
three or more acts, operas, musical albums, works of art, websites,
Parenthesis and individual trains, planes, or ships.
Parentheses are used to emphasize content. They place more Time
emphasis on the enclosed content than commas. Use parentheses to
set off nonessential material, such as dates, clarifying information, Romeo and Juliet by William Shakespeare
or sources, from a sentence.
The Metamorphosis of Narcissus by Salvador Dali
Muhammed Ali (1942-present), arguably the greatest athlete of all
time, claimed he would "float like a butterfly, sting like a bee." Amazon.com
Titanic
Dash
Italicize foreign words.
Dashes are used to set off or emphasize the content enclosed within
dashes or the content that follows a dash. Dashes place more Semper fi, the motto of the U.S. Marine Corps, means "always
emphasis on this content than parentheses. faithful."

Perhaps one reason why the term has been so problematic—so Italicize a word or phrase to add emphasis.
resistant to definition, and yet so transitory in those definitions—is
because of its multitude of applications. The truth is of utmost concern!

In terms of public legitimacy—that is, in terms of garnering Italicize a word when referring to that word.
support from state legislators, parents, donors, and university The word justice is often misunderstood and therefore misused.
administrators—English departments are primarily places where
advanced literacy is taught.
The U.S.S. Constitution became known as "Old Ironsides" during
Punctuation in Types of
the War of 1812—during which the cannonballs fired from the
British H.M.S. Guerriere merely bounced off the sides of the
Sentences
Constitution. Summary:
To some of you, my proposals may seem radical—even When speaking, we can pause or change the tone of our voices to
revolutionary. indicate emphasis. When writing, we must use punctuation to indicate
these places of emphasis. This resource should help to clarify when and
Use a dash to set off an appositive phrase that already includes how to use various marks of punctuation.
commas. An appositive is a word that adds explanatory or
clarifying information to the noun that precedes it. Contributors:Morgan Sousa, Dana Lynn Driscoll
Last Edited: 2010-04-17 06:00:46
The cousins—Tina, Todd, and Sam—arrived at the party together.
Learning rules for how and when to punctuate a sentence can be
Quotation Marks difficult, especially when you consider that different types of
sentences call for different types of punctuation. This handout
Use quotation marks to enclose direct quotations. Note that should help to clarify not only the types of sentences, but also what
commas and periods are placed inside the closing quotation mark, punctuation to use in what situation.
and colons and semicolons are placed outside. The placement of
question and exclamation marks depends on the situation. Punctuation in Types of Sentences
He asked, "When will you be arriving?" I answered, "Sometime Independent clause: a clause that has a subject and a verb and
after 6:30." can stand alone; a complete sentence

311
Dependent clause: a clause that has a subject and a verb but Contributors:Dana Lynn Driscoll, Allen Brizee
cannot stand alone; an incomplete sentence Last Edited: 2011-10-05 01:09:36

To punctuate a sentence, you can use and combine some of these


Simple: composed of 1 independent patterns. For more information on independent and dependent
clause. clauses plus independent and dependent markers, see our handouts
on those subjects.
No standard punctuation.
Pattern One: Simple sentence
Compound: composed of 2 or more
This pattern is an example of a simple sentence:
independent clauses.
Independent clause [ . ]
Join 2 independent clauses by a comma and a coordinating
conjunction (and, but, or, for, nor, so). Example: Doctors are concerned about the rising death rate
from asthma.
Road construction can be inconvenient, but it is necessary.
Join 2 independent clauses by a colon when you wish to emphasize Pattern Two : Compound Sentence
the second clause.
This pattern is an example of a compound sentence with a
Road construction in Dallas has hindered travel around town: parts coordinating conjunction:
of Main, Fifth, and West Street are closed during the construction.
Independent clause [ , ] coordinating conjunction independent
Join 2 independent clauses by a semicolon when the second clause clause [ . ]
restates the first or when the two clauses are of equal emphasis.
There are seven coordinating conjunctions: and, but, for, or, nor,
Road construction in Dallas has hindered travel around town; so, yet.
streets have become covered with bulldozers, trucks, and cones.
Example: Doctors are concerned about the rising death rate
from asthma,but they don't know the reasons for it.
Complex: composed of 1 or more
dependent clauses and 1 or more Pattern Three: Compound Sentence
independent clauses. This pattern is an example of a compound sentence with a
Join an introductory dependent clause with the independent clause semicolon.
by a comma. Independent clause [ ; ] independent clause [ . ]
Because road construction has hindered travel around town, many Example: Doctors are concerned about the rising death rate
people have opted to ride bicycles or walk to work. from asthma;they are unsure of its cause.
Many people have opted to ride bicycles or walk to work because
road construction has hindered travel around town. Pattern Four: Compound Sentence
This pattern is an example of a compound sentence with an
Compound-Complex: composed of 1 or independent marker.
more dependent clauses and 2 or more Independent clause [ ; ] independent marker [ , ] independent
independent clauses. clause [ . ]
Join an introductory dependent clause with an independent clause Examples of independent markers are the following: therefore,
with a comma. Separate 2 independent clauses with a comma and a moreover, thus, consequently, however, also.
coordinating conjunction (and, but, or, for, nor, so).
Example: Doctors are concerned about the rising death rate
When it is filtered, water is cleaner, and it tastes better. from asthma;therefore, they have called for more research into
its causes.
Join an introductory dependent clause with an independent clause
with a comma. Separate 2 independent clauses by a colon when Pattern Five: Complex Sentence
you wish to emphasize the second clause.
This pattern is an example of a complex sentence with a dependent
Whenever it is possible, you should filter your water: filtered water
marker.
is cleaner and tastes better.
Dependent marker dependent clause[ , ] Independent clause[ . ]
Join an introductory dependent clause with an independent clause
with a comma. Separate 2 independent clauses by a semicolon Examples of dependent markers are as follows: because, before,
when the second clause restates the first or when the two clauses since, while, although, if, until, when, after, as, as if.
are of equal emphasis.
Example: Because doctors are concerned about the rising death
When it is filtered, water is cleaner and tastes better; all things rate from asthma, they have called for more research into its
considered, it is better for you. causes.

Sentence Punctuation Pattern Six: Complex Sentence


This pattern is an example of a complex sentence with a dependent
Patterns marker.
Summary: Independent clause dependent marker dependent clause [ . ]
This handout describes eight sentence punctuation patterns with Examples of dependent markers are as follows: because, before,
examples. since, while, although, if, until, when, after, as, as if.

312
Example: Doctors are concerned about the rising death rate Some common dependent markers are: after, although, as, as
from asthmabecause it is a common, treatable illness. if, because, before, even if,even though, if, in order
to, since, though, unless, until, whatever, when, whenever,whet
Pattern Seven her, and while.

This pattern includes an independent clause with an embedded Connecting dependent and independent clauses
non-essential clause or phrase
There are two types of words that can be used as connectors at the
First part of an independent clause [ , ] non-essential clause or beginning of an independent clause: coordinating conjunctions and
phrase, rest of the independent clause [ . ] independent marker words.

A non-essential clause or phrase is one that can be removed 1. Coordinating Conjunction


without changing the meaning of the sentence or making it
The seven coordinating conjunctions used as connecting words at
ungrammatical. In other words, the non-essential clause or phrase
the beginning of an independent clause
gives additional information, but the sentence can stand alone
are and, but, for, or, nor, so, and yet. When the second
without it.
independent clause in a sentence begins with a coordinating
Example: Many doctors, including both pediatricians and conjunction, a comma is needed before the coordinating
family practice physicians, are concerned about the rising conjunction:
death rate from asthma.
Jim studied in the Sweet Shop for his chemistry quiz, but it was
hard to concentrate because of the noise.
Pattern Eight
2. Independent Marker Word
This pattern includes an independent clause with an embedded
essential clause or phrase An independent marker word is a connecting word used at the
beginning of an independent clause. These words can always begin
First part of an independent clause essential clause or a sentence that can stand alone. When the second independent
phrase rest of the independent clause [ . ] clause in a sentence has an independent marker word, a semicolon
An essential clause or phrase is one that cannot be removed is needed before the independent marker word.
without changing the overall meaning of the sentence. Jim studied in the Sweet Shop for his chemistry quiz; however, it
Example: Many doctors who are concerned about the rising was hard to concentrate because of the noise.
death rate from asthma have called for more research into its Some common independent markers
causes. are: also, consequently, furthermore, however,moreover, nevert
heless, and therefore.
Identifying Independent and Some Common Errors to Avoid
Dependent Clauses Comma Splices
Summary: A comma splice is the use of a comma between two independent
clauses. You can usually fix the error by changing the comma to a
This handout defines dependent and independent clauses and explores
how they are treated in standard usage. period and therefore making the two clauses into two separate
sentences, by changing the comma to a semicolon, or by making
Contributors:Chris Berry, Allen Brizee one clause dependent by inserting a dependent marker word in
Last Edited: 2010-04-17 06:01:23 front of it.
When you want to use commas and semicolons in sentences and Incorrect: I like this class, it is very interesting.
when you are concerned about whether a sentence is or is not a
fragment, a good way to start is to be able to recognize dependent  Correct: I like this class. It is very interesting.
and independent clauses. The definitions offered here will help you
with this.  (or) I like this class; it is very interesting.

Independent Clause  (or) I like this class, and it is very interesting.

An independent clause is a group of words that contains a subject  (or) I like this class because it is very interesting.
and verb and expresses a complete thought. An independent clause
is a sentence.  (or) Because it is very interesting, I like this class.

Jim studied in the Sweet Shop for his chemistry quiz. Fused Sentences

Dependent Clause Fused sentences happen when there are two independent clauses
not separated by any form of punctuation. This error is also known
A dependent clause is a group of words that contains a subject and as a run-on sentence. The error can sometimes be corrected by
verb but does not express a complete thought. A dependent clause adding a period, semicolon, or colon to separate the two sentences.
cannot be a sentence. Often a dependent clause is marked by
a dependent marker word. Incorrect: My professor is intelligent I've learned a lot from her.
When Jim studied in the Sweet Shop for his chemistry quiz . . .  Correct: My professor is intelligent. I've learned a lot
(What happened when he studied? The thought is incomplete.) from her.
Dependent Marker Word  (or) My professor is intelligent; I've learned a lot from
her.
A dependent marker word is a word added to the beginning of an
independent clause that makes it into a dependent clause.  (or) My professor is intelligent, and I've learned a lot
When Jim studied in the Sweet Shop for his chemistry quiz, it was from her.
very noisy.

313
 (or) My professor is intelligent; moreover, I've learned a Summary:
lot from her.
This resource offers a number of pages about comma use.
Sentence Fragments Contributors:Dana Driscoll, Allen Brizee
Sentence fragments happen by treating a dependent clause or other Last Edited: 2010-04-17 06:02:09
incomplete thought as a complete sentence. You can usually fix The comma is a valuable, useful punctuation device because it
this error by combining it with another sentence to make a separates the structural elements of sentences into manageable
complete thought or by removing the dependent marker. segments. The rules provided here are those found in traditional
Incorrect: Because I forgot the exam was today. handbooks; however, in certain rhetorical contexts and for specific
purposes, these rules may be broken.
 Correct: Because I forgot the exam was today, I didn't
The following is a short guide to get you started using commas.
study.
This resource also includes sections with more detailed rules and
 (or) I forgot the exam was today. examples.

Quick Guide to Commas


Run-ons - Comma Splices - 1. Use commas to separate independent clauses when they are
Fused Sentences joined by any of these seven coordinating conjunctions: and, but,
for, or, nor, so, yet.
Summary:
2. Use commas after introductory a) clauses, b) phrases, or c)
This handout defines dependent and independent clauses and explores words that come before the main clause.
how they are treated in standard usage.
3. Use a pair of commas in the middle of a sentence to set off
Contributors:Chris Berry, Allen Brizee clauses, phrases, and words that are not essential to the meaning of
Last Edited: 2013-10-07 02:10:25 the sentence. Use one comma before to indicate the beginning of
the pause and one at the end to indicate the end of the pause.
Run-ons, comma splices, and fused sentences are all names given
to compound sentences that are not punctuated correctly. The best 4. Do not use commas to set off essential elements of the sentence,
way to avoid such errors is to punctuate compound sentences such as clauses beginning with that (relative clauses). That clauses
correctly by using one or the other of these rules. after nouns are always essential. That clauses following a verb
expressing mental action are always essential.
1. Join the two independent clauses with one of the coordinating
conjunctions (and, but, for, or, nor, so, yet), and use a comma 5. Use commas to separate three or more words, phrases, or clauses
before the connecting word. written in a series.
_________________________, and 6. Use commas to separate two or more coordinate adjectives that
_________________________. describe the same noun. Be sure never to add an extra comma
between the final adjective and the noun itself or to use commas
He enjoys walking through the country, and he often goes with non-coordinate adjectives.
backpacking on his vacations.
7. Use a comma near the end of a sentence to separate contrasted
2. When you do not have a connecting word (or when you use a coordinate elements or to indicate a distinct pause or shift.
connecting word other than and, but, for, or nor, so, or yet between
the two independent clauses) use a semicolon (;). 8. Use commas to set off phrases at the end of the sentence that
refer back to the beginning or middle of the sentence. Such phrases
__________________________;___________________________ are free modifiers that can be placed anywhere in the sentence
__. without causing confusion.
He often watched TV when there were only reruns; she preferred to 9. Use commas to set off all geographical names, items in dates
read instead. (except the month and day), addresses (except the street number
or and name), and titles in names.

__________________________; 10. Use a comma to shift between the main discourse and a
however,____________________. quotation.

He often watched TV when there were only reruns; however, she 11. Use commas wherever necessary to prevent possible confusion
preferred to read instead. or misreading.

So, run-ons and fused sentences are terms describing two


independent clauses which are joined together with no connecting Extended Rules for Using
word or punctuation to separate the clauses.
INCORRECT: They weren't dangerous criminals they were
Commas
detectives in disguise. Summary:

CORRECT: They weren't dangerous criminals; they were This resource offers a number of pages about comma use.
detectives in disguise.
Contributors:Dana Driscoll, Allen Brizee
INCORRECT: I didn't know which job I wanted I was too Last Edited: 2013-04-03 10:39:33
confused to decide.
Comma Use
CORRECT: I didn't know which job I wanted, and I was too
confused to decide. 1. Use commas to separate independent clauses when they are
joined by any of these seven coordinating conjunctions: and,
Commas: Quick Rules but, for, or, nor, so, yet.

314
The game was over, but the crowd refused to leave. Word: I appreciate your hard work. In this case, however, you
seem to have over-exerted yourself.
The student explained her question, yet the instructor still didn't
seem to understand. 4. Do not use commas to set off essential elements of the
sentence, such as clauses beginning with that (relative
Yesterday was her brother's birthday, so she took him out to dinner. clauses). That clauses after nouns are always
2. Use commas after introductory a) clauses, b) phrases, or c) essential.That clauses following a verb expressing mental action
words that come before the main clause. are always essential.

a. Common starter words for introductory clauses that should be That clauses after nouns:
followed by a comma includeafter, although, as, because, if, since, The book that I borrowed from you is excellent.
when, while.
The apples that fell out of the basket are bruised.
While I was eating, the cat scratched at the door.
That clauses following a verb expressing mental action:
Because her alarm clock was broken, she was late for class.
She believes that she will be able to earn an A.
If you are ill, you ought to see a doctor.
He is dreaming that he can fly.
When the snow stops falling, we'll shovel the driveway.
I contend that it was wrong to mislead her.
However, don't put a comma after the main clause when a
dependent (subordinate) clause follows it (except for cases of They wished that warm weather would finally arrive.
extreme contrast).
Examples of other essential elements (no commas):
INCORRECT: The cat scratched at the door, while I was eating.
Students who cheat only harm themselves.
CORRECT: She was still quite upset, although she had won the
Oscar. (This comma use is correct because it is an example of The baby wearing a yellow jumpsuit is my niece.
extreme contrast.) The candidate who had the least money lost the election.
b. Common introductory phrases that should be followed by a Examples of nonessential elements (set off by commas):
comma include participial and infinitive phrases, absolute phrases,
nonessential appositive phrases, and long prepositional phrases Fred, who often cheats, is just harming himself.
(over four words).
My niece, wearing a yellow jumpsuit, is playing in the living
Having finished the test, he left the room. room.
To get a seat, you'd better come early. The Green party candidate, who had the least money, lost the
election.
After the test but before lunch, I went jogging.
Apples, which are my favorite fruit, are the main ingredient in this
The sun radiating intense heat, we sought shelter in the cafe. recipe.
c. Common introductory words that should be followed by a Professor Benson, grinning from ear to ear, announced that the
comma include yes, however, well. exam would be tomorrow.
Well, perhaps he meant no harm. Tom, the captain of the team, was injured in the game.
Yes, the package should arrive tomorrow morning. It is up to you, Jane, to finish.
However, you may not be satisfied with the results. She was, however, too tired to make the trip.
3. Use a pair of commas in the middle of a sentence to set off Two hundred dollars, I think, is sufficient.
clauses, phrases, and words that are not essential to the
meaning of the sentence. Use one comma before to indicate the 5. Use commas to separate three or more words, phrases, or
beginning of the pause and one at the end to indicate the end of clauses written in a series.
the pause.
The Constitution establishes the legislative, executive, and judicial
Here are some clues to help you decide whether the sentence branches of government.
element is essential:
The candidate promised to lower taxes, protect the
 If you leave out the clause, phrase, or word, does the environment, reduce crime, and end unemployment.
sentence still make sense?
The prosecutor argued that the defendant, who was at the scene of
 Does the clause, phrase, or word interrupt the flow of the crime, who had a strong revenge motive, and who had access to
words in the original sentence? the murder weapon, was guilty of homicide.

 If you move the element to a different position in the 6. Use commas to separate two or more coordinate adjectives
sentence, does the sentence still make sense? that describe the same noun. Be sure never to add an extra
comma between the final adjective and the noun itself or to use
If you answer "yes" to one or more of these questions, then the commas with non-coordinate adjectives.
element in question is nonessential and should be set off with
commas. Here are some example sentences with nonessential Coordinate adjectives are adjectives with equal ("co"-ordinate)
elements: status in describing the noun; neither adjective is subordinate to the
other. You can decide if two adjectives in a row are coordinate by
Clause: That Tuesday, which happens to be my birthday, is the asking the following questions:
only day when I am available to meet.
 Does the sentence make sense if the adjectives are
Phrase: This restaurant has an exciting atmosphere. The food, on written in reverse order?
the other hand, is rather bland.

315
 Does the sentence make sense if the adjectives are 11. Use commas wherever necessary to prevent possible
written with and between them? confusion or misreading.

If you answer yes to these questions, then the adjectives are To George, Harrison had been a sort of idol.
coordinate and should be separated by a comma. Here are some
examples of coordinate and non-coordinate adjectives: Comma abuse
He was a difficult, stubborn child. (coordinate) Commas in the wrong places can break a sentence into illogical
segments or confuse readers with unnecessary and unexpected
They lived in a white frame house. (non-coordinate) pauses.
She often wore a gray wool shawl. (non-coordinate) 12. Don't use a comma to separate the subject from the verb.
Your cousin has an easy, happy smile. (coordinate) INCORRECT: An eighteen-year old in California, is now
The 1) relentless, 2) powerful 3) summer sun beat down on them. considered an adult.
(1-2 are coordinate; 2-3 are non-coordinate.) INCORRECT: The most important attribute of a ball player, is
The 1) relentless, 2) powerful, 3) oppressive sun beat down on quick reflex actions.
them. (Both 1-2 and 2-3 are coordinate.) 13. Don't put a comma between the two verbs or verb phrases
7. Use a comma near the end of a sentence to separate in a compound predicate.
contrasted coordinate elements or to indicate a distinct pause INCORRECT: We laid out our music and snacks, and began to
or shift. study.
He was merely ignorant, not stupid. INCORRECT: I turned the corner, and ran smack into a patrol
The chimpanzee seemed reflective, almost human. car.

You're one of the senator's close friends, aren't you? 14. Don't put a comma between the two nouns, noun phrases,
or noun clauses in a compound subject or compound object.
The speaker seemed innocent, even gullible.
INCORRECT (compound subject): The music teacher from your
8. Use commas to set off phrases at the end of the sentence that high school, and the football coach from mine are married.
refer to the beginning or middle of the sentence. Such phrases
are free modifiers that can be placed anywhere in the sentence INCORRECT (compound object): Jeff told me that the job was
without causing confusion. (If the placement of the modifier still available, and that the manager wanted to interview me.
causes confusion, then it is not "free" and must remain 15. Don't put a comma after the main clause when a dependent
"bound" to the word it modifies.) (subordinate) clause follows it (except for cases of extreme
contrast).

Nancy waved enthusiastically at the docking ship, laughing INCORRECT: The cat scratched at the door, while I was eating.
joyously. (correct) CORRECT: She was still quite upset, although she had won the
INCORRECT:Lisa waved at Nancy, laughing joyously. (Who is Oscar. (This comma use is correct because it is an example of
laughing, Lisa or Nancy?) extreme contrast)

Laughing joyously, Lisa waved at Nancy. (correct)


Commas After Introductions
Lisa waved at Nancy, who was laughing joyously. (correct)
Summary:
This resource offers a number of pages about comma use.
9. Use commas to set off all geographical names, items in dates
(except the month and day), addresses (except the street Contributors:Dana Driscoll, Allen Brizee
number and name), and titles in names. Last Edited: 2013-01-04 12:37:54

Introductory Clauses
Birmingham, Alabama, gets its name from Birmingham, England. Introductory clauses are dependent clauses that provide
July 22, 1959, was a momentous day in his life. Who lives at 1600 background information or "set the stage" for the main part of the
Pennsylvania Avenue, Washington, DC? sentence, the independent clause. For example:
If they want to win, athletes must exercise every day.
Rachel B. Lake, MD, will be the principal speaker.
(introductory dependent clause, main clause)
(When you use just the month and the year, no comma is necessary
after the month or year: "The average temperatures for July 1998 Because he kept barking insistently, we threw the ball for Smokey.
are the highest on record for that month.")
(introductory dependent clause, main clause)
Introductory clauses start with adverbs likeafter, although, as,
10. Use a comma to shift between the main discourse and a because, before, if, since, though, until, when, etc.
quotation.
John said without emotion, "I'll see you tomorrow." Introductory phrases
"I was able," she answered, "to complete the assignment." Introductory phrases also set the stage for the main action of the
sentence, but they are not complete clauses. Phrases don't have
In 1848, Marx wrote, "Workers of the world, unite!" both a subject and a verb that are separate from the subject and
verb in the main clause of the sentence. Common introductory

316
phrases includeprepositional phrases, appositive phrases, Extracting the most profit for the least expenditure on labor and
participial phrases, infinitive phrases, andabsolute phrases. materials[x] is the primary goal of a capitalist.
To stay in shape for competition, athletes must exercise every day.
(introductory infinitive phrase, main clause)
Commas vs. Semicolons in
Barking insistently, Smokey got us to throw his ball for him. Compound Sentences
(introductory participial phrase, main clause) Summary:

A popular and well respected mayor, Bailey was the clear favorite This resource offers a number of pages about comma use.
in the campaign for governor.
Contributors:Dana Driscoll, Allen Brizee
(introductory appositive phrase, main clause) Last Edited: 2012-06-04 01:13:57

The wind blowing violently, the townspeople began to seek shelter. For more information on semicolons, please see the "90-Second
Semicolon" vidcast series on the Purdue OWL YouTube Channel.
(introductory absolute phrase, main clause)
A group of words containing a subject and a verb and expressing a
After the adjustment for inflation, real wages have decreased while complete thought is called a sentence or an independent clause.
corporate profits have grown. Sometimes, an independent clause stands alone as a sentence, and
sometimes two independent clauses are linked together into what is
(introductory prepositional phrases, main clause) called a compound sentence. Depending on the circumstances, one
of two different punctuation marks can be used between the
Introductory words independent clauses in a compound sentence: a comma or a
semicolon. The choice is yours.
Introductory words like however, still,
furthermore, and meanwhile create continuity from one sentence to
the next. Comma (,)
The coaches reviewed the game strategy. Meanwhile, the athletes Use a comma after the first independent clause when you link two
trained on the Nautilus equipment. independent clauses with one of the following coordinating
conjunctions: and, but, for, or, nor, so, yet. For example:
Most of the evidence seemed convincing. Still, the credibility of
some witnesses was in question. I am going home, and I intend to stay there.
It rained heavily during the afternoon, but we managed to have our
When to use a comma picnic anyway.
Introductory elements often require a comma, but not always. Use They couldn't make it to the summit and back before dark, so they
a comma in the following cases: decided to camp for the night.
 After an introductory clause.
Semicolon (;)
 After a long introductory prepositional phrase or more
Use a semicolon when you link two independent clauses with no
than one introductory prepositional phrase.
connecting words. For example:
 After introductory verbal phrases, some appositive I am going home; I intend to stay there.
phrases, or absolute phrases.
It rained heavily during the afternoon; we managed to have our
 If there is a distinct pause. picnic anyway.
 To avoid confusion. They couldn't make it to the summit and back before dark; they
decided to camp for the night.
When not to use a comma You can also use a semicolon when you join two independent
Some introductory elements don't require a comma, and sometimes clauses together with one of the following conjunctive adverbs
the subject of a sentence looks like an introductory element but (adverbs that join independent clauses): however, moreover,
isn't. Do not use a comma in the following cases: therefore, consequently, otherwise, nevertheless, thus, etc. For
example:
 After a brief prepositional phrase. (Is it a single phrase of
I am going home; moreover, I intend to stay there.
fewer than five words?)
It rained heavily during the afternoon; however, we managed to
 After a restrictive (essential) appositive phrase. (See our have our picnic anyway.
document on appositives.)
They couldn't make it to the summit and back before dark;
 To separate the subject from the predicate. (See below.) therefore, they decided to camp for the night.
Each of the following sentences may look like it requires a comma For more information about compound sentence patterns, see the
after the opening segment (marked with an x), but the opening Purdue OWL handout on Sentence Punctuation Patterns.
segment is really the subject. It's sometimes easy to confuse
gerund- or infinitive-phrase subjects like the following with
nonessential introductory phrases, so be careful. Commas with Nonessential
Preparing and submitting his report to the committee for evaluation
and possible publication[x] was one of the most difficult tasks Bill
Elements
had ever attempted. Summary:
To start a new business without doing market research and long- This resource offers a number of pages about comma use.
term planning in advance[x] would be foolish.

317
Contributors:Dana Driscoll, Allen Brizee 2. Use a hyphen with compound numbers:
Last Edited: 2011-05-10 04:18:44
forty-six
Some modifying elements of a sentence are essential, restricting sixty-three
the meaning of a modified term, while others are nonessential and Our much-loved teacher was sixty-three years old.
don't restrict the modified term's meaning. These nonessential
elements, which can be words, phrases, or clauses, are set off with 3. Use a hyphen to avoid confusion or an awkward
commas. combination of letters:

Rule: Use commas before and after nonessential words, phrases, re-sign a petition (vs. resign from a job)
and clauses, that is, elements embedded in the sentence that semi-independent (but semiconscious)
interrupt it without changing the essential meaning. shell-like (but childlike)

If you leave out the element or put it somewhere else in the 4. Use a hyphen with the prefixes ex- (meaning former),
sentence, does the essential meaning of the sentence change? If self-, all-; with the suffix -elect; between a prefix and a
so, the element is essential; if not, it is nonessential. capitalized word; and with figures or letters:

Nonessential: The average world temperature, however, has ex-husband


continued to rise significantly. (word) self-assured
mid-September
Essential: The sixth-century philosopher Boethius was arrested, all-inclusive
tortured, and bludgeoned to death. (word) mayor-elect
anti-American
Nonessential: Company managers, seeking higher profits, hired T-shirt
temporary workers to replace full-time staff. (phrase) pre-Civil War
Essential: The person checking tickets at the counter asked for a mid-1980s
form of identification. (phrase) 5. Use a hyphen to divide words at the end of a line if
Nonessential: My uncle, who is eighty years old, walks three miles necessary, and make the break only between syllables:
every day. (clause) pref-er-ence
Essential: The woman who interviewed you is my sister. (clause) sell-ing
in-di-vid-u-al-ist
Deciding whether an element is essential or nonessential can
sometimes be tricky. For help identifying two common types of 6. For line breaks, divide already-hyphenated words only at
phrases that can be either essential or nonessential, see the OWL the hyphen:
handouts on verbals, which includes information on participial mass-
phrases, as well as the handout on appositives, which produced
covers appositive phrases. Both of these documents address the self-
essential/nonessential distinction for these kinds of phrases. conscious
You can try three different interactive exercises that allow you to 7. For line breaks in words ending in -ing, if a single final
practice these rules, each with its own answer key. consonant in the root word is doubled before the suffix,
hyphenate between the consonants; otherwise, hyphenate
Hyphen Use at the suffix itself:
plan-ning
Summary:
run-ning
A comprehensive rundown on the proper use of the hyphen. driv-ing
call-ing
Contributors:Sean M. Conrey, Karl Stolley
Last Edited: 2013-06-07 09:00:40 8. Never put the first or last letter of a word at the end or
beginning of a line, and don't put two-letter suffixes at
Two words brought together as a compound may be written the beginning of a new line:
separately, written as one word, or connected by hyphens. For
example, three modern dictionaries all have the same listings for lovely (Do not separate in a way which
the following compounds: leaves ly beginning a new line.)
eval-u-ate (Separate only on either side of the u; do not
hair stylist leave the initial e- at the end of a line.)
hairsplitter
hair-raiser
Another modern dictionary, however, lists hairstylist, not hair
How to Use Quotation
stylist. Compounding is obviously in a state of flux, and authorities
do not always agree in all cases, but the uses of the hyphen offered
Marks
here are generally agreed upon. Summary:
1. Use a hyphen to join two or more words serving as a A rundown of the general rules of when and where to use quotation
single adjective before a noun: marks.
a one-way street Contributors:Sean M. Conrey, Mark Pepper, Allen Brizee
chocolate-covered peanuts Last Edited: 2013-04-18 08:14:53
well-known author
However, when compound modifiers come after a noun, Using Quotation Marks
they are not hyphenated:
The primary function of quotation marks is to set off and represent
The peanuts were chocolate covered. exact language (either spoken or written) that has come from
The author was well known. somebody else. The quotation mark is also used to designate

318
speech acts in fiction and sometimes poetry. Since you will most Martin Luther King Jr. believed that the end of slavery was
often use them when working with outside sources, successful use important and of great hope to millions of slaves done horribly
of quotation marks is a practical defense against accidental wrong.
plagiarism and an excellent practice in academic honesty. The
following rules of quotation mark use are the standard in the The above should never stand in for:
United States, although it may be of interest that usage rules for Martin Luther King Jr. said of the Emancipation Proclamation,
this punctuation do vary in other countries. "This momentous decree came as a great beacon light of hope to
The following covers the basic use of quotation marks. For details millions of Negro slaves who had been seared in the flames of
and exceptions consult the separate sections of this guide. withering injustice."
Use an indirect quotation (or paraphrase) when you merely need to
Direct Quotations summarize key incidents or details of the text.
Direct quotations involve incorporating another person's exact Use direct quotations when the author you are quoting has coined a
words into your own writing. term unique to her or his research and relevant within your own
paper.
1. Quotation marks always come in pairs. Do not open a quotation
and fail to close it at the end of the quoted material. When to use direct quotes versus indirect quotes is ultimately a
choice you'll learn a feeling for with experience. However, always
2. Capitalize the first letter of a direct quote when the quoted try to have a sense for why you've chosen your quote. In other
material is a complete sentence. words, never put quotes in your paper simply because your teacher
Mr. Johnson, who was working in his field that morning, said, "The says, "You must use quotes."
alien spaceship appeared right before my own two eyes."
3. Do not use a capital letter when the quoted material is a fragment
or only a piece of the original material's complete sentence.
Although Mr. Johnson has seen odd happenings on the farm, he
stated that the spaceship "certainly takes the cake" when it comes
to unexplainable activity.
4. If a direct quotation is interrupted mid-sentence, do not
capitalize the second part of the quotation.
"I didn't see an actual alien being," Mr. Johnson said, "but I sure
wish I had."
5. In all the examples above, note how the period or comma
punctuation always comes before the final quotation mark. It is
important to realize also that when you are using MLA or some
other form of documentation, this punctuation rule may change.
When quoting text with a spelling or grammar error, you should
transcribe the error exactly in your own text. However, also insert
the term sic in italics directly after the mistake, and enclose it in
brackets. Sic is from the Latin, and translates to "thus," "so," or
"just as that." The word tells the reader that your quote is an exact
reproduction of what you found, and the error is not your own.
Mr. Johnson says of the experience, "It's made me reconsider the
existence of extraterestials [sic]."
6. Quotations are most effective if you use them sparingly and keep
them relatively short. Too many quotations in a research paper will
get you accused of not producing original thought or material (they
may also bore a reader who wants to know primarily what YOU
have to say on the subject).

Indirect Quotations
Indirect quotations are not exact wordings but rather rephrasings or
summaries of another person's words. In this case, it is not
necessary to use quotation marks. However, indirect quotations still
require proper citations, and you will be commiting plagiarism if
you fail to do so.
Mr. Johnson, a local farmer, reported last night that he saw an alien
spaceship on his own property.
Many writers struggle with when to use direct quotations versus
indirect quotations. Use the following tips to guide you in your
choice.
Use direct quotations when the source material uses language that
is particularly striking or notable. Do not rob such language of its
power by altering it.

319
Punctuation for Connecting Words GR6.32

Learning Centre

PUNCTUATION for CONNECTING WORDS


(Explanations, Exercises, and Answers)

A connecting word helps you clarify to a reader how one idea logically links to the next
idea. However, using these words with the correct meanings is only half of the battle.
The other half is learning to punctuate them correctly.

While this handout does not provide a complete explanation of punctuation rules, it
provides basic guidelines that should help you avoid making most of these sentence-
level punctuation errors. It is important to know that punctuation rules can sometimes
be broken. However, to be safe, it is better for you to use these guidelines until you
become very familiar with punctuation possibilities. If you would like just a quick
reference for these rules, ask for the Learning Centre handout entitled Quick
Reference for Punctuating Connecting Words.

To improve your punctuation of connecting words, you need to know


-how to locate clauses in your writing
-how to punctuate four (4) different types of connecting words

What are Clauses?


A clause is like a simple sentence. It expresses a complete idea. A clause must have
both a subject and a complete verb. Here are some examples of clauses:

the student walked across the concourse

Jessica has been playing the piano for an hour

the table was set for dinner

the baby was crying all night

Discuss with your tutor how subjects and verbs can be located. For more practice identifying
subjects and verbs, work on the Learning Centre handout Subject/Verb Agreement.

C. Klassen & J. Robinson/Revised Winter 2009


1

320
Punctuation for Connecting Words GR6.32
Notice in the next set of examples that you can add connecting words to these ideas,
and they are still clauses because they have subjects and complete verbs; however,
they cannot stand on their own any longer as completed thoughts because they need
another clause to complete the thought.

when the student walked across the concourse


if Jessica has been playing the piano for an hour
because the table was set for dinner
even though the baby was crying all night

The following examples are not clauses. Can you see why not? Discuss with your tutor
how the following phrases are different from the clauses in the two sets of examples on
the previous page.

walking across the concourse


Jessica at the piano for an hour
a set table
the baby crying all night

Below you will find explanations, examples and exercises for four (4) different types of
connecting words

1) Co-ordinating Conjunctions

The most common co-ordinating conjunctions are “and, or, nor, but, yet, so.”

Clause Co-ordinator Clause


Chris decided to transfer to UBC , and John decided to apply at SFU.

You will notice in the examples on the next page that you do not always need to
include a comma before some co-ordinating conjunctions. With the connectors “and”
and “or”, if clauses are short and closely related (such as cause and effect), then
commas are not generally used. The best way to decide if a comma is needed with
“and” and “or” is to decide if there is a clear pause when you read the sentence out
loud. However, the other connectors “but, yet, and so” should almost always have
commas before them if they separate two clauses. *

*
It is important to know that “and” and “or” are also frequently used to join words and phrases in
lists that are not complete clauses. The punctuation rules for making lists are different than for joining
clauses. If you would like to learn rules for punctuating lists, ask a tutor for the Commas handout.

C. Klassen & J. Robinson/Revised Winter 2009


2

321
Punctuation for Connecting Words GR6.32

Co-ordinators Examples
and She loved that movie. It was exciting, and her favorite actor
had the lead role.
Sometimes the demand goes up and the price does not fall.

or You could take the children to the park, or you could take
them to a movie.
We need to go now or we will be late for class.

but James decided go to a movie, but Kim stayed home to


study.

so He made a lot of mistakes, so he had to do the assignment


again.

yet Carl usually gets at least eight hours of sleep most nights,
yet he’s always tired.

Exercise 1:

Combine the following groups of sentences into longer sentences by using “and, or,
but, so, yet;” Use correct punctuation.

1) Carlos lives in Canada


He was born in Peru

2) The car ran out of gas


Bob had to walk two kilometers to find a gas station

3) Maria could go to art school in New York


She could stay in Vancouver to look for a job

4) Penelope likes to drive fast


She got too many speeding tickets
She lost her driver’s license

C. Klassen & J. Robinson/Revised Winter 2009


3

322
Punctuation for Connecting Words GR6.32
Add correct punctuation (periods and commas) to the following paragraph:

Exercise 2:
Mark loves to travel but Amy doesn’t so they often argue about where to spend
their vacation time one summer they decided to try something different Mark went to
Thailand but Amy stayed home Mark’s brother, Alex, and sister-in-law, Jennifer, went
with him they knew that they could only spend one week in Thailand so they had to
decide what to do they could all go scuba diving in Southern Thailand or Mark could
go by himself on an elephant trek in Northern Thailand and Alex and Jennifer could
meet him later in Bangkok Mark chose the elephant trek and he had an exciting time
Amy was also having a good time back home sometimes she went out with her
friends at night and sometimes her mother would visit her during the day she also
liked to spend time by herself practicing her piano Mark and Amy learned that they
could spent their vacations separately yet they both could have a good time

2) Transitions

Notice in the examples below that the transition word “therefore” can have either a
semi-colon or a period before it, but that it also has a comma after it. Transition
words almost always have punctuation both before and after them.

Clause Transition Clause


Governments need money to ; therefore, they require us to pay taxes.
operate
Governments need money to . Therefore, they require us to pay taxes.
operate

Because so many transition words and phrases exist, only frequently used transition
words and phrases are listed on the next page:

C. Klassen & J. Robinson/Revised Winter 2009


4

323
Punctuation for Connecting Words GR6.32

Transitions Examples
for example Dogs are very useful animals; for instance, they
for instance can help the police find suspects.
to illustrate
also Students can learn to study more effectively by
in addition following realistic study schedules. Also, they
besides can improve their textbook reading and note
taking skills.

that is Universities require transcripts; that is, they want


in other words records of your grades from previous schools you
in fact went to.
actually Caffeine does not always wake you up; in fact, it
can slow your responses down.

similarly My brother has a great sense of humour; in the


in the same way same way, my father loves a good joke.

in contrast Vancouver winters are usually rainy and dark; in


however contrast, Edmonton winters tend to be cold and
on the other hand bright.
instead
otherwise
nevertheless Mike knows he needs more sleep; nevertheless,
even so he stays up late every night playing computer
admittedly games.

as a result The drunk driver crashed into a police car; as a


therefore result, he spent the night in jail and lost his
consequently driver’s license.
thus
previously First, he phoned his lawyer; later, he reported the
next accident to police.
later
after that
first, second,
since then
finally

C. Klassen & J. Robinson/Revised Winter 2009


5

324
Punctuation for Connecting Words GR6.32
Exercise 3:

Use transitions from the list above to combine the following groups of ideas. Use
correct punctuation.

1) Marjorie is turning fifty this weekend.


She says that she feels like a twenty-one year old.

2) Jimmy and Kevin spend most of their time playing hockey.


They are excellent players.

3) You could take a bus.


My father could drive you.

4) The garden tools should not be left outside.


They will rust.

Some, but not all, transitions can be moved to other places in a sentence; however,
the punctuation rules change when you move the transition. Compare the following
examples:

Vancouver winters are usually rainy and dark; however, Edmonton winters tend
to be cold and bright.

Vancouver winters are usually rainy and dark; Edmonton winters, however,
tend to be cold and bright.

The rule is simple. If you place the transition somewhere in the middle of a clause
rather than at the beginning of the clause, you still need to include punctuation on both
sides of it (a comma in front of it and after it).

C. Klassen & J. Robinson/Revised Winter 2009


6

325
Punctuation for Connecting Words GR6.32
Exercise 4:

Rewrite the following sentences by moving each transition several words later in the
second clause to a place where the transition still sounds good. Make sure you change
the punctuation as well. Check your changes with a tutor to make sure your revision
works.

1) The economy is strong. Therefore, interest rates tend to be high.

2) Everybody at the party saw the stranger. However, no one asked who he was.

3) You could finish your degree next year. On the other hand, you could choose
to work instead.

4) Agriculture contributes to greenhouse gases. For example, farm animals and


rice fields add a great deal of methane gas to the atmosphere.

3) Subordinators (Adverb Clauses)

There are two important differences between subordinators (below) and most other
connecting words (above):

1) when you put a subordinator between two clauses to connect them, you
should not use any punctuation.
2) when you put the subordinator at the beginning of a clause to introduce the
main clause, you need to put a comma between the two clauses.

Clause Subordinator Clause


He did his homework before he turned on the TV.
Before he turned on the TV, he did his homework.

C. Klassen & J. Robinson/Revised Winter 2009


7

326
Punctuation for Connecting Words GR6.32
Because there are many subordinators in English, only frequently used subordinating
words are listed below:

Subordinator Examples
while While Douglas offers only the first two years of
whereas university courses, Kwantlen offers complete
four year programs.

as Just as organisms compete for resources in an


just as ecosystem, businesses compete for consumers
in a market.

before She was listening to the radio when the phone


after rang.
when
whenever
while Whenever the fire alarm sounds, students must
until leave their classes to stand outside of the
since building.
as
as soon as
because She’s taking psychology courses because she’s
since curious about human behaviour.
as
though Even though Cathy works hard in that course,
although she still isn’t getting good grades.
even though
so+adjective+that She was so tired that she went to bed at 8
such+noun+that o’clock.
He was such a good doctor that she told her
friends about him.

if If you need help with one of your courses, you


only if can make an appointment with a tutor in the
even if Learning Centre.
as long as
unless We’ll have a picnic at Queens Park unless it
when rains.
whenever

C. Klassen & J. Robinson/Revised Winter 2009


8

327
Punctuation for Connecting Words GR6.32
Exercise 5:

Join the following ideas using the suggested subordinators. Write each sentence in two
ways, first using the subordinators between the two ideas and then rewriting each
sentence by placing the subordinator at the beginning of the two clauses.

1) because BC has a strong service industry


it is a favourite destination for tourists
a)

b)

2) even though my car is old and rusty


it takes me where I want to go
a)

b)

4) Prepositional Phrases that join ideas

Many times, a writer does not need a complete clause (subject + verb) to connect one
idea to another. Sometimes, small words called prepositions make it possible to
simplify the statement. Compare the following two sentences. Look at how the
preposition of works in the second sentence.

Because the test was scheduled for the next day, James stayed home to study.
Because of the test the next day, James stayed home to study.

Whenever a preposition is used to help join two parts of a sentence, the preposition is
never followed by a verb. Look at the following connecting prepositions to see more
examples of how verbs cannot be used after prepositions.

Prepositional Examples
Connectors
due to We didn’t go camping because of the rain.
because of
in spite of In spite of the rain, we went camping.
despite

C. Klassen & J. Robinson/Revised Winter 2009


9

328
Punctuation for Connecting Words GR6.32
In the following examples, you can see that prepositional phrase connectors use the
same punctuation rules as subordinators.

a) Do not use any punctuation when a prepositional phrase connector is in the


middle of sentence.

Margaret couldn’t call her husband on her cell phone because of a dead
battery.

b) Use a comma after the end of the prepositional phrase if the sentence begins
with the phrase:
Because of a dead battery, Margaret couldn’t call her husband on her cell
phone.

Exercise 6:

Join the following ideas using the suggested preposition phrases. Write each sentence
in two ways, first using the connectors between the two ideas and then rewriting each
sentence by placing the connectors at the beginning.

1) because of many people have jobs


BC’s strong service industry
a)

b)

2) In spite of my car takes me where I want to go


its age and rust

a)

b)

C. Klassen & J. Robinson/Revised Winter 2009


10

329
Punctuation for Connecting Words GR6.32
Practice what you have learned about punctuating connecting words by adding correct
punctuation to the following paragraph:

Exercise 7:
Although new technology creates new kinds of jobs new technology also usually
makes many existing jobs disappear for example when trains started to be used as a
primary form of transportation many jobs that depended on horses and oxen started
to disappear later automobiles became the major form of transportation as a result
railroad companies could no longer keep as many people employed another new kind
of technology that is leading to job loss is the digital camera companies that make
film are laying off workers because so many consumers are switching from cameras
that use film to cameras that take digital photographs in the same way cell phones
make public pay phones less profitable so telephone companies no longer need to
employ as many technicians to install and maintain pay phones while it is true that
technological change can produce new wealth and new jobs new inventions also tend
to lead to the loss of jobs

For more practice with punctuating connecting words, see:

Azar, B. (1999). Understanding and Using English grammar (3d Edition). Toronto:
Prentice Hall Regents. Chapters 16, 17, 19.

Benson, B. and Byrd, P. (1989). Improving the grammar of written English: The editing
process. Boston: Heinle and Heinle. pp. 49-74.

C. Klassen & J. Robinson/Revised Winter 2009


11

330
Punctuation for Connecting Words GR6.32
Douglas College Learning Centre

PUNCTUATION FOR CONNECTING WORDS – ANSWER KEY

1) Co-ordinating Conjuctions

Exercise 1:

1) Carlos lives in Canada, but he was born in Peru.


2) The car ran out of gas, so Bob had to walk two kilometers to find a gas station.
3) Maria could go to art school in New York, or she could stay in Vancouver to
look for a job.
4) Penelope likes to drive fast, but she got too many speeding tickets, so she lost
her driver’s license.

Exercise 2:

Mark loves to travel, but Amy doesn’t, so they often argue about where to spend
their vacation time. One summer, they decided to try something different. Mark went
to Thailand, but Amy stayed home. Mark’s brother, Alex, and sister-in-law, Jennifer,
went with him. They knew that they could only spend one week in Thailand, so they
had to decide what to do. They could all go scuba diving in Southern Thailand, or
Mark could go by himself on an elephant trek in Northern Thailand, and Alex and
Jennifer could meet him later in Bangkok. Mark chose the elephant trek, and he had
an exciting time. Amy was also having a good time back home. Sometimes she went
out with her friends at night, and sometimes her mother would visit her during the day.
She also liked to spend time by herself practicing her piano. Mark and Amy learned
that they could spent their vacations separately, yet they both could have a good time

C. Klassen & J. Robinson/Revised Winter 2009


12

331
Punctuation for Connecting Words GR6.32
2) Transitions

Exercise 3:

1) Marjorie is turning fifty this weekend. However, she says that she feels like a
twenty-one year old.
2) Jimmy and Kevin spend most of their time playing hockey. As a result, they
are excellent players.
3) You could take a bus. On the other hand, my father could drive you.
4) The garden tools should not be left outside. Otherwise, they will rust

Exercise 4:

1) The economy is strong. Interest rates, therefore, tend to be high.


2) Everybody at the party saw the stranger. No one, however, asked who he
was.
3) You could finish your degree next year. You could, on the other hand, choose
to work instead.
4) Agriculture contributes to greenhouse gases. Farm animals and rice fields, for
example, add a great deal of methane gas to the atmosphere.

4) Subordinators

Exercise 5:

1) because
a) Because BC has a strong service industry, it is a favorite destination for tourists.
b) BC is a favourite destination for tourists because it has a strong service industry.

2) Even though
a) Even though my car is old and rusty, it takes me where I want to go.
b) My car takes me where I want to go even though it is old and rusty.

C. Klassen & J. Robinson/Revised Winter 2009


13

332
Punctuation for Connecting Words GR6.32
4) Prepositional Phrases

Exercise 6:

1) because of
a) Many people have jobs because of BC’s strong service industry.
b) Because of BC’s strong service industry, many people have jobs.

2) In spite of
a) My car takes me where I want to go in spite of its age and rust.
b) In spite of its age and rust, my car takes me where I want to go.

Exercise 7:

Although new technology creates new kinds of jobs, new technology also usually
makes many existing jobs disappear. For example, when trains started to be used as a
primary form of transportation, many jobs that depended on horses and oxen started to
disappear. Later, automobiles became the major form of transportation; as a result,
railroad companies could no longer keep as many people employed. Another new kind
of technology that is leading to job loss is the digital camera. Companies that make film
are laying off workers because so many consumers are switching from cameras that
use film to cameras that take digital photographs. In the same way, cell phones make
public pay phones less profitable, so telephone companies no longer need to employ as
many technicians to install and maintain pay phones. While it is true that technological
change can produce new wealth and new jobs, new inventions also tend to lead to the
loss of jobs.

C. Klassen & J. Robinson/Revised Winter 2009


14

333
Punctuation for Connecting Words GR6.32

Exercise 7:
Although new technology creates new kinds of jobs, new technology also usually
makes many existing jobs disappear. For example, when trains started to be used as
a primary form of transportation, many jobs that depended on horses and oxen
started to disappear. Later, automobiles became the major form of transportation.
As a result, railroad companies could no longer keep as many people employed.
Another new kind of technology that is leading to job loss is the digital camera.
Companies that make film are laying off workers because so many consumers are
switching from cameras that use film to cameras that take digital photographs. In the
same way, cell phones make public pay phones less profitable, so telephone
companies no longer need to employ as many technicians to install and maintain pay
phones. While it is true that technological change can produce new wealth and new
jobs, new inventions also tend to lead to the loss of jobs.

C. Klassen & J. Robinson/Revised Winter 2009


15

334
1. For each of the following, choose the sentence with the correct punctuation.
A) You asked for forgiveness, he granted it to you.

B) You asked for forgiveness; he granted it to you.

C) You asked for forgiveness: he granted it to you.

D) You asked for forgiveness he granted it to you.

2.
A) We ask; therefore, that you keep this matter confidential.

B) We ask, therefore; that you keep this matter confidential.

C) We ask, therefore, that you keep this matter confidential.

D) We ask: therefore, that you keep this matter confidential.

3.
A) The order was requested six weeks ago; therefore, I expected the shipment to arrive by now.

B) The order was requested six weeks ago, therefore. I expected the shipment to arrive by now.

C) The order was requested: six weeks ago, therefore, I expected the shipment to arrive by now.

D) The order was requested six weeks ago: therefore, I expected the shipment, to arrive by now.

4.
A) The American flag has three colors; red, white, and blue.

B) The American flag has three colors: Red, white, and blue.

C) The American flag has: three colors. Red, white, and blue.

D) The American flag has three colors: red, white, and blue.

5.
A) She said she loved the book; so, I gave it to her.

B) She said she loved the book, so I gave it to her.

C) She said: she loved the book, so I gave it to her.

D) She said she loved the book: so I gave it to her.

6.
A) She has three dogs, two poodles and a beagle.

B) She has three dogs; two poodles and a beagle.

C) She has three dogs: two poodles and a beagle.

D) She has three dogs, two poodles, and a beagle.

7.

335
A) The orchestra, excluding the violin section, was not up to par.

B) The orchestra, excluding the violin section; was not up to par.

C) The orchestra; excluding the violin section, was not up to par.

D) The orchestra, excluding the violin section, was: not up to par.

8.
A) I have been to San Francisco, California, Reno, Nevada, and Seattle, Washington.

B) I have been to San Francisco California; Reno Nevada; and Seattle Washington.

C) I have been to San Francisco California, Reno Nevada, and Seattle Washington.

D) I have been to San Francisco, California; Reno, Nevada; and Seattle, Washington.

9.
A) I need a few items at the store, clothespins, a bottle opener, and napkins.

B) I need a few items at the store; clothespins, a bottle opener, and napkins.

C) I need a few items at the store: clothespins, a bottle opener, and napkins.

10.
A) I answered the phone; but no one seemed to be on the other end of the line.

B) I answered the phone: but no one seemed to be on the other end of the line.

C) I answered the phone, but no one seemed to be on the other end of the line.

1. For each of the following, choose the sentence with the correct punctuation.

336
Correct Answer: B You asked for forgiveness; he granted it to you.

Explanation: use a semicolon in place of a period to separate two closely related sentences.

2.
Correct Answer: C We ask, therefore, that you keep this matter confidential.

Explanation: do not use semicolons where commas can do the job.

3.
Correct Answer: A The order was requested six weeks ago; therefore, I expected the shipment to arrive by now.

Explanation: it is preferable to use a semicolon before, and a comma after, introductory words such as however,
therefore, for example when they introduce a complete sentence.

4.
Correct Answer: D The American flag has three colors: red, white, and blue.

Explanation: use a colon after a complete sentence to introduce any list.

5.
Correct Answer: B She said she loved the book, so I gave it to her.

Explanation: use a comma to separate two sentences joined by coordinating conjunctions such as and, but, or, so,
for, nor.

6.
Correct Answer: C She has three dogs: two poodles and a beagle.

Explanation: see Explanation 4.

7.
Correct Answer: A The orchestra, excluding the violin section, was not up to par.

Explanation: see Explanation 2.

8.
Correct Answer: D I have been to San Francisco, California; Reno, Nevada; and Seattle, Washington.

Explanation: use the semicolon to separate units of a series when one or more of the units contain commas.

9.
Correct Answer: C I need a few items at the store: clothespins, a bottle opener, and napkins.

Explanation: see Explanation 4.

10.
Correct Answer: C I answered the phone, but no one seemed to be on the other end of the line.

Explanation: see Explanation 5.

337
Punctuation Practice Test

INSTRUCTIONS: For questions 1-18 below, correct all punctuation errors by writing in the correct marks in their correct places.
Some sentences only need basic punctuation, like apostrophes and periods; others require more complex forms, such as dashes,
hyphens, colons, and ellipses.

1. The paper was clear pertinent and well written

2. Harry and Donnas honeymoon was just as frantic as their wedding

3. She won the race easily in fact she set a state record

4. I am recalling his story I believe as accurately as I can

5. The last year of the twentieth century is 2000 not 99

6. I expected a package this morning however I waited all day for it to arrive

7. Rainy days arent all that bad they provide the water crucial for all life

8. She witnessed a crime on her street she promptly locked her doors

9. We traveled to Rome Italy Athens Greece and Paris France

10. Shakespeare said it best Alls well that ends well

11. He is not well- liked although he says he is everyones friend

12. Sarah she had always loved animals took in the stray kitten

13. Certainly you may borrow my book Gary

14. The 1950s singer Patty Paige sang the novelty song How Much is That Doggie in the Window

15. Nearly all Americans own a Bible but few including scholars of literature have read it

16. Hmmm its a tough decision but Ill take the red one

17. Tuesday July 25 1967 is my birthday

18. I do the laundry make dinner and pick up the kids I should receive a medal for all of these chores

338
Punctuation Practice Test #2

1. Alas, poor Bill

2. Do you recall in the last grammar exercise, how he fell from his horse

3. You may remember that he cracked his skull as he landed on the rocky ground

4. Shall we resume the story

5. I seized Bill's lifeless wrist and felt for a pulse

6. Nothing

7. How could he have died so easily, by merely falling from a horse

8. What was I going to do

9. It was such a God-forsaken place

10. Help was at least a day's ride away

11. Suddenly I became aware of the large, icy drops of rain on the wind

12. There was nothing else I could do

13. I would have to make camp for the night

14. And what a very long and very cold night it was going to be

339
Punctuation Practice Test Answers

1. The paper was clear, pertinent, and well written.


Place commas to divide items in a series; "well written" would be hyphenated only if it preceded the noun.

2. Harry and Donna's honeymoon was just as frantic as their wedding.


Use only one apostrophe when showing joint possession of a compound subject.

3. She won the race easily; in fact, she set a state record.
She won the race easily -- in fact, she set a state record.
Use the semicolon to join two independent clauses using a conjunctive adverb; "in fact" is an interrupter; a dash would also be
acceptable because it leads into a surprise ending.

4. I am recalling his story, I believe, as accurately as I can.


"I believe" is an nonessential interrupter, so use commas to separate this from the rest of the sentence.

5. The last year of the twentieth century was 2000, not '99.
You must use an apostrophe to show the deletion of the "19" in the year 1999; use a comma to show negation of your previous
thought.

6. I expected a package this morning; however, I waited all day for it to arrive.
Use the semicolon to connect two independent clauses with a conjunctive adverb.

7. Rainy days aren't all that bad: they provide the water crucial for all life.
(alt.) Rainy days aren't all that bad -- they provide the water crucial for all life.
(alt.) Rainy days aren’t all that bad; they provide the water crucial for all life.
The colon is used to connect an independent clause with an explanation that follows; a dash would also be acceptable here; use
apostrophes to indicate the use of a contraction (which should not be used in formal writing). The use of a semi-colon to connect two
related complete sentences is also acceptable.

8. She witnessed a crime on her street; she promptly locked her doors.
(alt.) She witnessed a crime on her street -- she promptly locked her doors.
The semicolon simply connects two statements together to show their closeness; a colon would be inappropriate because the second
part does not explain the first part; a dash, however, would work because it progresses from a statement to an immediate, frantic
action.

9. We traveled to Rome, Italy; Athens, Greece; and Paris, France.


Normally, you would use commas to separate these items in a list; however, we need a stronger comma to separate the cities/states
from the other cities/states, so use the semicolon to represent a "super comma."

10. Shakespeare said it best: "All's well that ends well."


Use the quotes to encompass Shakespeare's statement; use the colon to lead into the quote because you have just stated an
independent clause; explanations follow colons, and this quote explains what Shakespeare said very directly; use the apostrophe to
signify a contraction used in the quote.

11. He is not well-liked, although he says he is everyone's friend.


Use a comma to separate the two independent clauses; use the apostrophe to indicate possession of a singular noun ("everyone").

12. Sarah -- she had always loved animals -- took in the stray kitten.
Use the dashes to indicate an abrupt change of thought, as in this example where the speaker changes his mind about the direction his
sentence would take; the dashes also help to organize the subject with its verb. Commas and parenthesis also work, but they are not
the strongest option.

13. Certainly, you may borrow my book, Gary.


Use a comma to separate introductory elements; use another to separate the recipient of the direct address ("Gary").

14. Patty Paige sang the novelty song "How Much is That Doggie in the Window?"
Place the song title within quotation marks since it is a minor title that can be compiled in an anthology; be sure to place the question

340
mark inside the quotes, since the title asks a question; you do not need a comma to separate the word "song" from the song's title -- it
is essential information.

15. Nearly all Americans own a Bible, but few, including scholars of literature, have read it.
(alt.) Nearly all Americans own a Bible, but few (including scholars of literature) have read it.
Use a comma to separate two independent clauses joined by a subordinating conjunction; further separate the phrase "including
scholars of literature" with either commas or parentheses to clarify that this information is nonessential.

16. Hmmm ... it's a tough decision, but I'll take the red one.
You can use an ellipses to indicate a thoughtful pause or the passing of silent time; use apostrophes to signify that two contractions
are being used in this very informally written sentence; use a comma to separate two independent clauses used with a conjunction.

17. Tuesday, July 25, 1967, is my birthday.


Use commas to separate days from dates, dates from years, and years from the rest of the sentence; you would need to place at least
two commas no matter what two parts of this complete date you used:
Tuesday, July 25, is my birthday.
July 25, 1967, is my birthday.

18. I do the laundry, make dinner, and pick up the kids -- I should receive a medal for all of these chores!
(alt.) I do the laundry, make dinner, and pick up the kids; I should receive a medal for all of these chores!
Use commas to separate items listed in a series; use the dash (or a semicolon) to connect the two clauses; a colon would not work in
this sentence because the second part does not explain the first part (actually, if you look carefully, the first part explains the second
part, but this is not the way we use a colon); you may use an exclamation point here because this person seems to be shouting -- keep
in mind that this sentence is informally written, so you can get away with exclamations here.

Answers Punctuation Practice Test #2


1. Alas, poor Bill!
2. Do you recall in the last grammar exercise, how he fell from his horse?
3. You may remember that he cracked his skull as he landed on the rocky ground.
4. Shall we resume the story?
5. I seized Bill's lifeless wrist and felt for a pulse.
6. Nothing!
7. How could he have died so easily, by merely falling from a horse?
8. What was I going to do?
9. It was such a God-forsaken place!
10. Help was at least a day's ride away.
11. Suddenly I became aware of the large, icy drops of rain on the wind.
12. There was nothing else I could do.
13. I would have to make camp for the night.
14. And what a very long and very cold night it was going to be!

"A woman, without her man, is nothing."


"A woman: without her, man is nothing."
A clever dog knows its master. not A clever dog knows it's master.
I saw a man eating lobster. not I saw a man-eating lobster.
The butler stood in the doorway and called the guests' names. not The butler stood in the doorway and called the guests names.
At summer camp I missed my dog, my little brother, the odor of my dad's pipe, and my boyfriend. not At summer camp I missed
my dog, my little brother, the odor of my dad's pipe and my boyfriend.
WANTED: piano to replace daughter's lost in fire not WANTED: piano to replace daughters lost in fire
FATHER-TO-BE STABBED TO DEATH IN STREET not FATHER TO BE STABBED TO DEATH IN STREET
To my parents, the Pope, and Mother Teresa not To my parents, the Pope and Mother Teresa

341
Connectors and LINKERS
PURPOSE or AIM
to / not to + infinitive He went to Scotland to learn English.
in order (not) to He went to Scotland in order to / so as to learn English.
so as (not) to I left home early in order not to / so as not to be late for the
appointment
so that + sentence They’ll make an appointment so that we won’t have to wait.
in order that*more formal (usually with I left a note so that he would know where to find me.
modal verbs)
for + gerund A knife is used for cutting.
(speaking about Newspapers are used for reading.
its function)
REASON or CAUSE
because + sentence I couldn’t go then because I had to study.
as Since there wasn’t enough show, they didn’t go skiing.
since*more formal As I am very busy studying for my exams, I won’t go with you.
for*very formal Sing your praises to the King for he is the King of kings.
for + noun / They put him in prison for stealing forbidden books. The coach
gerund punished the player for calling the referee a thief.
because of + noun The game was canceled because of the rain.
due to Due to his bad attitude, he lost his job.
owing to*more formal She walks with a limp owing to a childhood injury.
Thanks to the generosity of many donors, we have made major
thanks to progress over the past years.
RESULT
so*between two sentences + sentence The party was boring, so I left early.
That’s why + sentence The party was boring. That’s why I left early.
As a result, Someone called in a bomb threat. As a result, school was
For this reason, cancelled.

Therefore, I have long been interested in science and animals. Therefore, I


have decided to study biology in college.
Consequently, I think, therefore I am.
as a result of + noun As a result of the war, the prices of oil and bread went up
enormously.
so +adj/adverb+ that + sentence We were so impatient that we couldn’t wait for them.
such +noun + that They had such a big dog that nobody dared to go near their
so many/much + noun house.
+ that There were so many people that we couldn’t get into the pub.
CONDITION
if + sentence If you drink, don’t drive. If you don’t study, you won’t pass.
unless (if not) (No Future!) Unless you study, you won’t pass.
as long as I will go as long as it is free.
providing that + sentence She can come providing that you sleep in separate rooms.
provided (that) * formal (No Future!) They may do whatever they like provided that it is within the
law.
on condition that formal The bank lent the company 100,000 pounds on condition that
they repaid the money within six months.
otherwise + sentence We’d better send it express, otherwise it’ll take days. (If we do
not send it express, it will take days.)
whether … or…. + sentence I haven't decided yet whether I will attend tomorrow's meeting
or not.
Whether he leaves or stays is of absolutely no importance to me.
The decision depends on whether Alice wants to make the
investment (or not).

342
CONTRAST
but + sentence He wasn’t experienced, but he got the job.
However, Nevertheless, + sentence He spent all his spare money. However, he still had his credit
Nonetheless, card.
It cost a fortune to renovate our house; nevertheless, it was
worth it.
although + sentence Although /Even though it was raining, he decided to walk to
even though* stronger the station.
though* less formal Although /Though the car is old, it is still reliable.
The car is old. It’s still reliable, though.
in spite of + noun/gerund In spite of / Despite having no money, he got a ticket.
despite He got a ticket despite / in spite of not having any money.
in spite of the fact that + sentence In spite of the fact that he hadn’t any money, he got a ticket.
despite the fact that He got a ticket despite the fact that he hadn’t got any money.
unlike + noun Unlike your mum, hers is more distant and colder.
on the one hand + sentence On the one hand, reading is very enriching.
on the other hand On the other hand, it can also be boring when you don’t like
what you must read.
while + sentence Some married couples argue all time, whereas / while others
whereas never do.
In the UK the hottest month of the year is usually July, whereas
in southern Europe the hottest period is usually in August.
Adding
INFORMATION
and in the sentence Ann loves dance and also music.
too She likes singing too.
also
as well as in the sentence She must follow the rules as well as use good dance technique.
Also, + sentence She must follow the rules. Also, she must use proper dance
technique.
Moreover, * very formal The acceleration performance of this car is very poor; moreover,
Furthermore, * formal its fuel consumption is extremely high.
besides + sentence I wasn’t interested in the talk, besides I was feeling very tired.
+ noun / No one knows besides you.
pronoun
In addition, + sentence In addition to our previous order, we want some more books.
Additionally,
We made a previous order. In addition, we want to ask for some
In addition to + noun more books.
not only…,but also… + noun/gerund When writing, consider not only your topic, but also your
audience.
both … and … + noun/gerund Children like eating both pizza and chiken.
neither … nor … Both Mark and Jennifer like reading.
either … or … Neither Mark nor Jennifer like smoking.
Either you go or you stay. Make up your mind now!
TIME
when + sentence When I came home, Mary was waiting for me.
as soon as As soon as she saw me coming, she stood up and smiled.
while While she was telling me the news, I was thinking about the
options I had.
after + noun/gerund After finishing dinner she went up to her room.
before + sentence She went up to her room after she had finished dinner.
Jason read the manual before the installation / before installing
the system / before he installed the system.

343
EXERCISES ON LINKERS.

1. REWRITE THESE SENTENCES USING THE WORDS GIVEN IN BRACKETS.

a. Although the rain was falling heavily, they continued with the football match. (in
spite of)

b. Grandma cannot hear what you say because she is very deaf. (so..............that)

c. She was such a tall girl that I found it difficult to dance with her. (so..............that)

d. Many rivers have burst their banks because there has been a heavy rainfall.
(because of)

e. The driver instructor took Dan onto the main road because he wanted him to get
experience in high speed driving. (so that)

f. We couldn’t go to the concert because the tickets were too expensive. (since)

g. In spite of the difficulty of the questions, he answered them all. (although)

h. I’ve told you this story because I don’t want you to forget the old days. (so that)

I. He is so energetic that he never looks tired. (such)

j. As there were a lot of people, they had to open all the doors. (so many)

k. Although we had revised everything a hundred times, everything went wrong.


(despite)

l. He left home early because he was afraid of missing the plane. (so as)

m. Some people enjoy city life. Others prefer the quiet of the country. (whereas)

n. Everybody stayed near the fire since the day was very cold. (because of)

344
1. Although she was an expert swimmer, she drowned. (in spite of)

2. The school was closed down due to low inscription. (Because )

3 Helen went to the market so that she could buy some vegetables. (in order)

4 The match was cancelled because it was raining. (due to)

5 Despite having a terrible headache, he went to the office. (although)

6 He loves going to the theatre while his wife prefers to stay at home. (in contrast

to)

7 I sat by the window in order to see the landscape. (so that)

8 As well as being en excellent painter, she writes poems and plays the piano.

(Moreover)

9 Although Ann isn't very attractive, she is very popular. (However)

10 Seeing that we'd run out of milk, we went to the supermarket. (as)

11. Contrary to my neighbours, I don't like using the lift. (whereas)

12. As well as renewing the kitchen, she redecorated her bedroom. (In addition,)

13. Although the city has a 50 kph limit, people are often fined for exceeding the

speed limit. (Therefore)

345
14. Many sportsmen are disqualified as a consequence of their illegal use of

drugs. (consequently)

15. John won the race. Nevertheless, he didn't feel happy. (despite)

16. We need to increase sales. Therefore, we'll start a new advertising campaign.

(seeing that)

17. Although he was a millionaire, he behaved as an ordinary man. (YET)

18. Since you are her elder brother, you must take care of her. (BECAUSE)

19. She's an excellent secretary. However, she has never been promoted.

(despite)

20. She opened the car window so that she could get some fresh air. (to)

21. I opened the window due to the heat. (BECAUSE)

22. We didn't go swimming because the water was very dirty. (so....................that)

23. The government passed a new law in order to control terrorism. (so that)

24.Despite having some problems, she finally passed the test. (even though)

25 I enjoy going to the beach. My husband does not. (while)

26 As a consequence of his father's illness, he couldn't go to work. (As a result)

27 Although he was an excellent student, he didn't get the grant. (Nevertheless)

346
28 They lost the match due to the bad weather. (as a result of)

29 Although she wasn't hungry, she ate a few biscuits. (in spite of)

30 She missed the bus so she had to take a taxi. (since)

31.Although she was very intelligent, she pretended not to understand. (but)

32 In addition to buying a new coat for his wife, he bought himself a cardigan.

(besides)

33 .Sheila went to the garage to have her car serviced. (so that)

34 .He broke his left hand as a result of a skiing accident. (due to )

35 I like Harry Potter's books. My brother does not. (contrary to)

36 She didn't telephone John because she didn't have his number. (so)

37 Although he had excellent qualifications he didn't get the job. (YET)

38 They got lost in London because they didn't have a street map. (as)

39 Although she was very tired, she kept on working. (DESPITE)

40.She lost her purse, so she went to the police station. (BECAUSE)

A few more exercises? Go to this link.

http://www.xtec.cat/~ogodoy/sac/rephrasing/rephrexercises.htm

347
348
It was late, so I decided to take a taxi
home.
So
His wife left him. As a result, he became
Consequently / As a result / very depressed.
RESULT Therefore
The storm was soterrible that the roofs
So / Such (a) ……….. that/ were ripped off.
that’s why
He was such ahandsome man thatshe fell
in love immediately.

Since we arrived late, all the best seats


Because / Since / As
CAUSE had been taken.
AND
Because of / Due to/thanks
REASON We were unable to go by train because
to /owing to
ofthe rail strike.

Although / Even though / Although the car is old, it is still reliable.


But
CONTRAST Despite / In spite ofthe rain, I went for a
and Despite / In spite of walk.
consession
However / Nevertheless/ On Buying a house is expensive. However, it
the other hand is a good investment.

While I was driving home, I saw a terrible


When / While / After /
accident.
TIME Before / As soon as / Until /
By the time
He went out after he had finished work.

349
I don´t feel like going out
tonight. Besides, there is a good film on.
And

She writes novels. In addition,


In addition / Furthermore /
she also writes poetry.
Moreover / Besides
ADDITION
They robbed a bank as well as a post
In addition to / As well as
office.

Also / Too
They adore Indian food. I like Indian
food, too.

In order (not) to / So as Anita phoned me in order to recommend a


(not) to + inf film she had just seen.
PURPOSE
So that + subject + modal I left early so that I would meet Dan at
verb + inf the cinema on time.

1. FILL IN THE BLANKS WITH A SUITABLE


LINKER
1. ………………………….. their parents opposition, Tom and Ann decided to get
married.
2. We’ll leave the note on the table……………………….he can see it.
3. ………………………………. I have a great respect for him, I don´t particularly like
him.
4. Tim isn´t suitable for the job. He´s too old. ………………………………, he isn´t
interested.

350
5. …………………………….. there are no more questions to discuss, we can finish the
meeting.
6. Brian is leaving the company ……………………………… his age.
7. The sweater was ……………… big ……………….. I couldn’t wear it.
8. He worked for the same company all his life …………………………….. he retired.
9. Linkers are quite difficult to learn. ……………………….., they are worth studying.
10. The temperature in Saudi Arabia can reach 50º C. ………………….. , a lot of
business is done early in the day.
11. I failed the exam precisely ………………………….. I didn´t have time to study.
12. John, ………………………. you speak French, I wondered if I could ask you
some words.
13. Nora eats green vegetables ……………………………… they are good for her
health.
14. It was ………………………. wonderful view …………………………. we were left
speechless.
15. ……………………… the teacher was out of the class, some pupils wrote a
message on the board.
16. There´s time to get a sandwich from the cafeteria……………………….. the
next class begins.
17. The president was very unpopular. ………………………, .his resignation did
not come as a surprise.
18. Tim has to go to the dentist …………………………… an infected tooth.
19. I didn’t tell you…………………………. Make you angry.
20. It was …………………………..good cake …….I couldn’t help eating it.
21. I´ll look after the children ……………………… you are making dinner.
22. She walked carefully………………………….. the streets were covered in ice.

&nbsp;

2. REWRITE THE FOLLOWING SENTENCES


USING THE CONJUNCTIONS IN BRACKETS.

351
1. Since Bill had lived in Spain for six years, he spoke Spanish fluently. (so)

2. I am sick and tired of people phoning me during my favourite TV programme.


( while)

3. I don´t know Helen very much. Nevertheless, I like her. (in spite of )

4. The weather was very hot, so we all felt tired. (because)

5. If you don’t want to wake up the baby, speak softly. (so as to)

6. We are old but we enjoy life. (despite)

7. The game was so easy that even a child could play it. (such)

8. They opened the window because they didn’t want the smoke to go in the
kitchen. (so that)

9. He speaks Chinese and he can also read it. (furthermore)

10. I‘m going to do more sport because I want to lose some weight. (in order
to)

11. I worked hard although I didn´t get the results I expected. (nevertheless)

12. It’s such a fantastic story that I can’t believe it. (so)

13. They acted quickly because they wanted to avoid confusion. (so that)

14. You seem to know so much about it. For this reason, I would like to hear
your version of the incident. (as)

15. You may not feel like doing it but you should visit your aunt. (however)

16. Let’s take a taxi. I want to arrive in time. (so that)

17. The paintings were so beautiful that I went to the museum twice. (such)

352
353
Language: Style and Tone

1. The author wants to use understatement as a rhetorical style. Which choice best accomplishes this goal?

You took my brand-new car without asking, drove it 800 miles, and scraped the entire front bumper? Yeah, you could say I’m
extremely upset!

A. NO CHANGE
B. This makes me so happy!
C. Yeah, you could say I'm a little annoyed!
D. I've never been angrier in my life!

2. The author wants to use overstatement in order to emphasize the speaker’s exhaustion. Which choice best accomplishes this
goal?

I’ve been working on legal briefs all day. I’m so exhausted I could sleep for hours!

A. NO CHANGE
B. don't think I can sleep right now!
C. won't be able to sleep for hours!
D. could sleep for years!

3. Which choice best expresses enthusiasm consistent with the tone of a formal engagement announcement?

Mr. and Mrs. Joel Andersen of Sunnyville are pleased to announce the engagement of their daughter, Stephanie, to Guillermo
Torres, son of Mr. and Mrs. Carlos Torres of Crater City. Stephanie is a graduate of University of Utah and is a social worker at
St. Paul’s Hospital. Guillermo graduated from Beckman College and is employed with Kane Pharmaceuticals. The families are
eagerly looking forward to an October wedding.

A. NO CHANGE
B. are required to announce
C. are announcing
D. are totally stoked to announce

4. In order to emphasize the contrast between a sunrise and the author’s attitude, the latter wants to use two adjectives that are
derived from the same root word. Which choice best accomplishes this goal?

Sunrises are, I admit, generally very majestic. But usually (if I have my way) anything that occurs before 8 a.m. is wasted on me.
When the sun begins his incredible ascent, I prefer to be an inglorious huddle of sheets, crusted with sleep and willfully blind.

A. NO CHANGE
B. glorious ascent, I prefer to be an inglorious
C. glorious ascent, I prefer to be a ragged
D. stunning ascent, I prefer to be a pitiful

5. The writer wants to use exaggeration in order to emphasize the relative insignificance of a mistake. Which choice best
accomplishes this goal?

My friend Joseph, we are told, has committed the oversight of forgetting to bring his homework to class. This demonstrates a
multitude of failings, personal and academic, which will forever bar him from the ranks of human society.

A. NO CHANGE
B. committed the grave crime
C. made the inconvenient error
D. unfortunately made the mistake

354

You might also like